186
ФЕДЕРАЛЬНОE АГЕНТСТВО ПО ОБРАЗОВАНИЮ НОВОСИБИРСКИЙ ГОСУДАРСТВЕННЫЙ УНИВЕРСИТЕТ Д. В. СТАСЬ, В. Ф. ПЛЮСНИН КВАНТОВАЯ МЕХАНИКА МОЛЕКУЛ ЧАСТЬ 1 АТОМ Новосибирск 2008

КВАНТОВАЯ МЕХАНИКА МОЛЕКУЛ · PDF file2 ББК В22.31 УДК 544.11+539.183 isbn 978-5-94356-719-3 Стась Д.В., Плюснин В.Ф. Квантовая

Embed Size (px)

Citation preview

Page 1: КВАНТОВАЯ МЕХАНИКА МОЛЕКУЛ · PDF file2 ББК В22.31 УДК 544.11+539.183 isbn 978-5-94356-719-3 Стась Д.В., Плюснин В.Ф. Квантовая

ФЕДЕРАЛЬНОE АГЕНТСТВО ПО ОБРАЗОВАНИЮ

НОВОСИБИРСКИЙ ГОСУДАРСТВЕННЫЙ УНИВЕРСИТЕТ

Д. В. СТАСЬ, В. Ф. ПЛЮСНИН

КВАНТОВАЯ МЕХАНИКА МОЛЕКУЛ

ЧАСТЬ 1

АТОМ

Новосибирск2008

Page 2: КВАНТОВАЯ МЕХАНИКА МОЛЕКУЛ · PDF file2 ББК В22.31 УДК 544.11+539.183 isbn 978-5-94356-719-3 Стась Д.В., Плюснин В.Ф. Квантовая

2

ББК В22.31УДК 544.11+539.183

ISBN 978-5-94356-719-3

Стась Д.В., Плюснин В.Ф. Квантовая механика молекул. Часть 1:Атом / Новосиб. гос. ун-т. Новосибирск, 2008, 186 с.

Учебное пособие написано по материалам спецкурса «Квантоваямеханика молекул», в течение ряда лет читавшегося студентам Кафедрыхимической и биологической физики Физического факультета НГУ.Основной акцент сделан на получение практических навыков описаниястроения молекулярных систем на языке квантовой механики с исполь-зованием симметрии задачи. Главное внимание уделяется построениюматематической формализации взятой из реальной жизни задачи, разви-тию методов решения математической задачи и построению по возмож-ности наглядной картины, иллюстрирующей процесс решения задачи.Подробно рассматриваются стандартные приближения, обсуждаются де-лаемые при этом допущения, их происхождение и обоснование, а такжесвязанные с ними ограничения. Пособие написано на достаточно высо-ком уровне строгости и предполагает знание основ квантовой механики,органической химии, магнитного резонанса и теории групп. В первойчасти обсуждаются вопросы, связанные с электронным строением атомаи двухатомной молекулы.

Пособие предназначено для студентов и аспирантов, специализирую-щихся в области химической физики и физической химии, и может бытьполезно научным сотрудникам, работающим на стыке физики и химии.

РецензентПроф. П.А. Пуртов

c© Новосибирский государственныйуниверситет, 2008

ISBN 978-5-94356-719-3 c© Д.В. Стась, В.Ф. Плюснин, 2008

Page 3: КВАНТОВАЯ МЕХАНИКА МОЛЕКУЛ · PDF file2 ББК В22.31 УДК 544.11+539.183 isbn 978-5-94356-719-3 Стась Д.В., Плюснин В.Ф. Квантовая

Оглавление

Предисловие 5

1 Одноэлектронный атом 7

1.1 Уравнение Шредингера для атома водорода . . . . . . . . . . 71.2 Анализ радиальных функций . . . . . . . . . . . . . . . . . . 101.3 Анализ угловых функций . . . . . . . . . . . . . . . . . . . . 161.4 Базис вещественных угловых функций lζ . . . . . . . . . . . 191.5 Полные функции Ψnlm (r,ϑ ,ϕ), nlζ (r,ϑ ,ϕ) . . . . . . . . . . . 241.6 Базис гибридных орбиталей spn . . . . . . . . . . . . . . . . 27

2 Многоэлектронный атом 35

2.1 Сродство к электрону атома водорода . . . . . . . . . . . . . 352.2 Пределы применимости водородных функций . . . . . . . . 382.3 Упрощающие предположения . . . . . . . . . . . . . . . . . . 392.4 Слэйтеровские орбитали . . . . . . . . . . . . . . . . . . . . 402.5 Другие стандартные базисы . . . . . . . . . . . . . . . . . . 432.6 Электронная конфигурация атома . . . . . . . . . . . . . . . 452.7 Химические свойства атома . . . . . . . . . . . . . . . . . . . 502.8 Потенциал ионизации и сродство к электрону . . . . . . . . 54

3 Классификация термов атома 57

3.1 Иерархия взаимодействий в атоме . . . . . . . . . . . . . . . 573.2 Атом в приближении LS-связи . . . . . . . . . . . . . . . . . 603.3 Группа эквивалентных электронов . . . . . . . . . . . . . . . 643.4 Несколько групп эквивалентных электронов . . . . . . . . . 673.5 Более громоздкие конфигурации . . . . . . . . . . . . . . . . 693.6 Более чем наполовину заполненные оболочки . . . . . . . . 713.7 Атом в приближении j j-связи . . . . . . . . . . . . . . . . . 73

Page 4: КВАНТОВАЯ МЕХАНИКА МОЛЕКУЛ · PDF file2 ББК В22.31 УДК 544.11+539.183 isbn 978-5-94356-719-3 Стась Д.В., Плюснин В.Ф. Квантовая

4 ОГЛАВЛЕНИЕ

3.8 Эквивалентные электроны в пределе j j-связи . . . . . . . . 753.9 Разбиение на эквивалентные группы . . . . . . . . . . . . . 79

4 Многоэлектронные волновые функции 81

4.1 Процедура сложения моментов . . . . . . . . . . . . . . . . . 814.2 Детерминанты Слэйтера и спин-орбитали . . . . . . . . . . . 884.3 Функции |L,ML;S,MS〉 в пределе LS-связи . . . . . . . . . . . 924.4 Функции |L,S,J,Jz〉 в пределе LS-связи . . . . . . . . . . . . 994.5 Сведение вычислений к минимуму . . . . . . . . . . . . . . . 1024.6 Функции для термов предела j j-связи . . . . . . . . . . . . 106

5 Преобразования волновой функции 113

5.1 Преобразования изолированного атома . . . . . . . . . . . . 1135.2 Преобразования координат . . . . . . . . . . . . . . . . . . . 1155.3 Преобразование сферических гармоник . . . . . . . . . . . . 1205.4 Преобразование многоэлектронной функции . . . . . . . . . 1245.5 Термы двухатомной молекулы AB . . . . . . . . . . . . . . . 1275.6 Термы атомов и термы молекул . . . . . . . . . . . . . . . . 1295.7 Термы молекулы FO . . . . . . . . . . . . . . . . . . . . . . . 1345.8 Учет спин-орбитального взаимодействия . . . . . . . . . . . 136

6 Термы гомоядерной молекулы 141

6.1 Инверсия в центре молекулы . . . . . . . . . . . . . . . . . . 1416.2 Действие инверсии на волновую функцию . . . . . . . . . . 1436.3 Пространства состояний с нечетными Λ . . . . . . . . . . . 1466.4 Пространства состояний с четными Λ . . . . . . . . . . . . . 1496.5 Пространства состояний с Λ = 0 . . . . . . . . . . . . . . . . 1546.6 Правила Вигнера—Витмера . . . . . . . . . . . . . . . . . . . 1556.7 Одинаковые атомы в разных состояниях . . . . . . . . . . . 1586.8 Опознание задачи на сведение атомов . . . . . . . . . . . . . 1616.9 Учет спин-орбитального взаимодействия . . . . . . . . . . . 1656.10 Классификация термов и правила отбора . . . . . . . . . . . 169

A Задачи 177

A.1 Общие свойства связанной системы . . . . . . . . . . . . . . 177A.2 Одноэлектронный атом . . . . . . . . . . . . . . . . . . . . . 178A.3 Многоэлектронный атом . . . . . . . . . . . . . . . . . . . . 179A.4 Классификация состояний . . . . . . . . . . . . . . . . . . . 181A.5 Многоэлектронные волновые функции . . . . . . . . . . . . 182A.6 Корреляция атомных и молекулярных термов . . . . . . . . 183

Page 5: КВАНТОВАЯ МЕХАНИКА МОЛЕКУЛ · PDF file2 ББК В22.31 УДК 544.11+539.183 isbn 978-5-94356-719-3 Стась Д.В., Плюснин В.Ф. Квантовая

Предисловие

Курс квантовой механики молекул занимает особое место в цик-ле физикохимических дисциплин, предлагаемых на кафедре химическойи биологической физики Физического факультета НГУ. Приступающиек освоению курса студенты-физики уже имеют за плечами годовые об-щие курсы квантовой механики и методов математической физики с эле-ментами теории групп, семестровые курсы атомной физики, спектроско-пии магнитного резонанса и органической химии, в магистратуре же импредстоит освоить курсы теории элементарных химических реакций иквантовой химии. Таким образом, студенты к началу курса имеют заплечами значительную теоретическую подготовку, а на старших курсахим предложат дисциплины, охватывающие самые современные приложе-ния квантовой механики в задачах химической физики. Однако при этомоказывается, что при такой солидной базе студенты часто испытываютсерьезные трудности при решении практических задач — слишком ве-лик получается разрыв между чистыми и точно решаемыми задачамиобщего курса квантовой механики и абстрактной теории групп и совре-менными методами вычислительной квантовой химии, с одной стороны,и требующими большого количества неочевидных приближений, частоочень громоздких при лобовом решении реальными задачами. Нередкостуденты оказываются в тупике, столкнувшись с физической, не говоряо химической постановке задачи. Данный курс призван заполнить ука-занный пробел и дать студентам навыки решения практически важныхзадач прикладной квантовой механики.

Курс построен в формате расширенных интерактивных семинаров сосновным акцентом на решение задач и существенно опирается на ужеизвестный студентам материал, что в значительной степени определилостиль изложения и подбор материала для учебного пособия. Ряд вопро-сов, традиционно освещаемых в подобного рода курсах, либо опущен

Page 6: КВАНТОВАЯ МЕХАНИКА МОЛЕКУЛ · PDF file2 ББК В22.31 УДК 544.11+539.183 isbn 978-5-94356-719-3 Стась Д.В., Плюснин В.Ф. Квантовая

6 ПРЕДИСЛОВИЕ

вовсе, либо рассмотрен с точки зрения методов решения задач. Мно-гие рассматриваемые в курсе задачи достаточно сложны (естественнымограничителем сложности здесь выступала возможность решить задачуза одно занятие), а их решение доводится до конца с обсуждением всехключевых моментов, без отсылок к «очевидности» или «неинтересности».Главное внимание уделяется трем сторонам проблемы: построению мате-матической формализации взятой из реальной жизни задачи, развитиюметодов решения математической задачи, построению по возможностинаглядной картины, иллюстрирующей процесс решения задачи. Подроб-но рассматриваются стандартные приближения, на которые приходитсяидти для практического решения задач, обсуждаются делаемые при этомдопущения, их происхождение и обоснование, связанные с ними ограни-чения и подводные камни. Рассматриваются приближение LS и j j-связив атоме, приближения МО ЛКАО и Хюккеля в молекулах, приближениекристаллического поля в координационных соединениях. Обсуждаютсяоптимизированные для практического применения процедуры сложениямоментов в различных вариантах, графического решения детерминантовХюккеля высокой размерности, приложений теории групп для классифи-кации состояний и снижения размерности задачи. Подчеркивается необ-ходимость научиться видеть симметрию задачи и бережно относиться кней, формировать для себя наглядные образы, помогающие упростить за-дачу до начала вычислений, а иногда и сразу выписать ответ. Через веськурс проходит идея об иерархии имеющихся в системе взаимодействийи последовательной симметризации базиса задачи, отмечаются и под-черкиваются глубокие аналогии между решением задач об электронномстроении и классификации состояний таких разных систем, как изоли-рованный атом, двухатомная молекула, многоатомная молекула, ион вкристаллическом поле.

Пособие написано на основе семестрового цикла лекций и семинаров,в течение нескольких лет читаемого для студентов кафедры химичес-кой и биологической физики Физического факультета НГУ. В основуцикла положен переработанный общий курс «Строение вещества», раз-работанный Н. М. Бажиным и В. Ф. Плюсниным и в течение многих летчитаемый студентам ФЕН НГУ. В первой части пособия обсуждаются во-просы, связанные со строением атома, и непосредственно примыкающиек ним вопросы строения двухатомных молекул.

Page 7: КВАНТОВАЯ МЕХАНИКА МОЛЕКУЛ · PDF file2 ББК В22.31 УДК 544.11+539.183 isbn 978-5-94356-719-3 Стась Д.В., Плюснин В.Ф. Квантовая

Глава 1

Одноэлектронный атом.

Геометрические свойства и

ихнаглядноепредставление

Изучение курса квантовой механики молекул мы начнем с рассмот-рения свойств простейших квантовомеханических систем, имеющих от-ношение к молекулам — атома водорода и ряда родственных ему систем.Анализ имеющихся для этих задач компактных аналитических решенийпомогает представить, что происходит в более сложных системах. Цельюнаших дальнейших рассуждений будет не получить еще раз известныеточные решения, а проанализировать, как устроены волновые функцииатома водорода и как их можно качественно восстановить без полногорешения. Нашей задачей будет построить цепочку достаточно простыхрассуждений и сопровождающих их наглядных образов, пригодных длякачественного анализа задачи.

1.1 Уравнение Шредингера для атома

водорода

В качестве отправной точки возьмем стандартное уравнение Шредин-гера для волновой функции Ψ:

HΨ = EΨ. (1.1)

Page 8: КВАНТОВАЯ МЕХАНИКА МОЛЕКУЛ · PDF file2 ББК В22.31 УДК 544.11+539.183 isbn 978-5-94356-719-3 Стась Д.В., Плюснин В.Ф. Квантовая

8 ГЛАВА 1. ОДНОЭЛЕКТРОННЫЙ АТОМ

При этом сразу неявно делается два довольно серьезных допущения:первое — что нашу систему можно описать волновой функцией, то естьчто она всегда находится в чистом состоянии и не связана ни с какими«внешними» степенями свободы, и второе — что наша система нереля-тивистская. Такого рода неявные допущения делаются при выписываниипрактически любого физического выражения в теории молекул, и о нихнужно помнить, чтобы ощущать практические пределы применимостиполучаемых результатов.

Стандартный гамильтониан для рассматриваемой системы имеет вид:

H = − h2

2m4− e2

r, (1.2)

и соответствует движению электрона массы m и заряда −e в кулоновомполе бесконечно тяжелого (неподвижного) точечного ядра заряда +e.Искомое уравнение принимает вид:

[

− h2

2m4− e2

r

]

Ψ = E Ψ. (1.3)

Прежде всего давайте приведем наше уравнение к безразмерному ви-ду. При этом резко упрощается запись и снижается вероятность матема-тической ошибки при преобразованиях, а при обратном восстановленииразмерного решения на выходе мы сможем получить описание не одной,а целого класса родственных систем. Из входящих в уравнение (1.3)размерных констант h,m,e можно составить две независимые константы:размерности длины (a0) и энергии E0. Удобный способ получить их —оценить из соотношения неопределенности размер и энергию атома водо-рода в основном состоянии. Ищем минимум полной энергии при условииp ∼ h/r:

E =p2

2m− e2

r∼ h2

2mr2 −e2

r→ min ⇒

rmin =h2

me2 = a0; Emin = E(a0) = −12

(me4

h2

)

= −12

E0.

(1.4)

Боровский радиус a0 = 0,53A дает нам характерный масштаб размеров ватоме, характерная величина энергии E0 = 27,2 эВ называется «Хартри»и используется в качестве стандартной единицы энергии в квантовойхимии. Более привычная величина E0/2 = 13,6 эВ носит имя «Ридберг»и служит стандартной единицей энергии в атомной спектроскопии.

Page 9: КВАНТОВАЯ МЕХАНИКА МОЛЕКУЛ · PDF file2 ББК В22.31 УДК 544.11+539.183 isbn 978-5-94356-719-3 Стась Д.В., Плюснин В.Ф. Квантовая

1.1 УРАВНЕНИЕ ШРЕДИНГЕРА ДЛЯ АТОМА ВОДОРОДА 9

Поделим наше уравнение на E0 и введем безразмерную энергию ε =EE0

:

[

− h2

2mh2

me4 4− e2 h2

me4

1r

]

Ψ = ε Ψ. (1.5)

Введя теперь единицу длины a0, получим[

−a20

r42

− a0

r

]

Ψ = ε Ψ, (1.6)

где r4 — исходный оператор Лапласа в размерных координатах ~r. Вво-дя безразмерные координаты ~ρ =~r/a0, получаем теперь наше искомоебезразмерное уравнение:

[

−ρ42

− 1ρ

]

Ψ = ε Ψ, (1.7)

где ρ4 ≡ 4 — лапласиан в безразмерных координатах. Из уравненияисчезли все «лишние» буквы, и оно приняло максимально простой дляописываемой ситуации вид.

Теперь вспомним, что описываемый нами свободный атом сферическисимметричен и для него сохраняется момент импульса, который мы вявном виде выделим:

[

−4ρ

2+

12

l 2

ρ2

− 1ρ

]

Ψ(ρ,Ω) = ε Ψ(ρ,Ω). (1.8)

В фигурных скобках записан лапласиан, разбитый теперь на две части —радиальную (4ρ) и угловую (−l 2/ρ2), где l 2 — безразмерный операторквадрата момента импульса.

Решение этого уравнения, как известно, ищется в виде произведениярадиальной и угловой частей:

Ψ(ρ,Ω) = R(ρ)Y (Ω), (1.9)

где Y (Ω) — собственные функции операторов l2, lz — так называемые сфе-рические гармоники, носители сферической симметрии свободного атома.Покрываемым ими двум степеням свободы соответствуют два индекса,квантовые числа l,m:

l 2Ylm = l(l +1)Ylm,

lzYlm = mYlm.(1.10)

Page 10: КВАНТОВАЯ МЕХАНИКА МОЛЕКУЛ · PDF file2 ББК В22.31 УДК 544.11+539.183 isbn 978-5-94356-719-3 Стась Д.В., Плюснин В.Ф. Квантовая

10 ГЛАВА 1. ОДНОЭЛЕКТРОННЫЙ АТОМ

После подстановки функции в виде (1.9) в уравнение (1.8) и разделенияпеременных получаем известное уравнение на радиальную часть волно-вой функции R(ρ):

[

4ρ −l(l +1)

ρ2 +2ρ

+2εnl

]

Rnl = 0. (1.11)

У функции Rnl и соответствующего собственного значения εnl появилосьдва индекса. Один из них (l) нумерует эрмитов оператор R(l), которыйпараметрически зависит от l, а второй (n) — собственные функии/числаэтого оператора.

1.2 Анализ радиальных функций

Решения (пары Rnl, εnl) для уравнения (1.11) известны, но мы по-ка не будем выписывать функции Rnl(ρ) в явном виде, а попытаемсякачественно проанализировать их основные свойства, исходя из самоговида уравнения. Воспользуемся только легко запоминающимся фактом,что

εnl = − 12n2 , n = 1,2, ..., (1.12)

а от величины орбитального момента l энергия не зависит — имеет местотак называемое кулоново вырождение, связанное с наличием в задачедополнительного интеграла движения в потенциале вида 1/ρ.

Прежде всего давайте выделим асимптотику решения уравнения (1.11)на бесконечности. В первом приближении можно пренебречь членами 1/ρ2

и 1/ρ по сравнению с константой ε, что дает уравнение:

4ρ R =1n2 R. (1.13)

Теперь нужно вспомнить вид радиальной части оператора Лапласа 4ρв сферической системе координат. Для этого очень удобно держать вголове линейку выражений 4ρ для одномерного, двумерного (полярная)и трехмерного (сферическая системы координат) пространства:

41ρ = ·∂ρ · ∂ρ;

42ρ =

∂ρ ρ ∂ρ ;

43ρ =

1ρ2 ∂ρ ρ2 ∂ρ (≡ 1

ρ∂ 2

ρ ρ ).

(1.14)

Page 11: КВАНТОВАЯ МЕХАНИКА МОЛЕКУЛ · PDF file2 ББК В22.31 УДК 544.11+539.183 isbn 978-5-94356-719-3 Стась Д.В., Плюснин В.Ф. Квантовая

1.2 АНАЛИЗ РАДИАЛЬНЫХ ФУНКЦИЙ 11

Можно заметить, что в одномерном случае идут подряд два операторадифференцирования, в двумерном случае между ними прокладывается ρ,а в трехмерном — ρ2, то есть ρ в степени числа угловых переменных вданной системе координат, с компенсирующим размерность множителем(соответственно 1,1/ρ,1/ρ2) перед оператором.

После подстановки оператора 43 = 1ρ ∂ 2

ρ ρ и введения вспомогательнойфункции χ = ρR уравнение (1.13) принимает вид:

χ ′′ =χn2 , (1.15)

решение которого с физически разумным поведением на бесконечностиимеет вид χ ∼ e−ρ/n, и соответственно

R(ρ) ∼ 1ρ

e−ρ/n, (1.16)

а вероятность найти электрон на расстоянии ρ от ядра на больших рас-стояниях падает экспоненциально:

dW(ρ) ∼ ρ2 R2(ρ)dρ ∼ e−2ρ/n dρ, ρ → ∞. (1.17)

Теперь давайте уточним полученную асимптотику, удержав в оператореопущенные ранее члены. Будем искать решение в виде

χ(ρ) ∼ ρα e−ρ/n. (1.18)

Тогда уравнение (1.11) для функции χ принимает вид

[

∂ 2ρ − l(l +1)

ρ2 +2ρ− 1

n2

]

χ = 0. (1.19)

После подстановки χ в виде (1.18) получаем алгебраическое уравнение

ρα−2 [α(α −1)− l(l +1)]+2ρα−1 [1−α/n] = 0. (1.20)

Ведущая асимптотика при ρ → ∞ здесь ρα−1, откуда мы получаем α = n.Таким образом, уточненная асимптотика радиальной части волновой функ-ции на бесконечности имеет вид:

Rnl(ρ) ∼ ρn−1 e−ρ/n, ρ → ∞. (1.21)

Page 12: КВАНТОВАЯ МЕХАНИКА МОЛЕКУЛ · PDF file2 ББК В22.31 УДК 544.11+539.183 isbn 978-5-94356-719-3 Стась Д.В., Плюснин В.Ф. Квантовая

12 ГЛАВА 1. ОДНОЭЛЕКТРОННЫЙ АТОМ

Теперь давайте проанализируем поведение функции Rnl(ρ) в нуле.Оставив в уравнении (1.11) только самую сильную зависимость от ρ(1/ρ2), получим:

4ρ R− l(l +1)

ρ2 R = 0, или χ ′′ =l(l +1)

ρ2 χ. (1.22)

Решение однородного уравнения (1.22) стандартно ищется в виде χ ∼ ρα ,что после подстановки дает алгебраическое уравнение

α(α −1) = l(l +1) (1.23)

с корнями α = l + 1, α = −l, из которых конечную в нуле функцию Rдает корень α = l +1. Итого получаем, что

Rnl(ρ) ∼ ρ l , ρ → 0. (1.24)

Вывод, который следует из полученного выражения, оказывается верени для произвольного атома: ненулевую величину электронной плотностина ядре ψ2(0) дают только функции с l = 0 (так называемые s-функции).В частности, именно поэтому константы спин-спинового взаимодействияв ЯМР и изотропные константы сверхтонкого взаимодействия в ЭПРотражают только распределение s электронов.

Теперь на основе полученных асимптотик давайте проанализируемобщий вид функций Rnl(ρ), зная, что решения уравнений типа (1.11),так называемых уравнений гипергеометрического типа, всегда имеют видпроизведения функций-асимптот и конечного полинома. В нашем случаеRnl(ρ) ∼ P(ρ)e−ρ/n, где P(ρ) — некоторый полином. Поскольку на беско-нечности Rnl(ρ) ∼ ρn−1 e−ρ/n, полином P(ρ) имеет степень n−1:

Rnl(ρ) ∼ Pn−1(ρ)e−ρ/n. (1.25)

Давайте возьмем максимально возможный при данном n момент l(равный n−1). В этом случае асимптотика решения в нуле имеет вид

Rn,n−1(ρ) ∼ ρn−1, ρ → 0, (1.26)

а это означает, что и весь полином (степени n−1) имеет такой вид иимеет единственный корень кратности n−1 при ρ = 0:

Pn−1l=n−1(ρ) ∼ ρn−1, Rn,n−1(ρ) ∼ ρn−1 e−ρ/n. (1.27)

Page 13: КВАНТОВАЯ МЕХАНИКА МОЛЕКУЛ · PDF file2 ББК В22.31 УДК 544.11+539.183 isbn 978-5-94356-719-3 Стась Д.В., Плюснин В.Ф. Квантовая

1.2 АНАЛИЗ РАДИАЛЬНЫХ ФУНКЦИЙ 13

Возьмем теперь l на единицу меньше максимального (l = n−2):

Pn−1l=n−2(ρ) ∼ ρn−1, ρ → ∞,

Pn−1l=n−2(ρ) ∼ ρn−2, ρ → 0.

(1.28)

Самый общий вид такого полинома

Pn−1l=n−2(ρ) ∼ ρn−2(ρ −C1), (1.29)

где C1 — некоторая константа. Полином вида (1.29) имеет корень крат-ности n−2 при ρ = 0 и один ненулевой корень при ρ = C1. Продолжаяаналогичным образом, получим, что при l = lmax −2 = n−3 у полиномаимеется два ненулевых корня, при l = lmax −3 — три и так далее. Приэтом все ненулевые корни полиномов (1.29) с разными n оказываютсяразличны и положительны (то есть все константы Ci > 0 и различны),обеспечивая ортогональность собственных функций эрмитова операто-ра R(l). Вид нескольких первых функций Rnl приведен на рис. 1.1.

Для простейших представителей радиальных функций с орбиталь-ным моментом lmax = n−1 Rn,n−1 максимум на радиальном распределенииэлектронной плотности ρ2 R2

n,n−1 находится при

ρmaxn,n−1 = n2, (1.30)

причем с увеличением n он не только сдвигается в сторону больших ρ,но и становится все шире за счет все более медленного затухания экс-поненциальной асимптотики e−ρ/n — с ростом n орбитали становятсявсе более и более диффузными, размытыми. Этот же принципиальныйвывод оказывается справедлив и для орбиталей других типов. Среднеерасстояние от электрона до ядра для функции Rnl дается следующимвыражением:

ρnl = n2(

32− l(l +1)

2n2

)

. (1.31)

И наконец давайте обсудим вопрос о возвращении к исходной раз-мерной задаче. Мы знаем, что безразмерные энергии и безразмерныеволновые функции имеют вид:

εnl = − 12n2 ;

Rnl(ρ) ∼ Pn−1l (ρ)e−ρ/n.

(1.32)

Page 14: КВАНТОВАЯ МЕХАНИКА МОЛЕКУЛ · PDF file2 ББК В22.31 УДК 544.11+539.183 isbn 978-5-94356-719-3 Стась Д.В., Плюснин В.Ф. Квантовая

14 ГЛАВА 1. ОДНОЭЛЕКТРОННЫЙ АТОМ

!" "" "#"$"%" &'(

)*+( , - ., .- /, /-,0,,0/,01,02,03.0, 456

789:

; < =; =< >; ><?;@A?;@>;@;;@>;@A;@B;@C=@; DEF

GHIJ K L MK ML NK NLOKPNKPKKPNKPQKPRKPSMPK TUV

WXYZРис. 1.1. Нормированные на единицу радиальные функции Rnl для n = 1− 3.

Пунктирной линией показан уровень нуля.

Восстанавливая размерности, получаем, что искомые размерные энергиии функции для нашей задачи будут иметь вид:

En = − 12n2 E0 = − 1

2n2

me4

h2

Rnl(r) = Anl a−3/20 Pn−1

l

(r

a0

)

e− r

na0 , a0 =h2

me2 ,

(1.33)

где Anl — нормировочная константа (число).Теперь давайте учтем конечность массы ядра. Для этого нужно во

всех формулах заменить массу электрона m на приведенную массу ядраи электрона µ = mM/(m + M). Поскольку протон на три порядка тяже-лее электрона, получаемая при этом поправка имеет порядок 10−3, чтотем не менее вполне детектируемо в спектроскопическом эксперименте.

Page 15: КВАНТОВАЯ МЕХАНИКА МОЛЕКУЛ · PDF file2 ББК В22.31 УДК 544.11+539.183 isbn 978-5-94356-719-3 Стась Д.В., Плюснин В.Ф. Квантовая

1.2 АНАЛИЗ РАДИАЛЬНЫХ ФУНКЦИЙ 15

Заодно мы получили решения и для целого ряда родственных систем:тяжелых изотопов атома водорода (дейтерия и трития) — нужно в µподставить соответственно M = 2 а.е.м. и M = 3 а.е.м., атома позитрония(атома водорода, в котором протон заменен на позитрон e+) — нужно вµ положить M = m, и атома мюония (атом водорода с заменой e− на от-рицательный мюон µ− — «тяжелый электрон», который примерно в 100раз тяжелее электрона) — нужно заменить m на 100m.

Затем давайте введем неединичный заряд ядра Z ≥ 1. Единственноеместо в гамильтониане, которое затронет такая замена, это потенциал

U(r) = −Ze2

r , и искомые решения получаются из уже имеющихся (1.33)заменой e2 → Ze2. Энергия так называемого водородоподобного иона (E0)оказывается квадратичной по Z, а его линейные размеры (a0) обратнопропорциональны заряду ядра, и с ростом последнего электронная обо-лочка атома (пока только из одного электрона) сжимается линейно по Z.Водородоподобные ионы (He+, Li2+ и т.д.) — вполне реальные системы, ив справочнике можно найти экспериментально определенные параметрытаких частиц вплоть до 36Kr35+ — атома криптона, потерявшего 35 изсвоих 36 электронов.

Мы уже сняли ограничение на неподвижность ядра и единичностьего заряда, давайте теперь еще раз вернемся к самому исходному нашемудопущению: взяв уравнение Шредингера, мы автоматически использова-ли нерелятивистское приближение, удержав в уравнении только членынулевого порядка по v/c. Для скорости электрона в основном состоя-нии атома водорода v0/c = α = 1/137 (так называемая постоянная тонкойструктуры), и мы имеем запас по малости в два порядка. Для оценкиудобно взять, что vZ,n ∼

EZ,n/m ∼ (Z/n)v0 — характерные скорости вводородоподобном ионе линейно растут с Z, и для нашего примера скриптоном мы бы получили v/c ∼ 1/4 — разумность нерелятивисткогоописания в такой ситуации весьма спорна.

Подводя итог, можно сказать, что общие выражения для энергий ирадиальных частей волновой функции водородоподобной системы с яд-ром массы M и не слишком большим зарядом ядра Z имеют вид:

En = − 12n2

µZe4

h2 ,

Rnl(r) = Anl

(Za0

)3/2

Pn−1l

(Zra0

)

e− Zr

na0 ,

a0 =h2

µe2 ; µ =mM

m+M.

(1.34)

Page 16: КВАНТОВАЯ МЕХАНИКА МОЛЕКУЛ · PDF file2 ББК В22.31 УДК 544.11+539.183 isbn 978-5-94356-719-3 Стась Д.В., Плюснин В.Ф. Квантовая

16 ГЛАВА 1. ОДНОЭЛЕКТРОННЫЙ АТОМ

Явный вид нескольких младших функций Rnl для водородоподобных ио-нов с единственным электроном дается следующими выражениями:

R10(ρ) = 2(

Za0

)3/2e−ρ ,

R21(ρ) = 12√

6

(Za0

)3/2ρ e−ρ/2,

R20(ρ) = 12√

2

(Za0

)3/2(2−ρ)e−ρ/2,

R32(ρ) = 481√

30

(Za0

)3/2ρ2 e−ρ/3,

R31(ρ) = 481√

6

(Za0

)3/2(6ρ −ρ2)e−ρ/3,

R30(ρ) = 281√

3

(Za0

)3/2(27−18ρ +2ρ2)e−ρ/3.

(1.35)

Здесь ρ = Zr/a0, функции нормированы условием∫ ∞

0 R2nl r2 dr = 1.

1.3 Анализ угловых функций

Давайте теперь посмотрим на угловые части волновых функций атомаводорода — сферические гармоники Ylm(ϑ ,ϕ). Их общий вид следующий:

Ylm(ϑ ,ϕ) ∼ sin|m| ϑ P l−|m|g/u (cosϑ)e imϕ , (1.36)

где P l−|m|g/u (cosϑ) — полином степени l −|m| и определенной четности от-

носительно своего аргумента x = cosϑ : четного (g) или нечетного (u) исодержащего соответственно только четные и только нечетные степени x.Области изменения независимых переменных 0≤ ϕ ≤ 2π, 0≤ ϑ ≤ π.

Функции устроены следующим образом:

• зависимость от ϕ всегда имеет вид e imϕ , Y ∗lm = Yl,−m;

• сумма степеней sinϑ и полинома P l−|m|g/u (cosϑ) всегда равна l;

• при |m| = |m|max = l имеем

Yl, l ∼ sinl ϑ e imϕ , (1.37)

вся зависимость от ϑ заключена в sinl ϑ ;

Page 17: КВАНТОВАЯ МЕХАНИКА МОЛЕКУЛ · PDF file2 ББК В22.31 УДК 544.11+539.183 isbn 978-5-94356-719-3 Стась Д.В., Плюснин В.Ф. Квантовая

1.3 АНАЛИЗ УГЛОВЫХ ФУНКЦИЙ 17

• при уменьшении |m| на единицу

Yl, l−1 ∼ sinl−1ϑ P1(cosϑ)e imϕ ∼ sinl−1ϑ cosϑ e imϕ , (1.38)

поскольку единственно возможный вид нечетного полинома первойстепени от cosϑ — это собственно cosϑ ;

• при уменьшении |m| еще на единицу

Yl, l−2 ∼ sinl−2ϑ P2(cosϑ)e imϕ ∼ sinl−2ϑ (acos2 ϑ −b)e imϕ (1.39)

и так далее.

Поскольку ни sinϑ , ни e imϕ не обращаются в нуль внутри областиопределения гармоник, нули в Ylm(ϑ ,ϕ) дает только часть, зависящаяот cosϑ . Если в радиальных функциях с убыванием «младшего» ин-декса l постепенно снижалась кратность нулевого корня и появлялисьненулевые корни внутри области определения по r — корни стоящегопри экспоненте полинома (1.35), то для угловых функций с убывани-ем (по модулю) «младшего» индекса m постепенно снижается степеньсомножителя sinϑ и появляются ненулевые корни внутри области опре-деления по ϑ . Функции Ylm(ϑ ,ϕ) имеют по l −|m| узловых поверхностейпо ϑ , отвечающих решению уравнения P l−|m|(cosϑ) = 0. Явный вид пер-вых нескольких сферических гармоник с одним из стандартных выборовфаз приведен ниже:

l = 0:

Y00 =√

14π ,

l = 1:

Y1,±1 = ∓√

38π sinϑ e±iϕ ,

Y10 =√

34π cosϑ ,

l = 2:

Y2,±2 =√

1532π sin2 ϑ e±2iϕ ,

Y2,±1 = ∓√

158π sinϑ cosϑ e±iϕ ,

Y20 =√

516π (3cos2 ϑ −1),

(1.40)

Page 18: КВАНТОВАЯ МЕХАНИКА МОЛЕКУЛ · PDF file2 ББК В22.31 УДК 544.11+539.183 isbn 978-5-94356-719-3 Стась Д.В., Плюснин В.Ф. Квантовая

18 ГЛАВА 1. ОДНОЭЛЕКТРОННЫЙ АТОМ

l = 3:

Y3,±3 =√

3564π sin3 ϑ e±3iϕ ,

Y3,±2 =√

10532π sin2 ϑ cosϑ e±2iϕ ,

Y3,±1 = ∓√

2164π sinϑ (5cos2 ϑ −1)e±iϕ ,

Y30 =√

716π (5cos2 ϑ −3) cosϑ ,

l = 4:

Y4,±4 =√

315512π sin4 ϑ e±4iϕ ,

Y4,±3 = ∓√

31564π sin3 ϑ cosϑ e±3iϕ ,

Y4,±2 =√

45128π sin2 ϑ (7cos2 ϑ −1)e±2iϕ ,

Y4,±1 = ∓√

4564π sinϑ (7cos2 ϑ −3) cosϑ e±iϕ ,

Y40 =√

9256π (35cos4 ϑ −30cos2 ϑ +3).

(1.40)

Функции нормированы условием∫ 2π0 dϕ

∫ π0 sinϑ dϑ |Ylm|2 = 1.

Давайте посмотрим, как ведут себя функции Ylm по отношению коперации инверсии относительно центра I. Как показывает рис. 1.2, всферической системе координат операции инверсии соответствует преоб-разование:

r → r, ϑ → π −ϑ , ϕ → π +ϕ. (1.41)

При этом последовательно получаем:

e im(π+ϕ) = (−1)|m| e imϕ ,

sin(π −ϑ) = sinϑ , cos(π −ϑ) = −cosϑ ,

P l−|m|(cos(π −ϑ)) = (−1)l−|m|P l−|m|(cosϑ),

IYlm(ϑ ,ϕ) = (−1)|m| (−1)l−|m|Ylm(ϑ ,ϕ) = (−1)l Ylm(ϑ ,ϕ).

(1.42)

Таким образом, мы показали, что функции Ylm обладают определеннойчетностью PI = (−1) l по отношению к операции I. Такого рода рассужде-ния нам еще не раз встретятся в дальнейшем.

Page 19: КВАНТОВАЯ МЕХАНИКА МОЛЕКУЛ · PDF file2 ББК В22.31 УДК 544.11+539.183 isbn 978-5-94356-719-3 Стась Д.В., Плюснин В.Ф. Квантовая

1.4 БАЗИС ВЕЩЕСТВЕННЫХ УГЛОВЫХ ФУНКЦИЙ Lζ 19

x

y

z

Рис. 1.2. Преобразование инверсии в сферической системе координат.

1.4 Базис вещественных угловых функций lζ

Сферические гармоники Ylm часто очень удобны для расчетов, но всилу своей комплексности ненаглядны. Для облегчения визуализациибазиса угловых функций вводится стандартный базис вещественных ли-нейных комбинаций сферических гармоник — так называемый s, p,d, ...базис. Буквы s, p, d, f и далее по латинскому алфавиту соответствуютсостояниям с определенной величиной орбитального момента: s ↔ l = 0(одно состояние), p ↔ l = 1 (три состояния), d ↔ l = 2 (пять состояний),затем f ↔ l = 3 (семь состояний) и так далее. Используемые буквы взятыиз атомной спектроскопии и соответствуют исходным названиям серийлиний в спектрах испускания паров щелочных металлов, которые, какпотом выяснилось, порождались переходами с участием этих состояний:

• s состояния ↔ «sharp» ↔ переходы (n + 2)s → (n + 1)p, линии нерасщепляются при наложении внешнего поля и всегда остаются«четкими»;

Page 20: КВАНТОВАЯ МЕХАНИКА МОЛЕКУЛ · PDF file2 ББК В22.31 УДК 544.11+539.183 isbn 978-5-94356-719-3 Стась Д.В., Плюснин В.Ф. Квантовая

20 ГЛАВА 1. ОДНОЭЛЕКТРОННЫЙ АТОМ

Рис. 1.3. Слева направо: контурные диаграммы для трех вещественных угловыхфункций 2px, 2py, 2pz момента l = 1.

• p состояния ↔ «principal» ↔ переходы (n +1)p → ns, дающие дляпаров щелочных металлов «основную», самую интенсивную спек-тральную серию;

• d состояния ↔ «diffuse» ↔ переходы (n + 2)d → (n + 1)p, дающие«размытые» линии.

Далее пропускается и так перегруженный символ в квантовой механикесимвол «e», и буквы следуют по алфавиту ( f , g и т.д.). «n» в описаниипереходов обозначает квантовое число основного состояния единствен-ного внешнего электрона в атоме щелочного металла, n = 2 для атомалития, 3 — для натрия, 4 — калия и т.д. Внутри соответствующего каж-дому l набора имеется по 2l +1 линейно независимых состояний, которыеразличаются между собой индексами.

Давайте построим несколько младших функций s, p,d, ... базиса ипосмотрим, как они выглядят:

• s состояния: l = 0, m = 0,

Y00(ϑ ,ϕ) = const, (1.43)

s состояние сферически симметрично и выглядит как шарик;

• p состояния: l = 1, m = 0,±1,

Y11(ϑ ,ϕ) = A sinϑ e iϕ ,

Y10(ϑ ,ϕ) = B cosϑ ,

Y1−1(ϑ ,ϕ) = A sinϑ e−iϕ .

(1.44)

Page 21: КВАНТОВАЯ МЕХАНИКА МОЛЕКУЛ · PDF file2 ББК В22.31 УДК 544.11+539.183 isbn 978-5-94356-719-3 Стась Д.В., Плюснин В.Ф. Квантовая

1.4 БАЗИС ВЕЩЕСТВЕННЫХ УГЛОВЫХ ФУНКЦИЙ Lζ 21

Собирая комплексно-сопряженные функции Y11 и Y1−1 в веществен-ные комбинации, получим:

p1(ϑ ,ϕ) ∼ Y11+Y1−1 ∼ sinϑ cosϕ ∼ x (px),

p2(ϑ ,ϕ) ∼ Y11−Y1−1 ∼ sinϑ sinϕ ∼ y (py),

p3(ϑ ,ϕ) ∼ Y10 ∼ cosϑ ∼ z (pz).

(1.45)

Как видно из рис. 1.3, функции имеют вид «гантелей», уложен-ных соответственно вдоль осей x (p1), y (p2), z (p3), с одной узло-вой плоскостью, перпендикулярной оси гантели, при пересечениикоторой функции меняют знак. 1 Вид самих функций совпадает свыражениями для декартовых координат x, y, z в сферической сис-теме координат, и стандартные «говорящие» обозначения для такихфункций — соответственно px, py, pz.

• d состояния: l = 2, m = 0,±1,±2,

Y22(ϑ ,ϕ) = A sin2 ϑ e2iϕ ,

Y21(ϑ ,ϕ) = B sinϑ cosϑ e iϕ ,

Y20(ϑ ,ϕ) = C (3cos2 ϑ −1),

Y2−1(ϑ ,ϕ) = B sinϑ cosϑ e−iϕ ,

Y2−2(ϑ ,ϕ) = A sin2 ϑ e−2iϕ .

(1.46)

Снова собирая суммы и разности пар комплексно-сопряженных функ-ций Y2m и Y2−m в вещественные комбинации, получим:

Y22+Y2−2 ∼ sin2 ϑ cos2ϕ ∼ x2− y2 (dx2−y2),

Y22−Y2−2 ∼ sin2 ϑ sin2ϕ ∼ xy (dxy),

Y21+Y2−1 ∼ sinϑ cosϑ cosϕ ∼ xz (dxz),

Y21−Y2−1 ∼ sinϑ cosϑ sinϕ ∼ yz (dyz),

Y20 ∼ 3cos2 ϑ −1 ∼ 3z2− r2 (dz2).

(1.47)

1Приведенные контурные диаграммы — поверхности постоянного значения — здесь идалее построены в программе «Orbital Viewer», автор David Manthey, которую можно бес-платно получить на сайте www.orbitals.com.

Page 22: КВАНТОВАЯ МЕХАНИКА МОЛЕКУЛ · PDF file2 ББК В22.31 УДК 544.11+539.183 isbn 978-5-94356-719-3 Стась Д.В., Плюснин В.Ф. Квантовая

22 ГЛАВА 1. ОДНОЭЛЕКТРОННЫЙ АТОМ

Рис. 1.4. Пять 3d орбиталей (момент l = 2).

Функции по виду напоминают квадратичные выражения для про-изведений декартовых координат, которые снова используются вкачестве «говорящих» индексов — для пяти функций стандартноиспользуются мнемонические обозначения dx2−y2, dxy, dxz, dyz и dz2,соответственно. Как показывает рис. 1.4, функции имеют харак-терный вид «бабочек» с двумя узловыми поверхностями: либо обепо ϑ (dz2), либо одна по ϑ , одна по ϕ (dxz, dyz), либо обе по ϕ (dxy,dx2−y2). Для функции d2

z r2 = x2 + y2 + z2.

• f состояния: l = 3, m = 0,±1,±2,±3,

Для f электронов аналогичным образом из семи комплексных сфе-рических гармоник Y3m можно собрать семь линейно-независимыхвещественных функций fζ . Однако явный вид функций fζ (ϑ ,ϕ)здесь получается довольно громоздким, и, кроме того, в отличие отs, p и d функций, существует несколько общепринятых стандарт-ных базисов f функций, оптимизированных для расчетов в поляхразной симметрии. Вид функций двух базисов — кубический набор(для высокосимметричных систем, левая колонка) и низкосиммет-ричный набор (правая колонка) — через кубичные выражения от

Page 23: КВАНТОВАЯ МЕХАНИКА МОЛЕКУЛ · PDF file2 ББК В22.31 УДК 544.11+539.183 isbn 978-5-94356-719-3 Стась Д.В., Плюснин В.Ф. Квантовая

1.4 БАЗИС ВЕЩЕСТВЕННЫХ УГЛОВЫХ ФУНКЦИЙ Lζ 23

Рис. 1.5. Семь 4 f орбиталей (момент l = 3).

декартовых координат приведен ниже:

fx3 ∼ x(5x2−3r2); fz3 ∼ z(5z2−3r2),

fy3 ∼ y(5y2−3r2); fxz2 ∼ x(5z2−3r2),

fz3 ∼ z(5z2−3r2); fyz2 ∼ y(5z2−3r2),

fxyz ∼ xyz; fxyz ∼ xyz,

fx(z2−y2) ∼ x(z2− y2); fz(x2−y2) ∼ z(x2− y2),

fy(z2−x2) ∼ z(z2− x2); fx(x2−3y2) ∼ x(x2−3y2),

fz(x2−y2) ∼ z(x2− y2); fy(3x2−y2) ∼ y(3x2− y2).

(1.48)

Все функции имеют по три узловые поверхности и имеют харак-терный вид «ежиков» (рис. 1.5).

Функции вещественного s, p,d, ... базиса имеют всегда ровно по lузловых поверхностей. Система их мнемонических обозначений оченьудобна и позволяет при необходимости быстро восстановить явный видфункции. Обозначения также в явном виде показывают эквивалентностьфункций: так, например, становится понятной эквивалентность внутритроек функций px, py, pz и dxy, dxz, dyz, которые отличаются между собойтолько выбором направления (соответственно плоскостей), на которые

Page 24: КВАНТОВАЯ МЕХАНИКА МОЛЕКУЛ · PDF file2 ББК В22.31 УДК 544.11+539.183 isbn 978-5-94356-719-3 Стась Д.В., Плюснин В.Ф. Квантовая

24 ГЛАВА 1. ОДНОЭЛЕКТРОННЫЙ АТОМ

они уложены. Явные выражения для функций s, p,d, ... базиса черезлинейные комбинации сферических гармоник привязаны к выбору фазфункций Ylm, и для гармоник из (1.40) нормированные p и d функциивыражаются через функции Y1m, Y2m следующим образом:

px = −(Y11−Y1−1)/√

2 =√

316π sinϑ cosϕ,

py = −(Y11+Y1−1)/i√

2 =√

316π sinϑ sinϕ,

pz = Y10 =√

34π cosϑ ,

dx2−y2 = (Y22+Y2−2)/√

2 =√

1564π sin2 ϑ cos2ϕ,

dxy = (Y22−Y2−2)/i√

2 =√

1564π sin2 ϑ sin2ϕ,

dxz = −(Y21−Y2−1)/√

2 =√

1516π sinϑ cosϑ cosϕ,

dyz = −(Y21+Y2−1)/i√

2 =√

1516π sinϑ cosϑ sinϕ,

dz2 = Y20 =√

516π (3cos2 ϑ −1).

(1.49)

1.5 Полные функции Ψnlm (r,ϑ ,ϕ), nlζ (r,ϑ ,ϕ)

Полные волновые функции для водородоподобных систем собираютсякак произведения соответствующих радиальных и угловых частей со-гласно нашему исходному разбиению (1.9), при этом в качестве угловойчасти берутся либо сферические гармоники Ylm, либо функции s, p,d, ...базиса lζ . Функции параметризуются тремя индексами, соответственнотройками n, l,m или n, l,ζ:

|n, l,m >= Rnl (r)Ylm (ϑ ,ϕ); nlζ = Rnl (r) lζ (ϑ ,ϕ). (1.50)

Любопытно отметить, что присутствие в полной функции радиальнойчасти может существенно изменить распределение электронной плотнос-ти для казалось бы похожих функций, например, pz орбиталей. Давайтесравним функции 2pz, 3pz и 4pz:

2pz ≡ R21(r)Y10(ϑ ,ϕ) ≡ |2,1,0>,

3pz ≡ R31(r)Y10(ϑ ,ϕ) ≡ |3,1,0>,

4pz ≡ R41(r)Y10(ϑ ,ϕ) ≡ |4,1,0> .

(1.51)

Page 25: КВАНТОВАЯ МЕХАНИКА МОЛЕКУЛ · PDF file2 ББК В22.31 УДК 544.11+539.183 isbn 978-5-94356-719-3 Стась Д.В., Плюснин В.Ф. Квантовая

1.5 ПОЛНЫЕ ФУНКЦИИ ΨNLM (R,ϑ ,ϕ), NLζ (R,ϑ ,ϕ) 25

Рис. 1.6. Три «одинаковые» pz орбитали атома водорода: 2pz, 3pz, 4pz.

На рис. 1.6 эти три орбитали показаны в одном масштабе. Как видноиз рисунка, помимо увеличения размеров орбитали с ростом n, замена ра-диальных частей R21 → R31 и R21 → R41 привела к появлению у функциидополнительно одной и двух узловых поверхностей — радиальных пере-тяжек. В этом заключается одна из причин довольно большого различияв химических свойствах элементов второго периода и их гомологов изтретьего и более далеких периодов, например, кислорода и серы или азо-та и фосфора. В общем случае функция nlζ имеет l узловых поверхностейот угловой части (|m| по ϕ и l−|m| по ϑ ) и n− l−1 перетяжек от радиаль-ной части. Величина nr = n− l−1, так называемое радиальное квантовоечисло, нумерует собственные значения эрмитова оператора R(l) (1.11),и в соответствии с известной теоремой о нулях функций дает и чис-ло нулей отвечающих ему собственных функций. Итого получаем n−1узловых поверхностей. Таким образом, чем выше n, тем более изрезан-ным будет распределение электронной плотности для соответствующегосостояния. При этом нужно конечно оговориться, что свободный атомвсегда остается сферически симметричным, «круглым» — из него вовсене торчат гантели и бабочки, это просто один из возможных базисов,удобный для визуализации.

Любопытный вопрос — полон ли набор функций |n, l,m > или nlζ ?Оказывается нет, поскольку он ограничен лишь функциями дискретногоспектра, в то время как полный спектр гамильтониана для данной зада-чи имеет и непрерывную компоненту. Простейший возможный расчет —разложение (известного аналитически) основного состояния иона He+

по функциям атома водорода — без учета состояний непрерывного спек-

Page 26: КВАНТОВАЯ МЕХАНИКА МОЛЕКУЛ · PDF file2 ББК В22.31 УДК 544.11+539.183 isbn 978-5-94356-719-3 Стась Д.В., Плюснин В.Ф. Квантовая

26 ГЛАВА 1. ОДНОЭЛЕКТРОННЫЙ АТОМ

тра дает ошибку в энергии основного состояния около 10%. При этомможно сразу сказать, что такого рода ошибка всегда будет в сторонуболее высоких энергий, поскольку ограничение базиса, сокращая набордоступных для варьирования функций, может только завышать энергиюосновного состояния.

Наконец давайте обсудим вопрос об ортогональности функций |n, l,m >.Известно, что

< n, l,m |n′, l′,m′ >= δnn′ δll ′ δmm′ , (1.52)

или∫

Ψ∗nlm (r,ϑ ,ϕ)Ψn′l ′m′ (r,ϑ ,ϕ) r2sinϑ dr dϑ dϕ = δnn′ δll ′ δmm′ . (1.53)

Интеграл в (1.53) разбивается в произведение независимых интеграловпо трем переменным:

∫ ∞

0Rnl (r)Rn′ l (r) r2 dr ·

∫ π

0Θlm (cosϑ)Θl ′m′ (cosϑ)d cosϑ ×

×∫ 2π

0e−imϕ e im′ϕ dϕ = Ir · Iϑ · Iϕ .

(1.54)

Давайте разберемся, за счет чего выполняется условие ортогональностипо каждому из индексов.

Если m 6= m′, то Iϕ = 0 — ортогональность по младшему индексу mопределяется переменной ϕ. Интеграл Iϑ при разных m не обязан бытьравен нулю, поскольку входящие в него функции в этом случае будут ре-шениями разных уравнений, параметрически зависящих от m, собствен-ными функциями разных операторов, и никаких условий на их автома-тическую ортогональность не накладывается. Так, несложно проверить,например, что

∫ π

0(3cos2 ϑ −1) sin2 ϑ d cosϑ 6= 0, (1.55)

где взяты Θ-компоненты функций Y20 и Y22. Если же имеем m = m′, товходящие в Iϑ функции будут собственными функциями одного и тогоже оператора, нумеруемые индексом l, и поэтому обязательно должныбыть ортогональны при l 6= l′.

Та же ситуация имеет место и для радиальных функций: при l 6= l′

интеграл Ir вовсе не обязан быть равен нулю. Например, мы знаем, чтовсе функции Rn,n−1 с максимально возможными при данном n значении lимеют вид ρ n−1 e−ρ/n и везде неотрицательны, и потому они просто не

Page 27: КВАНТОВАЯ МЕХАНИКА МОЛЕКУЛ · PDF file2 ББК В22.31 УДК 544.11+539.183 isbn 978-5-94356-719-3 Стась Д.В., Плюснин В.Ф. Квантовая

1.6 БАЗИС ГИБРИДНЫХ ОРБИТАЛЕЙ SPN 27

могут быть ортогональны. Функции Rnl являются решениями уравне-ния R(l)Rnl = εnlRnl с оператором R(l), параметрически зависящим от l.При l 6= l′ стоящие в Ir функции будут собственными функциями разныхоператоров, и на их ортогональность не накладывается никаких условий.Если же l = l′, то Ir = δnn′ .

Таким образом, условие ортогональности (1.54) разворачивается в по-следовательности ϕ → ϑ → r (или m → l → n):

Iϕ = δmm′ ,если m = m′, то Iϑ = δll ′

если l = l′, то Ir = δnn′ .(1.56)

Условие ортогональности для угловой части (Iϑ Iϕ) обеспечивает дляатомов большинство правил отбора. Связано это с тем, что основныевызывающие переходы операторы либо вовсе не зависят от углов (как,например, распад n → p+ +e−), либо зависят от них достаточно простымобразом — как компоненты вектора (оператор дипольного момента ~d),тензора второго (поляризуемость) или более высокого (кристаллическоеполе) ранга. Возникающие при этом интегралы сводятся к линейнымкомбинациям интегралов типа Iϑ Iϕ . Условие ортогональности по r в этомсмысле почти бесполезно. Поэтому расчет обычно ведется в два этапа:сначала отыскиваются заведомо нулевые матричные элементы по ϑ , ϕ(или по l, m), и только затем рассчитываются действительно требуемыезадачей интегралы с радиальными функциями.

1.6 Базис гибридных орбиталей spn

До сих пор речь шла о базисах, удобных для описания изолиро-ванного, сферически симметричного атома. Однако при собирании изатомов молекул окружение атома перестает быть сферически симмет-ричным, и классификация атомных состояний по полному моменту —деление на s, p,d, ... состояния — для рассматриваемой задачи пере-стает быть естественной, эти состояния перемешиваются. Для описанияатома, участвующего в образовании химической связи в молекуле, бо-лее естественным часто оказывается базис так называемых гибридных,смешанных орбиталей пониженной симметрии, позволяющих удобным инаглядным образом описать направленность химической связи. Типич-ным и наиболее часто используемым гибридным базисом является такназываемый spn базис, функции которого выражаются через линейные

Page 28: КВАНТОВАЯ МЕХАНИКА МОЛЕКУЛ · PDF file2 ББК В22.31 УДК 544.11+539.183 isbn 978-5-94356-719-3 Стась Д.В., Плюснин В.Ф. Квантовая

28 ГЛАВА 1. ОДНОЭЛЕКТРОННЫЙ АТОМ

комбинации одной 2s и n = 1, 2 или 3 2p орбиталей. Остальные n−3из 2p орбиталей в образовании гибридных орбиталей не участвуют иостаются «чистыми».

Смешивание с равными весами 2s и одной из 2p орбиталей даетдве гибридные sp орбитали, направленные под углом 180 друг к другу,оставляя две перпендикулярные 2p орбитали чистыми: sp гибридизацияприводит к линейной геометрии молекулы, в качестве типичного примераздесь можно привести молекулу ацетилена HC ≡CH. Можно рассуждатьи обратным образом, сказав, что для описания строения линейных мо-лекул типа молекулы ацетилена естественным будет базис sp гибридныхорбиталей. Смешивание 2s и двух 2p орбиталей даст три гибридные sp2

орбитали, лежащие в одной плоскости и направленные под углом 120

друг к другу, приводя к геометрии правильного треугольника, харак-терной, например, для CH2 групп в молекулах типа молекулы этиленаH2C = CH2. Наконец, при задействовании в гибридизации всех трех 2pорбиталей получается четыре эквивалентные sp3 гибридные орбитали стетраэдрической геометрией (угол 109,5), примерно реализуемой в CH3

группах молекулы этана H3C −CH3. На рис. 1.7 показано стандартноеизображение трех типов гибридных орбиталей spn, хотя реально узелв распределении электронной плоскости оказывается несколько смещен-ным из нуля. На практике степень гибридизации атома можно извлечь изизмеренных углов между образуемыми им связями — углы в 180, 120 и109,5 говорят соответственно об sp, sp2 и sp3 гибридизации. Посколькуреальные углы редко соответствуют приведенным теоретическим значе-ниям, орбитали реального атома обычно оказываются не идеализованны-ми гибридами, а некоторыми обобщенными орбиталями вида hi = s+λi p,эквивалентными spλ 2

i гибридным орбиталям. Вес λ 2i может формально

меняться от 0 до бесконечности, соответственно покрывая весь диапазонот чистой s до чистой p орбитали. Для любой пары гибридных орбита-лей hi, h j из одного набора выполняется соотношение ортогональности

1+λiλ j cosϑi j = 0, (1.57)

где ϑi j — угол между орбиталями. Соотношения нормировки для такихобобщенных гибридных орбиталей выглядят следующим образом:

∑ 1

1+λ 2i

= 1;1

1+λ 2i

= «s характер» орбитали hi,

∑ λ 2i

1+λ 2i

= 1,2,3;λ 2

i

1+λ 2i

= «p характер» орбитали hi.

(1.58)

Page 29: КВАНТОВАЯ МЕХАНИКА МОЛЕКУЛ · PDF file2 ББК В22.31 УДК 544.11+539.183 isbn 978-5-94356-719-3 Стась Д.В., Плюснин В.Ф. Квантовая

1.6 БАЗИС ГИБРИДНЫХ ОРБИТАЛЕЙ SPN 29

[\ [\

]^_`

spa

[\b

[\b

cde f

[\b

[\g

[\g[\g

[\g

hijk l m

Рис. 1.7. Гибридные sp, sp2 и sp3 орбитали.

Чтобы построить правильно ориентированную гибридную орбиталь,нам нужно выполнить две операции: построить соответствующим обра-зом направленную p орбиталь и выбрать коэффициент смешивания λi.Для построения spn гибридных орбиталей существует очень нагляднаяи удобная процедура, основанная на аналогии между функциями pζ икомпонентами вектора.

Возьмем нормированную линейную комбинацию орбиталей px и py:

Ψ = apx +bpy, a2 +b2 = 1. (1.59)

Для простоты коэффициенты a и b взяты вещественными. Естественныйвыбор для них — a = cosα, b = sinα, и, вспоминая вид px и py, получим:

Ψ ∼ cosα · sinϑ cosϕ +sinα · sinϑ sinϕ = sinϑ cos(α −ϕ). (1.60)

Мы снова получили p орбиталь, лежащую в плоскости xy и направлен-ную под углом α к оси x. Давайте возьмем нормированную линейную

Page 30: КВАНТОВАЯ МЕХАНИКА МОЛЕКУЛ · PDF file2 ББК В22.31 УДК 544.11+539.183 isbn 978-5-94356-719-3 Стась Д.В., Плюснин В.Ф. Квантовая

30 ГЛАВА 1. ОДНОЭЛЕКТРОННЫЙ АТОМ

комбинацию всех трех p функций:

Ψ = Apx +Bpy +Cpz, A2 +B2 +C2 = 1. (1.61)

Поскольку из трех коэффициентов A, B, C только два независимы, ихудобно параметризовать следующим образом: A = sinγ cosα, B = sinγ sinα,C = cosγ. Прямая подстановка дает

Ψ ∼ sinγ sinϑ cos(α −ϕ)+cosγ cosϑ . (1.62)

Это снова p орбиталь, и прямое дифференцирование дает направления, вкоторых она имеет экстремумы: ϑ = γ, ϕ = α и ϑ = π −γ, ϕ = π +α, т. е.вдоль линии, положение которой в пространстве задается сферическимиуглами γ,α.

Можно заметить, что наша конструкция для функции Ψ выглядиткак скалярное произведение символического вектора ~P = (px, py, pz) наединичный вектор ~eγα = (sinγ cosα,sinγ sinα,cosγ) в направлении (γ,α).Давайте теперь обратим наши рассуждения: чтобы построить p орби-таль с положительным плечом в направлении (γ,α), нужно скалярноумножить символический вектор ~P на орт ~eγα . Чтобы построить гибрид-ные spn орбитали, нужно построить p орбитали в нужном направлениии примешать к ним s орбиталь с соответствующим весом. Давайте мыэто сейчас и проделаем для орбиталей с n = 2, получив показанные нарис. 1.8 функции.

• sp орбитали (вдоль оси x)

соответствующие орты, направленные вдоль них p орбитали и по-лучаемые гибридные орбитали имеют вид:

~e1 = (1,0,0), p1 = (~P,~e1) = px, ψ1 = 1√2(s+ px),

~e2 = (−1,0,0), p2 = (~P,~e2) = −px, ψ2 = 1√2(s− px).

(1.63)

Две гибридные sp орбитали имеют вид асимметричных гантелей,орбитали py и pz остались чистыми p орбиталями. Построенныеорбитали удобны для описания локализованной связи в направле-нии положительной и отрицательной полуосей x. Другими словами,базис ψ1,ψ2, py, pz будет удобен для описания связи в линейноймолекуле с «тройной» связью типа молекулы ацетилена или моле-кулярном фрагменте аналогичного строения.

Page 31: КВАНТОВАЯ МЕХАНИКА МОЛЕКУЛ · PDF file2 ББК В22.31 УДК 544.11+539.183 isbn 978-5-94356-719-3 Стась Д.В., Плюснин В.Ф. Квантовая

1.6 БАЗИС ГИБРИДНЫХ ОРБИТАЛЕЙ SPN 31

nopqr sqtuqvwxy z| utq~o

nopqr sqtuqvwxy z utq~o

nopqr qr~xy z q utq~o

nopqr sqtuqvwxy z u tq~o

Рис. 1.8. Четыре стандартных базиса для 2s2p орбиталей: чистые s и p функциии три типа гибридных базисов.

Page 32: КВАНТОВАЯ МЕХАНИКА МОЛЕКУЛ · PDF file2 ББК В22.31 УДК 544.11+539.183 isbn 978-5-94356-719-3 Стась Д.В., Плюснин В.Ф. Квантовая

32 ГЛАВА 1. ОДНОЭЛЕКТРОННЫЙ АТОМ

Рис. 1.9. Удобный способ задать положение вершин тетраэдра в пространстве —вписать его в куб.

• sp2 орбитали (в плоскости xy)

соответствующие орты и гибридные орбитали:

~e1 = (1,0,0), ψ1 =√

13 s+

√23 px,

~e2 = (−12 ,

√3

2 ,0), ψ2 =√

13 s+

√23

(

− px2 +

√3py2

)

,

~e3 = (−12 ,−

√3

2 ,0), ψ3 =√

13s+

√23

(

− px2 −

√3py2

)

.

(1.64)

Орбиталь pz осталась чистой p орбиталью. Базис ψ1,ψ2,ψ3, pzбудет удобен для описания плоской молекулы с «двойной» связью,например, этилена, или аналогичного молекулярного фрагмента.

Page 33: КВАНТОВАЯ МЕХАНИКА МОЛЕКУЛ · PDF file2 ББК В22.31 УДК 544.11+539.183 isbn 978-5-94356-719-3 Стась Д.В., Плюснин В.Ф. Квантовая

1.6 БАЗИС ГИБРИДНЫХ ОРБИТАЛЕЙ SPN 33

• sp3 орбитали

Для наглядного построения sp3 орбиталей впишем тетраэдр в куби пустим декартовы оси через середины граней куба (рис. 1.9).Соответствующие орты и гибридные орбитали имеют вид:

~e1 = 1√3(1,1,1), ψ1 = 1

2s+ 12(px + py + pz),

~e2 = 1√3(−1,−1,1), ψ2 = 1

2s+ 12(−px − py + pz),

~e3 = 1√3(1,−1,−1), ψ3 = 1

2s+ 12(px − py − pz),

~e4 = 1√3(−1,1,−1), ψ4 = 1

2s+ 12(−px + py − pz).

(1.65)

В sp3 базисе вовсе не осталось чистых p орбиталей. Базис ψ1,ψ2,ψ3,ψ4будет полезен для описания органических молекул с «насыщенными»связями, такими как алканы CnH2n+2, и других систем с «тетраэдричес-кими» атомами, например, аммиака NH3.

Page 34: КВАНТОВАЯ МЕХАНИКА МОЛЕКУЛ · PDF file2 ББК В22.31 УДК 544.11+539.183 isbn 978-5-94356-719-3 Стась Д.В., Плюснин В.Ф. Квантовая
Page 35: КВАНТОВАЯ МЕХАНИКА МОЛЕКУЛ · PDF file2 ББК В22.31 УДК 544.11+539.183 isbn 978-5-94356-719-3 Стась Д.В., Плюснин В.Ф. Квантовая

Глава 2

Электронное строение

многоэлектронного атома

В первой главе мы с вами разобрали свойства и подходы к описаниюпростейших одноэлектронных систем — атома водорода и водородоподоб-ных ионов. Теперь давайте сделаем следующий шаг и посмотрим, к чемуприводит включение в рассмотрение более одного электрона (пока дляизолированного атома), что совершенно необходимо для описания моле-кул. Начнем мы с простейшего примера квантовохимического расчета —вычисления сродства к электрону атома водорода.

2.1 Сродство к электрону атома водорода

Сродство атома к электрону EEA определяется как энергия, котораявыделяется при добавлении свободного электрона в атом, в нашем случаек атому водорода в основном состоянии. Другими словами, сродство кэлектрону равно потенциалу ионизации отрицательного иона — энергия,которую нужно затратить, чтобы высвободить из иона лишний электрон.Сродство к электрону считается положительным, если при присоедине-нии электрона к атому энергия выделяется. Таким образом, нам нужнорассчитать разность энергии основного состояния атома водорода E0

1s иэнергии основного состояния иона H−, которую мы обозначим E−. Энер-

Page 36: КВАНТОВАЯ МЕХАНИКА МОЛЕКУЛ · PDF file2 ББК В22.31 УДК 544.11+539.183 isbn 978-5-94356-719-3 Стась Д.В., Плюснин В.Ф. Квантовая

36 ГЛАВА 2. МНОГОЭЛЕКТРОННЫЙ АТОМ

гию E01s можно взять из (1.33):

E01s = −1

2me4

h2 = −E0

2. (2.1)

Чтобы найти E−, нужно выписать оператор энергии H для иона H−,составить волновую функцию Ψ и рассчитать среднее 〈Ψ|H|Ψ〉.

Гамильтониан системы запишем в виде

H = H1 + H2 +e2

|~r1−~r2|, (2.2)

где H1,2 — операторы энергии (кинетическая энергия плюс кулоновскийпотенциал притяжения к ядру) для каждого из электронов (1.3), а третьеслагаемое отвечает кулоновскому отталкиванию электронов в ионе H−.Волновую функция возьмем в виде

Ψ = φ0(~r1)φ0(~r2), (2.3)

где φ0(~r1,2) — орбитальные волновые функции основного состояния дляатома водорода, в котором окажутся оба электрона. Корректный расчет сдвухэлектронной функцией конечно требует включения спиновой части иантисимметризации полной функции, однако в данном случае последняяраспадется на произведение орбитальной функции (2.3) и спиновой (син-глетной) функции, которая прозрачно пронесется через все вычисления,поскольку операторы энергии не содержат спин.

Итак,

E− =

φ0(~r1)φ0(~r2)

∣∣∣∣H1 + H2 +

e2

|~r1−~r2|

∣∣∣∣φ0(~r1)φ0(~r2)

. (2.4)

Две трети расчета можно сделать сразу, воспользовавшись удачной груп-пировкой слагаемых в гамильтониане (2.2) — операторы H1,2 дадут E1s:

⟨φ0(~r1)φ0(~r2)

∣∣H1∣∣φ0(~r1)φ0(~r2)

⟩=⟨φ0(~r1)

∣∣H1∣∣φ0(~r1)

⟩= E1s. (2.5)

Отсюда получаем, что

E− = 2E1s +

φ0(~r1)φ0(~r2)

∣∣∣∣

e2

|~r1−~r2|

∣∣∣∣φ0(~r1)φ0(~r2)

= 2E1s + I, (2.6)

Page 37: КВАНТОВАЯ МЕХАНИКА МОЛЕКУЛ · PDF file2 ББК В22.31 УДК 544.11+539.183 isbn 978-5-94356-719-3 Стась Д.В., Плюснин В.Ф. Квантовая

2.1 СРОДСТВО К ЭЛЕКТРОНУ АТОМА ВОДОРОДА 37

где интеграл I придется считать. Вспоминая явный вид функции φ0(~r) инормируя ее:

φ0(~r) = R10(r)Y00(ϑ ,ϕ) =1

πa30

e−r/a0 , (2.7)

получим:

I =

(1

πa30

)2∫

e− 2(r1+r2)

a0e2

|~r1−~r2|d~r1 d~r2. (2.8)

Переходя к сферическим координатам и интегрируя по трем из четырехуглов, получаем:

I = e2(

1

πa30

)2

8π2∫

e− 2(r1+r2)

a0 r21r2

2 dr1 dr2

∫ 0

π

d(cosϑ)√

r21 + r2

2−2r1r2cosϑ. (2.9)

Интегрируя по оставшемуся углу и переходя в оставшемся интеграле кк безразмерным координатам, получим:

I = 8E0

∫ ∞

0

∫ ∞

0e−2(ρ1+ρ2)ρ1ρ2(ρ1 +ρ2−|ρ1−ρ2|)dρ1dρ2, (2.10)

где e2/a0 = E0, ρ1,2 = r1,2/a0. Раскрывая модуль и выполняя несложное,но довольно кропотливое интегрирование, в итоге получим

I =58

E0. (2.11)

Таким образом, при помещении второго электрона в атом водорода мыдолжны выиграть энергию −E0/2 за счет притяжения второго электро-на к ядру и проиграть 5/8E0 за счет кулоновского отталкивания двухэлектронов. Итого мы получаем чистый проигрыш 1/8E0 ∼ 3 эВ, то естьу атома водорода, по нашим расчетам, должно быть отрицательное иочень большое по абсолютной величине сродство к электрону. Однакоэксперимент говорит, что это не так: реальное сродство к электрону ато-ма водорода положительно и составляет примерно 0,7 эВ. Мы не тольконе смогли получить разумное число, но даже не смогли качественнопредсказать знак эффекта. Где же ошибка?

Page 38: КВАНТОВАЯ МЕХАНИКА МОЛЕКУЛ · PDF file2 ББК В22.31 УДК 544.11+539.183 isbn 978-5-94356-719-3 Стась Д.В., Плюснин В.Ф. Квантовая

38 ГЛАВА 2. МНОГОЭЛЕКТРОННЫЙ АТОМ

2.2 Пределы применимости водородоподобных

функций

На самом деле ошибки как таковой в проведенном расчете нет, ис точки зрения математики все сделано правильно. Проблема кроетсяв радиальной функции, которую мы использовали при расчете сродст-ва к электрону. Гидрид-ион H− столь же кругл, как и атом водорода,и отражающая это угловая часть волновой функции остается неизмен-ной. Однако теперь уже вокруг ядра имеется не один электрон, так чтоэлектростатическое взаимодействие для электрона уже не сводится кчистому кулоновскому притяжению к ядру, которое закладывалось в ис-ходное уравнение на R(r) (1.11). Другими словами, при внесении в атомводорода второго электрона радиальное распределение для обоих элек-тронов перестает описываться функцией R10(r) для атома водорода, хотяоба состояния остаются сферически симметричными состояниями s типа.Это обстоятельство делает принципиально невозможным точно рассчи-тывать квантовохимические величины, используя аналитически извест-ные функции для одноэлектронных водородоподобных систем.

В квантовой механике существует очень похожая на нашу задача обатоме гелия — те же два электрона, только заряд ядра равен +2, а не +1.Потенциал ионизации атома гелия известен (примерно 24 эВ) — можноли из него получить потенциал ионизации гидрид-иона H− так же, какмы получали энергию иона гелия He+ из энергии атома водорода, путемсведения задачи к безразмерной? Нет, обезразмеривание с исключениемзаряда ядра здесь тоже произвести не получается, поскольку теперь взадаче присутствует два кулоновских взаимодействия, электрона с ядроми электрона с другим электроном, и только одно из них зависит от зарядаядра. И с этой стороны задача оказывается нерешаемой.

Таким образом, уже для очень простой задачи о двух электронахна одном центре водородоподобные функции оказываются практичес-ки бесполезными для количественных расчетов, а задача о компактноманалитическом решении для радиальной функции становится слишкомсложной и требует либо численного, либо аппроксимирующего аналити-ческого подхода. Главная трудность здесь, которую удалось обойти прирешении задачи об атоме водорода, заключается в слишком большомчисле зацепленных переменных. Возможность решения задачи определя-ется возможностью разделить переменные, и именно на это направленыстандартно используемые при аналитических расчетах приближения.

Page 39: КВАНТОВАЯ МЕХАНИКА МОЛЕКУЛ · PDF file2 ББК В22.31 УДК 544.11+539.183 isbn 978-5-94356-719-3 Стась Д.В., Плюснин В.Ф. Квантовая

2.3 УПРОЩАЮЩИЕ ПРЕДПОЛОЖЕНИЯ 39

2.3 Упрощающие предположения

Задача об описании электронного строения атома часто формулирует-ся в виде задачи о нахождении так называемых атомных орбиталей (АО).Этим термином мы уже интуитивно пользовались в предыдущей гла-ве при рассмотрении атома водорода как синонимом волновой функции.Однако для многоэлектронного атома понятие орбитали имеет особоезначение: орбиталью называют одноэлектронное состояние в многоэлек-тронной системе. Тем самым, когда говорится об орбиталях (атомных,молекулярных и т. д.), неявно делается очень важное предположениео возможности введения индивидуальных состояний для электронов вмногоэлектронной системе. Это позволяет отделить переменные, отно-сящиеся к «выбранному» электрону, и тем самым снизить размерностьрешаемой задачи. При этом приходится пренебрегать так называемымиэлектронными корреляциями — коллективными эффектами в наборе при-сутствующих в системе электронов. Масштаб вносимой при этом ошибкиможет быть достаточно велик, порядка эВ, что сопоставимо с масшта-бом энергий, определяющих все химическое поведение атома. Следу-ющее важное упрощающее предположение — о возможности сведениявсех остальных электронов, кроме «выбранного», к некоторому эффек-тивному потенциалу Ue f f(~r), что эффективно превращает задачу в одно-электронную, но с потенциалом, отличным от чисто кулоновского потен-циала притяжения электрона к ядру. Задача при этом решается итераци-онными методами, а сам подход получил название метода самосогласо-ванного поля, или метода Хартри—Фока. Наконец, можно сделать ещеодно естественное упрощающее предположение, о сферической симмет-рии эффективного потенциала, что позволяет выделить в АО радиальныеи угловые части, причем последние даются теми же сферическими гар-мониками, что и для атома водорода. Это позволяет довести разделениепеременных до конца и получить в итоге уравнение (точнее, системууравнений) на радиальную функцию.

Таким образом, при описании электронного строения многоэлектрон-ного атома приходится идти на следующие упрощающие предположения:

1. приближение о возможности введения индивидуальных одноэлек-тронных состояний в многоэлектронной системе;

2. приближение о возможности введения для каждого электрона неко-торого эффективного потенциала Ue f f(~ri), который описывает еговзаимодействие со всеми остальными электронами и не зависит

Page 40: КВАНТОВАЯ МЕХАНИКА МОЛЕКУЛ · PDF file2 ББК В22.31 УДК 544.11+539.183 isbn 978-5-94356-719-3 Стась Д.В., Плюснин В.Ф. Квантовая

40 ГЛАВА 2. МНОГОЭЛЕКТРОННЫЙ АТОМ

явным образом от их координат, что позволяет разделить наборыпеременных, относящихся к разным электронам;

3. приближение о сферически симметричном эффективном потенциа-ле Ue f f (~ri) →Ue f f(ri), что позволяет разделить переменные внутриуже расцепленных наборов для индивидуальных электронов.

В результате получаемые одноэлектронные состояния, как и в атоме

водорода, описываются волновой функцией вида Ψinlm = R(i)

nl (r)Ylm(ϑ ,ϕ), а

для нахождения радиальных функций R(i)nl (r) решается система интегро-

дифференциальных уравнений Хартри—Фока, о которых вам расскажутна курсе квантовой химии. При необходимости от этих приближенийможно последовательно отказаться и, например, учесть несферичностьэффективного потенциала или электронные корреляции, но это резкоподнимает сложность вычислений и переводит квантовохимический рас-чет на новый уровень. Сегодня все это делается, конечно, только накомпьютере. Возможность выделить угловую часть в отдельный сомно-житель и сложность нахождения радиальной функции придают особуюважность уже упоминавшемуся ранее «разделению ролей» угловой и ра-диальной частей волновой функции в атоме при расчетах: следующие изанализа угловых частей правила отбора позволяют без вычислений найтизаведомо нулевые матричные элементы, и только затем выполняется рас-чет требуемых ненулевых матричных элементов с найденными заранеерадиальными функциями.

2.4 Слэйтеровские орбитали

Методы самосогласованного поля дают возможность получить ради-альные функции только в виде набора чисел, практически непригодногодля простых аналитических оценок, которые можно было бы делать прос-то с помощью карандаша и бумаги. Для приближенных расчетов удобноиспользовать аналитические аппроксимации атомных орбиталей методасамосогласованного поля. Наибольшую известность из них получили такназываемые Слэйтеровские орбитали, предложенные в 30-х годах ХХ в.Дж. Слэйтером:

Ψ ∼ R(r) ·Ylm(ϑ ,ϕ); R(r) ∼(

z∗ra0

)n∗−1

e− z∗r

n∗a0 , (2.12)

Page 41: КВАНТОВАЯ МЕХАНИКА МОЛЕКУЛ · PDF file2 ББК В22.31 УДК 544.11+539.183 isbn 978-5-94356-719-3 Стась Д.В., Плюснин В.Ф. Квантовая

2.4 СЛЭЙТЕРОВСКИЕ ОРБИТАЛИ 41

где z∗ и n∗ — подгоночные параметры, имеющие смысл эффективногозаряда ядра и эффективного главного квантового числа. Можно видеть,что функции имеют вид старших функций для водородоподобного ионаRn,n−1 (1.27) с потенциалом

U(r) = −e2z∗

r+

h2n∗(n∗−1)

2mr2 (2.13)

и соответствуют энергиям

E∗ = −E0

2z∗2

n∗2 , (2.14)

а параметры z∗ и n∗ были подобраны таким образом, чтобы результа-ты оценок разумно согласовывались с экспериментальными данными порентгеновским спектрам атомов. Такие расчеты были выполнены прак-тически для всех элементов, и их результаты (правила Слэйтера) можносформулировать следующим образом:

1. значение n∗ определяется только главным квантовым числом дляописываемого электрона (номером его электронной оболочки):

n 1 2 3 4 5 6n∗ 1 2 3 3,7 4 4,2

(2.15)

2. эффективный заряд ядра z∗ для описываемого электрона меньшереального заряда ядра в атоме за счет частичного экранированияядра другими электронами:

z∗ = z−Sэкр. (2.16)

Константа экранирования Sэкр зависит от электронного строениявсего атома и находится следующим образом:

(a) все электроны в атоме делятся на группы: (1s), (2s,2p), (3s,3p),(3d), (4s,4p), (4d), (4 f ), (5s,5p), ..., при этом считается, чтовсе электроны внутри группы имеют одинаковую радиальнуюфункцию и соответственно одинаковую энергию, формируя еди-ный электронный слой;

(b) все электроны внешних по отношению к рассматриваемомуэлектрону групп не экранируют его;

(c) электрон сам себя не экранирует;

Page 42: КВАНТОВАЯ МЕХАНИКА МОЛЕКУЛ · PDF file2 ББК В22.31 УДК 544.11+539.183 isbn 978-5-94356-719-3 Стась Д.В., Плюснин В.Ф. Квантовая

42 ГЛАВА 2. МНОГОЭЛЕКТРОННЫЙ АТОМ

(d) все остальные электроны в той же группе дают в константуэкранирования вклад 0,35, за исключением 1s электронов, длякоторых вклад считается равным 0,3;

(e) вклад в константу экранирования для s и p электронов равен0,85 для каждого электрона с n на единицу меньше главногоквантового числа для рассматриваемого электрона и по 1 длявсех электронов с еще меньшим n;

(f) вклад в константу экранирования для d и f электронов от всехэлектронов внутренних групп считается равным 1.

В качестве примера давайте попробуем оценить энергии электронов,например, в атомах углерода и кислорода. В атоме углерода всего 6 элек-тронов, 2 на 1s, 2 на 2s и 2 на 2p орбиталях, заряд ядра +6, а у кислородана два 2p электрона больше при заряде ядра +8. Применяя изложенныевыше правила, получаем:

C : группы (1s) (2s,2p)n∗ 1 2

Sэкр 1×0,3 = 5,7 2×0,85+3×0,35= 2,75z∗ 6−0,3 = 5,7 6−2,75= 3,25

E∗ = 13,6z∗2

n∗2 442 36

Eсправ 269 16,6/11,3

(2.17)

O : группы (1s) (2s,2p)n∗ 1 2

Sэкр 1×0,3 = 5,7 2×0,85+5×0,35= 3,45z∗ 8−0,3 = 7,7 8−3,45= 4,55

E∗ = 13,6z∗2

n∗2 806 70

Eсправ 538 28,5/13,6.

Такого рода оценки полезны для ощущения порядка величин энер-гий в многоэлектронных атомах и дополняют уже известные нам оценкидля водородоподобных ионов. Аналогичные оценки можно сделать и дляразмеров многоэлектронных атомов, взяв качестве характерного размераположение максимума на распределении электронной плотности (1.30)для самого внешнего электрона атома:

r∗ =n∗2

z∗a0. (2.18)

Page 43: КВАНТОВАЯ МЕХАНИКА МОЛЕКУЛ · PDF file2 ББК В22.31 УДК 544.11+539.183 isbn 978-5-94356-719-3 Стась Д.В., Плюснин В.Ф. Квантовая

2.5 ДРУГИЕ СТАНДАРТНЫЕ БАЗИСЫ 43

В качестве характерных чисел приведем полученные из (2.18) оценки«атомного радиуса» r∗est и их принятые стандартные значения rstand дляуглерода 6C, кислорода 8O, калия 19K и хлора 17Cl, а также оценки иэкспериментальные значения для ионных радиусов 19K+ и 17Cl− (в пм):

r∗est rstand6C 67 708O 48 6017Cl 79 10019K 243 22017Cl− 83 18119K+ 62 138

(2.19)

Как можно видеть, для нейтральных атомов (по которым и калиброва-лось соотношение (2.18)) согласие получается достаточно разумное, а дляионных радиусов получается более чем двухкратное расхождение, хотяосновные тенденции передаются правильно. Любопытно отметить, чтоионы 19K+ и 17Cl− имеют одинаковую электронную оболочку и отличают-ся только зарядом ядра и соответственно z∗. Отметим также, насколькосильно добавление электрона в ион 19K+, дающее атом 19K, увеличива-ет размер атома по сравнению с добавлением электрона в атом 17Cl собразованием иона 17Cl−: в атоме калия электрон поступает на следую-щую электронную оболочку. Приближенные соотношения (2.18) и (2.14)оказываются очень полезны, когда нужно быстро оценить характерныеразмеры и энергии атомов, не требуя слишком большой точности.

2.5 Другие стандартные базисы

Физический смысл Слэйтеровских орбиталей достаточно прозрачен —это асимптотики волновых функций при больших r, распространенныена всю область определения. Поэтому функции (2.12) разумно описыва-ют асимптотическое поведение атомных орбиталей на бесконечности, ноне могут воспроизвести АО приближения самосогласованного поля прималых r, в частности, они вовсе не имеют нулей внутри области опреде-ления. Для улучшения функции (2.12) на малых r можно ввести в неееще одну, более быстро затухающую экспоненту:

Ψnlm ∼ rn∗−1(

ae−ζ1r +be−ζ2r)

Ylm(ϑ ,ϕ). (2.20)

Такие функции известны как функции дубль-дзета (DZ) базиса.

Page 44: КВАНТОВАЯ МЕХАНИКА МОЛЕКУЛ · PDF file2 ББК В22.31 УДК 544.11+539.183 isbn 978-5-94356-719-3 Стась Д.В., Плюснин В.Ф. Квантовая

44 ГЛАВА 2. МНОГОЭЛЕКТРОННЫЙ АТОМ

Орбитали Слэйтеровского типа оказались очень неудобны в расчетахмногоцентровых систем из-за экспоненциальной зависимости от модулярасстояния от электрона до ядра, поскольку интегрировать произведенияразноцентровых функций вида e−|~r−~R1| · e−|~r−~R2| очень тяжело. Эту про-блему частично решает популярное сегодня разложение функций (2.12)по гауссовым функциям вида:

Ψnlm ∼ rn−1e−αr2Ylm(ϑ ,ϕ), (2.21)

произведение которых снова эффективно дает одноцентровую гауссовуфункцию. Даже с учетом возросшего числа рассчитываемых интеграловэто значительно упрощает вычисления. Широкое распространение полу-чили разложения вида «STO-3G» (Slater Type Orbital = 3 Gaussians),и особенно так называемые контрактированные базисные наборы вида«6-31G» (для элементов второго периода). Символ «6» означает, что ор-битали внутренних 1s электронов представляются суммой шести гаус-соид, а «31» указывает, что орбитали валентных (2s и 2p) орбиталейсоставлены из двух частей, внутренней из трех гауссоид и одной болеедиффузной внешней гауссоиды. Цель введения последней компоненты —частично компенсировать слишком быстрое затухание гауссоиды по срав-нению с экспонентой, которую функция должна аппроксимировать прибольших r. Такие линейные комбинации оптимизированы и фиксирова-ны для всех элементов первого и второго периодов. Например, наборфункций для атома углерода содержит одну 1s орбиталь (фиксированнаялинейная комбинация шести гауссоид) и по две фиксированные комби-нации, из трех и из одной гауссоид, для каждой из 2s, 2px, 2py и 2pz

орбиталей — итого всего 9 функций, а для набор функций для атомаводорода состоит из двух 1s функций (3+1 гауссоид).

Еще больше поднять гибкость базиса можно добавлением в базис такназываемых поляризованных функций: d-функций к набору для атомоввторого периода (получаемый при этом базис содержит по 15 функцийдля каждого из атомов: 1 — 1s, по 2 — 2s, 2px, 2py и 2pz, и шесть декар-товых d-функций, он обозначается «6-31G(d)» или «6-31G∗») и возможноеще p-функций к набору для атома водорода, который теперь разрастает-ся до 5 функций (такой базис обозначается «6-31G(d,p)» или «6-31G∗∗»).Введение поляризованных функций преследует ту же цель, что и пе-реход к базису гибридных орбиталей для водородоподобного атома, иделает более естественным описание направленных связей.

Page 45: КВАНТОВАЯ МЕХАНИКА МОЛЕКУЛ · PDF file2 ББК В22.31 УДК 544.11+539.183 isbn 978-5-94356-719-3 Стась Д.В., Плюснин В.Ф. Квантовая

2.6 ЭЛЕКТРОННАЯ КОНФИГУРАЦИЯ АТОМА 45

¡¢£¤

¥¦§¨©¨

ª«¬­

®¯®°®±®¯²°²±

³ ´ µ³ µ´ ¶³ ¶´³µ¶·¸´¹º

»¼½¾¿¾

ÀÁÂÃ

ÄÅÆÄÇÈÄÇÆ

É Ê ËÉ ËÊ ÌÉ ÌÊÉËÌÍÎÊÏÐ

ÑÒÓÔÕÔ

Ö×ØÙ

ÚÛÚÜÚÝÞÚÝßÚÝà

áâ ãâ

äâ åâ

Рис. 2.1. Распределение электронной плотности 4πρ2R2nl для водородоподобных

функций с n = 1,2,3: а) для индивидуальных функций; б) просуммированное пооболочкам; в) проникновение 2s и 2p орбиталей через оболочку с n = 1; г) про-

никновение 3s, 3p и 3d орбиталей через оболочки с n = 1,2.

2.6 Электронная конфигурация атома

Последовательность заполнения электронных оболочек в многоэлек-тронном атоме регулируется двумя принципами:

1. принцип последовательного заполнения: электроны добавляются поодному в состояние с минимально возможной энергией;

2. принцип Паули: в атоме не может быть более одного электронас набором квантовых чисел n, l,m,ms, где ms = ±1

2 — спиновоеквантовое число, то есть в каждом состоянии n, l,m, может бытьне более двух электронов.

В многоэлектронном атоме из-за отличия эффективного потенциалаот кулоновского снимается имевшее место в атоме водорода вырождениепо l, и энергии начинают зависть от n и l (но по-прежнему не от m

Page 46: КВАНТОВАЯ МЕХАНИКА МОЛЕКУЛ · PDF file2 ББК В22.31 УДК 544.11+539.183 isbn 978-5-94356-719-3 Стась Д.В., Плюснин В.Ф. Квантовая

46 ГЛАВА 2. МНОГОЭЛЕКТРОННЫЙ АТОМ

для «круглого» изолированного атома). В общем случае с повышениематомного номера (то есть заряда ядра) энергии формально одноименныхорбиталей постепенно снижаются, но чем больше l, тем позднее начина-ется это снижение.

Для объяснения этого эффекта используется очень простая идея о«проникновении» орбиталей. Электронная плотность в атоме расположе-на не четко разграниченными в пространстве слоями, но распределенапо r. Типичный вид радиального распределения электронной плотностидля функций с n = 1,2,3 приведен на рис. 2.1. Распределения для раз-ных оболочек даются достаточно широкими максимумами с крыльями,доходящими до r = 0. Из рис. 2.1в можно видеть, что орбитали 2s и2p «проникают» сквозь 1s орбиталь, но как показывает расчет, для 2sорбитали «внутри» электронной плотности 1s электронов лежит боль-шая доля плотности, чем для 2p орбитали — говорят, что 2s орбитальпроникает к ядру сильнее, чем 2p орбиталь. Еще лучше это видно дляорбиталей с n = 3, проникающими через внутренние для них оболоч-ки с n = 1,2 (рис. 2.1г). Рис. 2.1 иллюстрирует общую ситуацию: чембольше l, тем при том же n орбиталь меньше проникает к ядру, темэффективнее расположенные на ней электроны экранируются от ядрадругими электронами. А поскольку понижение энергии (увеличение поабсолютной величине отрицательной энергии) орбитали с увеличениематомного номера связано прежде всего с бoльшим кулоновским притя-жением к ядру, то чем меньше экранируется электрон от ядра, тем нижебудет его энергия. В нашем примере при занятой 1s орбитали 2s орби-таль будет иметь энергию ниже, чем 2p орбиталь, и разность энергий 2sи 2p состояний остается на уровне около 200 кДж/моль (несколько эВ)для всех элементов, начиная с лития.

На рис. 2.2 слева показано расчетное изменение энергии атомныхорбиталей в многоэлектронном атоме при увеличении заряда ядра. Мож-но видеть, что орбитали с одинаковым главным квантовым числом nпри Z = 1 стартуют с одинаковых энергий, как и должно быть для атомаводорода, и постепенно опускаются по энергии с ростом Z. Однако чемвыше орбитальный момент, тем позднее орбиталь начинает замечать уве-личение заряда ядра, которое компенсируется экранирующим эффектомэлектронов внутренних оболочек. Особенно хорошо это заметно для fорбиталей, энергия которых просто не зависит от атомного номера призаполнении оболочек с малыми n. Именно это обстоятельство и былоположено в основу деления электронов на группы при введении Слэйте-ровских орбиталей. Можно также видеть, что 3d орбиталь в некотором

Page 47: КВАНТОВАЯ МЕХАНИКА МОЛЕКУЛ · PDF file2 ББК В22.31 УДК 544.11+539.183 isbn 978-5-94356-719-3 Стась Д.В., Плюснин В.Ф. Квантовая

2.6 ЭЛЕКТРОННАЯ КОНФИГУРАЦИЯ АТОМА 47

æç èç éç êç ëçç

æçççç

æççç

æçç

æç

ìíîïðñòóôõ

ö÷øùúûü úøùýþÿ

ÿ

ÿ

ÿ

ÿ

!"# $!!"%&'!

()*+,-. ,*+/0

1234

Рис. 2.2. Расчетная зависимость энергий индивидуальных орбиталей в многоэ-лектронном атоме от атомного номера (слева) и экспериментально измеренныеэнергии внутренних электронов (справа). Большее количество уровней связано с

дополнительным релятивистким расщеплением уровней в тяжелых атомах.

диапазоне атомных номеров оказывается по энергии выше 4s орбитали.Подобные эффекты приводят к отличию порядка заполнения орбиталей вмногоэлектронном атоме от интуитивно ожидаемого «сначала все с n = 1,потом все с n = 2, затем все с n = 3, . . . ». Справа на рис. 2.2 показанэкспериментальный аналог расчетных энергий орбиталей — полученныеиз экспериментальных рентгеновских спектров энергии внутренних элек-тронов в многоэлектронных атомах. К сожалению, таким образом можноизмерить только энергии занятых в атоме орбиталей, поэтому у кривыхотсутствуют наиболее интересные начальные участки с малыми Z. Мож-но также заметить, что на рисунке имеется больше кривых, чем можнобыло бы ожидать исходя из количества атомных орбиталей. Связано этос дополнительным расщеплением уровней энергии в атоме, вызваннымвзаимодействием со спином электрона, поэтому каждый уровень с l > 0расщепляется на два.

Page 48: КВАНТОВАЯ МЕХАНИКА МОЛЕКУЛ · PDF file2 ББК В22.31 УДК 544.11+539.183 isbn 978-5-94356-719-3 Стась Д.В., Плюснин В.Ф. Квантовая

48 ГЛАВА 2. МНОГОЭЛЕКТРОННЫЙ АТОМ

l

n 0 1 2 3

1 1s X X X

2 2s 2p X X

3 3s 3p 3d X

4 4s 4p 4d 4f

5 5s 5p 5d 5f

6 6s 6p 6d 6f

7 7s 7p 7d 7f

Рис. 2.3. Эмпирически установленная последовательность заполнения уровней в

многоэлектронном атоме.

Общее правило, описывающее эмпирически установленный порядокзаполнения электронных состояний, известно как правило (n + l), илиправило Клечковского: в многоэлектронном атоме первыми заполняют-ся состояния с минимально возможной суммой (n + l), а внутри каждойгруппы с одинаковым (n + l) — состояния с минимальным n. На рис. 2.3приведено удобное мнемоническое изображение этого правила: нарисуемтаблицу, перенумеруем ее строки и столбцы возможными значениями nи l, соответственно, затем вычеркнем несуществующие комбинации nl,такие как 1p и 2 f , и последовательно пойдем сверху вниз по диагона-лям, стартуя с левого верхнего угла — состояния 1s, получая требуемуюпоследовательность: 1s; 2s; 2p, 3s; 3p, 4s; 3d, 4p, 5s; 4d, 5p, 6s;...

Электронную структуру атомов символически записывают в виде такназываемой электронной конфигурации, указывая, сколько электроновнаходится на каких орбиталях. На рис. 2.4 приведена таблица Менде-

Page 49: КВАНТОВАЯ МЕХАНИКА МОЛЕКУЛ · PDF file2 ББК В22.31 УДК 544.11+539.183 isbn 978-5-94356-719-3 Стась Д.В., Плюснин В.Ф. Квантовая

2.6 ЭЛЕКТРОННАЯ КОНФИГУРАЦИЯ АТОМА 49

5 5 5 5 5 5 56 6 6 5 6 55 6 5 5 57 8 9 7

7 :; ; <7= >

? 8 @ 7A 8 B 9 CD :; ; C E7= F

C G H 9 IE :J D 7C = >

K B 9 DJ :; 7C CC = F

K 9 L7; :< 7 7C = F C M >

N 9 E7C :; 7 7C = F C M F

O 9 P7D :; ; E PC = F C M Q

R 9 <7L :J J JC = F C M S

T @ J7< :J J <C = F C M U

O B 9 7;C ; : 7P JC = F C M V

I O W 9 7 7C C :J JI = >

X Y 9 7 CC D :I ; LI = F

Z [ 9 7 IC E :J <I = F I M >

\ H 9 7DC < :; < L LI = F I M F

] 9 7 LI ; :J P DI = F I M Q

\ 9 7 EI C :; E EI = F I M S

N [ @ 7 PI L :D L II = F I M U

Z ^ 9 7 <I J :J D <I = F I M V

D _ 9 7 JI < :; J <D = >

N W 9 C ;D ; :; P <D = F

\ ` 9 C 7D D :J L EI a >D = F

b H 9 C CD P :< <I a F D = F

6 9 C IL ; :J D CI a Q D = F

N ^ 9 C DL 7 :J J EI a U D = >

X c 9 C LL D :J I <I a U D = F

T B 9 C EL L :< D PI a V D = F

N d 9 C PL < :J I II a e D = F

O H 9 C <L < :E JI a f D = F

N g 9 C JE I :L D EI a >h D = >

i c 9 I ;E L :I JI a >h D = F

j W 9 I 7E J :P C ID = F D M >

j B 9 I CP C :L JD = F D M F

Z = 9 I IP D :J C CD = F D M Q

\ B 9 I DP < :J ED = F D M S

K ^ 9 I LP J :J ; DD = F D M U

_ ^ 9 I E< I :< ;D = F D M V

L k l 9 I P< L :D E <L = >

\ ^ 9 I << P :E CL = F

m 9 I J< < :J ; ED a >L = F

i ^ 9 D ;J 7 :C C DD a F L = F

O l 9 D 7J C :J ; ED a S L = >

X d 9 D CJ L :J DD a U L = >

b ` 9 D IJ P :J ; PD a U L = F

k g 9 D D7; 7 :; PD a e L = >

k n 9 D L7; C :J ; ED a f L = >

] a 9 D E7; E :D CD a >h L = h

Z Y 9 D P7; P :< E <D a >h L = >

N a 9 D <7 7 C :D 7D a >h L = F

5c 9 D J7 7 D :< CL = F L M >

\ c 9 L ;7 7 < :P 7L = F L M F

\ l 9 L 77C 7 :P LL = F L M Q

b B 9 L C7C P :< ;L = F L M S

5 9 L I7C E :J ; LL = F L M U

o B 9 L D7I 7 :C JL = F L M V

E N = 9 L L7I C :J ; DE = >

K W 9 L E7I P :I IE = F

G W 9 L P7I < :J ; EL a >E = F

8 p 9 P C7P < :D JL a F E = F

b W 9 P I7< ; :J D <L a Q E = F

q 9 P D7< I :< LL a S E = F

k B 9 P L7< E :C ; PL a U E = F

R = 9 P E7J ; :CL a V E = F

5^ 9 P P7J C :C CL a e E = F

] r 9 P <7J L :; <L a s E = >

Z g 9 P J7J E :J E EL a >h E = >

8 Y 9 < ;C ; ; :L JL a >h E = F

b [ 9 < 7C ; D :I < IE = F E M >

] l 9 < CC ; P :CE = F E M F

K H 9 < IC ; < :J <E = F E M Q

] d 9 < DC ; < :J < CE = F E M S

Z r 9 < LC ; J :J < PE = F E M U

k c 9 < EC C C :; 7<E = F E M V

P T ^ 9 < PC C I :; C ;

P = >

k W 9 < <C C E :; C L

P = F

Z ` 9 < JC C P :; C <E a >P = F

_ g 9 7; DC E 7

E a F P = F

O = 9 7; LC E C

E a Q P = F

N B 9 L <7D ; :7 C

D p > L a >E = F

] ^ 9 L J7D ; :J ; <D p Q E = F

O a 9 E ;7D D :C DD p S E = F

] t 9 E 77D D :J 7ID p U E = F

\ t 9 E C7L ; :I ED p V E = F

u g 9 E I7L 7 :J ED p e E = F

j a 9 E D7L P :C L

D p e L a >E = F

b l 9 E L7L < :J C LD p s E = F

v w 9 E E7E C :L ;D p > h E = F

8 d 9E P7E D :J ID p > >E = F

u ^ 9 E <7E P :C ED p > F E = F

b t 9 E J7E < :J I DD p > Q E = F

m l 9 P ;7P I :; DD p > S E = F

G g 9 P 77P D :J E PD p > S L a >E = F

b n 9 J ;C I C :; I <E a F P = F

] W 9 J 7C I 7 :; I EL p F E a >P = F

x 9 J CC I < :; C JL p Q E a >P = F

O M 9 J IC I P :; D <L p S E a > P = F

] g 9 J DC D D :; E DL p V P = F

Z t 9 J LC D I :; E 7L p e P = F

N t 9 J EC D P :; PL p e E a >P = F

K y 9 J PC D P :; PL p f E a >P = F

N p 9 J <C L 7 :; <L p > h P = F

u = 9 J JC L C :; <L p > >P = F

T t 9 7 ; ;C L P :; J LL p > F P = F

X a 9 7 ; 7C L < :; J JL p > Q P = F

O d 9 7 ; CC L J :7L p > S P = F

G ^ 9 7 ; IC E ; :7 ; LL p > S E a >P = F

Рис. 2.4. Таблица Менделеева с указанием электронных конфигураций атомов.

Page 50: КВАНТОВАЯ МЕХАНИКА МОЛЕКУЛ · PDF file2 ББК В22.31 УДК 544.11+539.183 isbn 978-5-94356-719-3 Стась Д.В., Плюснин В.Ф. Квантовая

50 ГЛАВА 2. МНОГОЭЛЕКТРОННЫЙ АТОМ

леева с указанием электронных конфигураций элементов. Электронныеконфигурации водорода H и гелия He — соответственно 1s1 и 1s2, литияLi — 1s22s1, бериллия Be — 1s22s2, бора B — 1s22s22p1 и т. д. Элементыот лития до неона Ne составляют второй, или первый короткий период.Напомним, что у каждого атома свой набор орбиталей, и, например, 2pорбитали атомов кислорода и углерода вовсе не одинаковы. Более того,набор орбиталей меняется даже при изменении числа электронов в атоме(ионизации) или при переводе атома в состояние с возбужденной элек-тронной конфигурацией, типичный пример которого — так называемоепромотирование электрона в следующее доступное по энергии состояниедля увеличения количества неспаренных электронов, которые и опреде-ляют химическую активность атома. Однако классификация по типамсостояний тем не менее оказывается очень удобна, поскольку отражаетсимметрийные свойства орбиталей (s, p, d,...) и их относительные энер-гии (1 — самое низкое по энергии состояние данной симметрии в данноматоме, 2 — следующее и т. д.).

2.7 Химические свойства атома

Электронное строение атома определяет его химические свойства,причем определяющую роль здесь играет строение самой внешней час-тично заполненной, так называемой валентной оболочки, энергии элек-тронов на которой сопоставимы с энергиями образования химическихсвязей. Энергии внутренних электронов атомов слишком велики (см.оценки 2.14, 2.17), поэтому внутренние электроны практически не чувст-вуют химическое окружение атома, что позволяет использовать спектро-скопию переходов с их участием — рентгеновскую спектроскопию — дляидентификации атомов в составе химических соединений. Расстояние поэнергии от валентной до следующей свободной оболочки (следующий попорядку n) при малых n тоже обычно достаточно велики, чтобы исклю-чить участие последней в химизме атома. Таким образом, химическоеповедение атома в первом приближении определяется числом имеющих-ся электронов и свободных мест (дырок) в валентной оболочке.

Атом водорода, в зависимости от своего партнера по связи, можетлибо отдать свой единственный электрон с образованием иона H+, либопринять еще один электрон до заполнения 1s оболочки с образовани-ем гидрид-иона H−, либо образовать одну ковалентную связь, то естьсвязь через общую пару электронов. Атом гелия химически абсолют-

Page 51: КВАНТОВАЯ МЕХАНИКА МОЛЕКУЛ · PDF file2 ББК В22.31 УДК 544.11+539.183 isbn 978-5-94356-719-3 Стась Д.В., Плюснин В.Ф. Квантовая

2.7 ХИМИЧЕСКИЕ СВОЙСТВА АТОМА 51

но инертен, будучи не в состоянии ни отдать, ни принять электрон, неимея неспаренных электронов и не имея возможности промотирования.Литий легко отдает свой единственный внешний 2s электрон, оставаясьв формальной электронной конфигурации He — отобрать у него вто-рой электрон химически невозможно. Поэтому литий либо выступает вхимических процессах как однозарядный катион, либо образует одну ко-валентную связь, то есть одновалентен. Li и его гомологи (Na, K, Rb, Cs,Fr), имеющее сходное строение внешней электронной оболочки, называ-ются щелочными металлами.

Следующий за литием элемент, бериллий Be, имеет основную элек-тронную конфигурацию 1s22s2 и формально не способен к образованиюковалентной химической связи. Кроме того, его первый и второй потен-циалы ионизации (соответственно 9,3 и 18,2 эВ) настолько велики, чтокатионы в химических процессах он практически не образует, а сродст-во к электрону отрицательно, так что образование аниона из атома тоженевыгодно. Однако реально бериллий образует две ковалентные связи:один из 2s электронов промотируется в относительно близколежащеесостояние 2p, а требуемая для этого энергия черпается из образова-ния химических связей. Be и его гомологи по электронному строениюобразуют вторую группу периодической системы, так называемые ще-лочноземельные металлы. Бор B (электронная конфигурация 1s22s22p1 ≡[He]2s22p1) оказывается совершенно неспособен к отдаче электрона иобразует по три ковалентные связи, промотируя один из 2s электроновна 2p орбиталь. Углерод C в основной конфигурации ([He]2s22p2) дол-жен был бы быть двухвалентным, но реально почти всегда оказываетсячетырехвалентным за счет промотирования до конфигурации [He]2s12p3.Азот N (электронная конфигурация [He]2s22p3), кислород O ([He]2s22p4)и фтор F ([He]2s22p5) всегда соответственно трех-, двух- и одновалент-ны в ковалентных соединениях, в этом же ряду растет их склонность кобразованию отрицательных ионов. Неон Ne, подобно гелию, имеет пол-ностью заполненную электронную оболочку и не имеет низколежащихсвободных орбиталей для промотирования электронов, а потому такжехимически совершенно инертен. Группы гомологов кислорода, фтора инеона называют соответственно группами халькогенов, галогенов и бла-городных газов.

Для всех элементов второго периода выполняется «правило октета» —атом имеет не более восьми валентных электронов, удвоенного числадоступных орбиталей (одна 2s и три 2p орбитали). Элементы со свобод-ными местами в валентной оболочке (бериллий, бор) могут принимать на

Page 52: КВАНТОВАЯ МЕХАНИКА МОЛЕКУЛ · PDF file2 ББК В22.31 УДК 544.11+539.183 isbn 978-5-94356-719-3 Стась Д.В., Плюснин В.Ф. Квантовая

52 ГЛАВА 2. МНОГОЭЛЕКТРОННЫЙ АТОМ

них пары электронов, выступая в качестве акцептора пары электронов,или кислоты Льюиса, а элементы с более чем наполовину заполнен-ными оболочками (азот, кислород, фтор) — подавать свою «неподелен-ную пару» на свободную орбиталь акцептора, выступая в роли донорапары электронов, или основания Льюиса. Образуемые при этом связиназываются донорно-акцепторными, или дативными. В качестве типич-ного примера можно привести устойчивые тетрафторборат-анион BF−

4 икатион аммония NH+

4 , по строению сходные с молекулой метана CH4:четыре эквивалентные связи и геометрия правильного тетраэдра. В ка-честве акцептора пары при образовании дативной связи может выступатьи атом с более чем наполовину заполненной оболочкой, нужна лишь воз-можность предоставить свободную орбиталь. Примером может служитьN-оксидная группа ≥ N → O, в которой атом кислорода принимает насвою свободную 2p орбиталь неподеленную пару атома азота с образова-нием формально четырехвалентного атома азота и одновалентного атомакислорода, при этом оба атома имеют полностью заполненные валент-ные оболочки из восьми электронов. При образовании дативной связидонор пары получает положительный заряд, что снижает его электронно-донорные свойства, а акцептор пары — отрицательный заряд, что такжеснижает его акцепторные свойства. Поэтому количество образуемых ато-мом дативных связей обычно невелико, не превышает двух, даже еслиформально он способен на образование дополнительных связей.

У элементов третьего периода и выше появляются относительно низ-колежащие 3d орбитали, на которые можно промотировать электроны,поэтому они могут проявлять и более высокие валентности по сравнениюсо своими гомологами из второго периода, например, 5 для фосфора P(максимум 3 для азота) и 6 для серы S (максимум 2 для кислорода).Даже в группе благородных газов, начиная с криптона Kr, становит-ся химически возможным промотирование, и они начинают проявлятьнекоторую химическую активность. Так, на сегодня известно более 100соединений криптона и ксенона, например, XeF2, XeF4, XeO2. Посколькудоступных орбиталей стало больше, правило октета для этих элементовуже не является ограничением, и, например, в SF6 центральный атомсеры имеет вокруг себя 12 валентных электронов. Эта молекула имеетгеометрию октаэдра, а для описания ее центрального атома удобно ис-пользовать sp3d2 гибридные орбитали. Помимо появления доступных dорбиталей, у элементов третьего периода снижается эффективность свя-зывания через p орбитали, которые становятся более рыхлыми и имеют ктому же радиальную узловую поверхность (см. рис. 1.6). Это приводит к

Page 53: КВАНТОВАЯ МЕХАНИКА МОЛЕКУЛ · PDF file2 ББК В22.31 УДК 544.11+539.183 isbn 978-5-94356-719-3 Стась Д.В., Плюснин В.Ф. Квантовая

2.7 ХИМИЧЕСКИЕ СВОЙСТВА АТОМА 53

тому, что формально имеющие одинаковые валентные оболочки гомоло-ги имеют существенно разные химические свойства. Так, например, длякислорода и азота наиболее устойчивыми гомоядерными молекулами яв-ляются двухатомные O2 и N2, причем последняя — самая прочная из всехдвухатомных гомоядерных молекул. Однако молекулы S2 и P2 неустойчи-вы, элементные сера и фосфор существуют во множестве модификаций,из которых наиболее стабильны корона S8 и пирамидка P4.

Начиная со скандия (21Sc) происходит заполнение 3d оболочки. Блокэлементов от 21Sc до меди 29Cu называется первым переходным периодом,а их аналоги с 4d орбиталями (от иттрия 39Y до серебра 47Ag) — вторымпереходным периодом. Заполнение 4 f оболочки происходит в элементахот лантана 57La до лютеция 71Lu — лантаноидах, а 5 f оболочки — вактиноидах от актиния 89Ac до лоренция 103Lr. В переходных элементахпервого блока заполнение d оболочки происходит достаточно регулярно,за исключением скачка ванадий 23V ([Ar]3d34s2) → хром 24Cr([Ar]3d54s1)и никель 28Ni([Ar]3d84s2) → медь 29Cu([Ar]3d104s1). Чтобы понять такойскачок, уточним запись электронной конфигурации, приняв во внимание,что а) электроны в атоме занимают разные орбитали, пока это энерге-тически допустимо, и б) находящиеся на разных орбиталях неспарен-ные электроны имеют параллельные спины. Другими словами, наиболееустойчивой электронной конфигурацией среди нескольких возможныхс формально одинаковой записью будет конфигурация с максимальнымчислом неспаренных электронов. При последовательном добавлении ватом электронов они, пока возможно, занимают разные орбитали одноготипа (например, 3d), что и приводит к большей устойчивостью наполо-вину заполненных (на каждой орбитали по одному электрону при мак-симально возможном заряде ядра) и полностью заполненных оболочек,в нашем случае 3d5 и 3d10, соответственно. При заполнении 4d и 5dоболочки конфигурации nd5 и nd10 также выделены, но скачки в запол-нении электронной оболочки здесь более неопределенные из-за уменьше-ния энергетических зазоров при повышении n. Аналогичные нерегуляр-ности, только еще менее определенные, наблюдаются и при заполнении4 f и 5 f оболочек в лантаноидах и актиноидах, где выделены конфигу-рации n f 7 и n f 14. Все d и f элементы имеют внешние ns электроны ипотому являются металлами.

Наконец, отметим возможную несимметричность добавления электро-на при переходе к следующему элементу (с увеличением на единицу за-ряда ядра) и потери электрона при ионизации атома. Так, например, припереходе от марганца 25Mn (электронная конфигурация [Ar]3d54s2) к же-

Page 54: КВАНТОВАЯ МЕХАНИКА МОЛЕКУЛ · PDF file2 ББК В22.31 УДК 544.11+539.183 isbn 978-5-94356-719-3 Стась Д.В., Плюснин В.Ф. Квантовая

54 ГЛАВА 2. МНОГОЭЛЕКТРОННЫЙ АТОМ

z | ~

z

z

¡¢

£¤

¥ ¦

§¨

©ª

«¬

­¤

©®

£¯

°¢

±²

³¤

°

´ ¢

µ²

µ¬

¶·

§¬

§¹º»¼

½¾¿ÀÁÂà Á¿ÀÄÅ

Рис. 2.5. Зависимость потенциала ионизации атома от атомного номера (слева)и растянутый участок кривой для элементов второго периода (справа).

лезу 26Fe ([Ar]3d64s2) в атоме добавляется один d электрон. Однако приионизации атома железа с потерей одного электрона уходит s электрон:экспериментально полученная основная электронная конфигурация ио-на Fe+ — [Ar]3d64s1. Подобная же ситуация может иметь место и приприсоединении электрона, например, при добавлении электрона в атомскандия 21Sc ([Ar]3d14s2) c образованием отрицательного иона в атомепоявляется p электрон (основная электронная конфигурация иона 21Sc−

[Ar]3d14s24p1), хотя сосед скандия по периодической системе, титан 22Ti,имеет конфигурацию [Ar]3d24s2. Поэтому, если по каким-то причинамнужно знать точную электронную конфигурацию атома или иона, необ-ходимо обращаться к спектроскопичаским справочникам.

2.8 Потенциал ионизации и сродство

к электрону

Давайте посмотрим на типичные величины и закономерности изме-нения энергетических характеристик атома — потенциалов ионизации исродства к электрону — при движении по периодической системе. Придвижении сверху вниз по столбцу таблицы («по группе») потенциалыионизации монотонно падают. Вызвано это ростом размеров атомов дляболее тяжелых элементов-гомологов. В результате теряемые при иони-

Page 55: КВАНТОВАЯ МЕХАНИКА МОЛЕКУЛ · PDF file2 ББК В22.31 УДК 544.11+539.183 isbn 978-5-94356-719-3 Стась Д.В., Плюснин В.Ф. Квантовая

2.8 ПОТЕНЦИАЛ ИОНИЗАЦИИ И СРОДСТВО К ЭЛЕКТРОНУ 55

зации внешние электроны с ростом атомного номера становятся все бо-лее внешними и легче уходят. При движении слева направо по строкамтаблицы («по периоду») потенциал ионизации атома растет почти моно-тонно. Связано это с доминирующим эффектом увеличения заряда ядра.Зависимость потенциала ионизации IP от номера элемента Z показанана рис. 2.5. Можно отметить следующие характерные особенности кри-вой IP(Z): нижнюю полку дают атомы щелочных металлов (от Li до Cs)с потенциалами ионизации на уровне 4-5 эВ. Верхняя огибающая — бла-городные газы. Самый высокий потенциал ионизации имеет атом гелия(24,5 эВ), затем при движении по группе он постепенно снижается.

На примере элементов второго периода (от лития до неона) мож-но проследить более тонкие закономерности изменения IP внутри пе-риода. Соответствующий участок кривой I(Z) показан в увеличенноммасштабе слева на рис. 2.5. С увеличением атомного номера IP(Z) рас-тет, однако вместо строго монотонной зависимости кривая IP(Z) дваж-ды проваливается. Первый раз это происходит между Be (IP ∼ 9,3 эВ)и B (IP ∼ 8,3 эВ) — связано это с началом заполнения 2p подоболоч-ки, которая, как мы уже обсуждали, энергетически не столь выгод-на, как 2s подоболочка. Второй раз кривая IP(z) проваливается междуN (IP ∼ 14,5 эВ) и O (IP ∼ 13,6 эВ) — здесь это объясняется началом«подселения» вторых электронов на уже занятые орбитали, посколькунеобходимость размещения пары электронов на одной пространствен-ной орбитали также приводит к некоторому проигрышу энергии из-зароста межэлектронного отталкивания. Та же картина затем повторяет-ся и при заселении s и p оболочек в следующих периодах. Потенциалыионизации для d и f элементов одного блока очень близки, что ещераз подчеркивает роль «внешних» s и p электронов, определяющих хи-мическое поведение атома. Отметим для дальнейшего некоторые репер-ные значения, которые полезно держать в голове: IP(щел. мет.) ∼ 5 эВ,IP(H) ∼ IP(O) ∼ 13,6 эВ, IP(He) ≡ IPmax ∼ 24,5 эВ.

Сходные эффекты проявляются и на зависимости сродства атома кэлектрону EA от порядкового номера элемента. На рис. 2.6 приведенакривая EA(Z) для начала таблицы Менделеева. Можно заметить, что, вотличие от кривой IP(Z), на кривой EA(Z) имеются периодические разры-вы, поскольку ряд элементов не имеет положительного сродства к элек-трону. Это прежде всего благородные газы Ne, Ar, Kr, Xe с полностьюзаполненной электронной оболочкой для данного n, у которых дополни-тельный электрон может разместиться только на следующую оболочкуc n + 1, которая лежит значительно выше по энергии. Другую группу

Page 56: КВАНТОВАЯ МЕХАНИКА МОЛЕКУЛ · PDF file2 ББК В22.31 УДК 544.11+539.183 isbn 978-5-94356-719-3 Стась Д.В., Плюснин В.Ф. Квантовая

56 ГЛАВА 2. МНОГОЭЛЕКТРОННЫЙ АТОМ

Æ ÇÆ ÈÆ ÉÆ ÊÆ ËÆ ÌÆ

Æ

Ç

È

É

Ê

ÍÎÏÐÑÒ

ÓÔÕÖ

×Ø

ÙÚÛÚ

ÜÝÞÎ

ßà

áâ

ãà

äå

×ÔæÚ

çèÎ

ãéêÖ

Ñëì

Ïíá

ê

ë

ÓÖ

Íå

îâ

î

ïðñòóôõ óñòö÷

øùúûü

Рис. 2.6. Зависимость сродства атома к электрону от атомного номера для эле-ментов второго периода.

с отрицательным сродством к электрону составляют щелочноземельныеметаллы Be, Mg, Ca, Sr, Ba (а также примыкающие к ним Zn и Cd) с элек-тронной конфигурацией ns2 (соответственно (n−1)d10ns2), у которых дляразмещения дополнительного электрона нужно переходить на p подо-болочку. Наконец, отметим отрицательное сродство к электрону атомовазота (электронная конфигурация 2s22p3) и марганца (3d54s2), у которыхдополнительный электрон должен попасть на ровно наполовину запол-ненную подоболочку (соответственно p3 и d5). Самое высокое сродствок электрону имеют атомы галогенов (порядка 3,5 эВ).

Page 57: КВАНТОВАЯ МЕХАНИКА МОЛЕКУЛ · PDF file2 ББК В22.31 УДК 544.11+539.183 isbn 978-5-94356-719-3 Стась Д.В., Плюснин В.Ф. Квантовая

Глава 3

Классификация термов

изолированного

многоэлектронного атома

В предыдущей главе мы обсудили, как устроена электронная оболочкамногоэлектронного атома. Теперь давайте обратимся к спектроскопичес-кой классификации атомных состояний. Как и любая реальная физи-ческая задача, задача о классификации состояний атома относительнопросто решается только в рамках некоторых приближений, построенныхна иерархии последовательно накладываемых возмущений и связанных свозможностью разделить взаимодействия по относительным величинамиих энергий. С анализа последних мы и начнем.

3.1 Иерархия взаимодействий

в многоэлектронном атоме

В многоэлектронном атоме имеются следующие четыре основные вза-имодействия, от относительной величины которых зависит возможностьи используемая система классификации электронных состояний атома:

1. кулоновское взаимодействие электронов с ядром Uen;

2. электростатическое взаимодействие электронов между собой Uee;

Page 58: КВАНТОВАЯ МЕХАНИКА МОЛЕКУЛ · PDF file2 ББК В22.31 УДК 544.11+539.183 isbn 978-5-94356-719-3 Стась Д.В., Плюснин В.Ф. Квантовая

58 ГЛАВА 3. КЛАССИФИКАЦИЯ ТЕРМОВ АТОМА

3. спин-орбитальное взаимодействие Uls;

4. взаимодействие электронов с внешним полем, при его наличии, UH .

Для нормального атома самым сильным всегда является взаимодейст-вие Uen, оно собственно и определяет существование атома или иона —положительно заряженного ядра, удерживающего в своем поле отри-цательно заряженные электроны. Сегодня с появлением сверхмощныхфемтосекундных лазеров появилась возможность создавать внешнее дляатома поле, соизмеримое с внутриатомным, но это уже приводит к со-зданию особого состояния вещества, которого мы касаться не будем.Взаимодействия Uen (и частично Uee) отвечают за самое грубое разде-ление состояний электронов по энергиям и делают возможным введениеэлектронной конфигурации атома.

Более тонкие взаимодействия Uee и Uls определяют используемыйтип классификации электронного состояния атома. В предельном слу-чае Uee Uls взаимодействие орбитальных и спиновых степеней свободыэлектронов мало по сравнению с электростатическим взаимодействиемэлектронов между собой, поэтому естественным является приближениесвободного спина, в котором спин-орбитальное взаимодействие можнона первом этапе опустить. В результате получается, что взаимодействиеэлектронов приводит к коллективным состояниям электронной оболочкис определенными значениями полного орбитального момента электро-нов L, при этом спины электронов оказываются явно не задействованыи складываются независимо в полный спин S. Эффект спинов на энер-гию электронных состояний здесь проявляется косвенно, через принципПаули. Если требуется, затем можно как следующее по малости учестьи спин-орбитальное взаимодействие, связывающее теперь полный орби-тальный момент и полный спин электронов взаимодействием вида A~L~Sи дополнительно расщепляющее коллективные электронные состоянияпо величине полного момента J. Оставшееся вырождение по проекцииполного момента электронов Jz может быть снято наложением слабоговнешнего магнитного поля.

Описанный предельный случай называется случаем LS-связи, илислучаем Рассел—Саундерса. Принятая для него иерархия взаимодейст-вий (последовательность их учета в порядке ослабления) имеет порядокUen Uee Uls( UH), который схематически изображен на рис. 3.1.Любопытно отметить, что самое сильное (после Uen) расщепление поэнергии определяется спином, который формально учитывается тольков следующих порядках теории возмущений — столь велики оказывают-

Page 59: КВАНТОВАЯ МЕХАНИКА МОЛЕКУЛ · PDF file2 ББК В22.31 УДК 544.11+539.183 isbn 978-5-94356-719-3 Стась Д.В., Плюснин В.Ф. Квантовая

3.1 ИЕРАРХИЯ ВЗАИМОДЕЙСТВИЙ В АТОМЕ 59

ýþÿÿýþ

ýþ ÿ

ÿ

ÿ

ÿ

Рис. 3.1. Иерархия взаимодействий в многоэлектронном атоме в предельном

случае LS-связи.

ся эффекты US, связанные с симметризацией коллективного состояниямногоэлектронной системы. Опыт показывает, что приближение LS-связихорошо работает для не слишком высоко возбужденных состояний неслишком тяжелых атомов (примерно до железа). Термы в этом случаеобозначаются 2S+1LJ.

Другой предельный случай реализуется при обращении порядка спин-орбитального и межэлектронного взаимодействий Uls и Uee: Uls Uee. Этаситуация характерна для тяжелых атомов и внутренних (слабо экрани-рованных) электронов, поскольку константа спин-орбитального взаимо-действия быстро (квадратично) растет с ростом заряда ядра, а также длявысоковозбужденных, так называемых Ридберговских, состояний атомов,когда один из его электронов оказывается в состоянии с главным кванто-вым числом n порядка нескольких десятков. При этом его взаимодейст-вие с остальными электронами ослабевает настолько, что опускается поэнергии ниже спин-орбитального. В этой ситуации говорят о предель-ном случае j j-связи: более разумным теперь будет учесть сначала болеесильное спин-орбитальное взаимодействие для каждого из рассматри-ваемых электронов, сложив его орбитальный момент l и спин s = 1

2 вполный момент одного электрона j = l ± 1

2, и затем уже учесть болееслабое межэлектронное взаимодействие, сложив полные моменты j ин-дивидуальных электронов в возможные полные моменты всей электрон-ной оболочки J. Схематически иерархия взаимодействий в этом случаепоказана на рис. 3.2.

Термы атома в приближении j j-связи обозначают ( j1, j2, . . ., jn)J, гдев скобках перечислены полные моменты отдельных электронов, нижнийиндекс J указывает полный момент многоэлектронной системы. Как в

Page 60: КВАНТОВАЯ МЕХАНИКА МОЛЕКУЛ · PDF file2 ББК В22.31 УДК 544.11+539.183 isbn 978-5-94356-719-3 Стась Д.В., Плюснин В.Ф. Квантовая

60 ГЛАВА 3. КЛАССИФИКАЦИЯ ТЕРМОВ АТОМА

! " # Рис. 3.2. Иерархия взаимодействий в многоэлектронном атоме в предельном

случае j j-связи.

случае Рассел—Саундерса, так и в случае j j-связи самым внешним ин-дексом классификации электронных состояний многоэлектронного атомабудет полный момент J, отражающий сферическую симметрию изоли-рованного атома как «черного ящика» для внешнего мира. Наложениемслабого внешнего магнитного поля симметрию «черного ящика» можнопонизить до аксиальной, что и снимает вырождение по проекции полногомомента Jz, расщепляя термы на 2J +1 компонент.

Наконец, в рамках предела LS-связи можно рассмотреть предельныеслучаи Uls UH и UH Uls при наложении внешнего магнитного поля.О первой ситуации говорят как об эффекте Зеемана, когда внешнее полевзаимодействует с полным моментом атома J и расщепляет состояния повеличине Jz на 2J +1 компонент. Второй случай соответствует эффектуПашена—Бака: говорят, что внешнее поле в этом случае настолько силь-но, что оно разрывает связь между орбитальным моментом и спином.В этом случае расщепление идет по величине (2Lz + Sz). В большинст-ве случаев для полей до нескольких Тесла работает предел Зеемана, нопараметры спин-орбитального взаимодействия для атомов различаютсядостаточно сильно, и для легких атомов, например, H и Li, в полях око-ло 3 Тл наблюдаются спектры, характерные для режима Пашена—Бака.

3.2 Атом в приближении LS-связи

Давайте начнем разбор системы классификация для случая LS-связис простейшего примера атома водорода. Электронная конфигурация ос-новного состояния атома водорода 1s1 порождает всего один терм 2S1/2 с

Page 61: КВАНТОВАЯ МЕХАНИКА МОЛЕКУЛ · PDF file2 ББК В22.31 УДК 544.11+539.183 isbn 978-5-94356-719-3 Стась Д.В., Плюснин В.Ф. Квантовая

3.2 АТОМ В ПРИБЛИЖЕНИИ LS-СВЯЗИ 61

орбитальным моментом L = 0 (S терм), спином S = 12 (указываемая в пра-

вом верхнем углу мультиплетность 2S+1 = 2) и полным моментом J = 12.

Мультиплетность и J в символе термина указываются цифрами, а дляобозначения орбитального момента используются те же буквы, что и дляодноэлектронных орбитальных состояний, только прописные: S (L = 0),P (L = 1), D (L = 2), F (L = 3) и далее по латинскому алфавиту.

При оптическом возбуждении атом водорода перейдет в однократновозбужденную электронную конфигурацию 2p1, которая порождает дватерма: 2P1/2 и 2P3/2. При наличии нескольких термов их порядок по энер-гии определяется тремя эмпирически установленными правилами Гунда:

1. При прочих равных условиях наименьшую энергию имеет терм смаксимальным полным спином S. В качестве ключа можно ука-зать, что для многоэлектронной системы максимальный суммарныйспин соответствует максимально симметричной по спину волновойфункции, что приводит к максимально антисимметричной коорди-натной волновой функции, в которой электроны пространственноразнесены друг от друга. Это же в свою очередь минимизируетмежэлектронное отталкивание и тем самым позволяет получитьнаименьшую энергию. По мере снижения полного спина энергиятермов повышается.

2. При равных S наименьшую энергию имеет терм с максимальнымполным орбитальным моментом L. Здесь можно дать следующийключ: максимальный суммарный момент соответствует максималь-но «согласованному» движению электронов. Если максимальныйспин дает статическое разделение электронов в пространстве(локализацию электронов в разных областях), то максимальныйорбитальный момент снижает шансы электронов на динамическуювстречу, что также эффективно уменьшает межэлектронное оттал-кивание. Как и для спина, снижение полного орбитального моментавнутри группы термов с одинаковым спином приводит к постепен-ному повышению их энергии.

3. При одинаковых S и L наименьшую энергию для оболочки, запол-ненной менее чем наполовину, имеет терм с минимальным пол-ным моментом J, давая так называемый «нормальный мультип-лет». Для более чем наполовину заполненной оболочки нижним поэнергии оказывается терм с максимальным J («обращенный муль-типлет»). Объясняется это следующим образом. Константа спин-орбитального взаимодействия λ для индивидуального электрона

Page 62: КВАНТОВАЯ МЕХАНИКА МОЛЕКУЛ · PDF file2 ББК В22.31 УДК 544.11+539.183 isbn 978-5-94356-719-3 Стась Д.В., Плюснин В.Ф. Квантовая

62 ГЛАВА 3. КЛАССИФИКАЦИЯ ТЕРМОВ АТОМА

положительна, и спин-орбитальное взаимодействие с операторомVls = λ (~l ·~s) с собственными числами

Els = λ ( j( j +1)− l(l +1)− s(s+1))/2

дает для одного электрона нижнее состояния с минимальным j.Для многоэлектронной системы полный оператор спин-орбитальноговзаимодействия дается суммой одноэлектронных операторов Vi, арезультат его усреднения по состояниям с определенным L и S даетэффективный оператор вида VLS = A~L~S. При менее чем наполовинузаполненной оболочке все входящие в VLS константы λi окажутсяположительны, потому положительна и эффективная константа A,и нижним по энергии оказывается состояние с минимальным J.Для более чем наполовину заполненной оболочки последняя эф-фективно заполняется не электронами, а положительно заряжен-ными дырками, поэтому все зависящие от знака заряда носителейконстанты λ , а с ними и константа A, меняют знак. Спин-орбиталь-ное взаимодействие отсутствует для полностью заполненных обо-лочек (L = S = 0), для замкнутых оболочек (S = 0), а также для техровно наполовину заполненных оболочек, где все внешние электро-ны находятся в разных орбитальных состояниях (L = 0). Типичныевеличины константы спин-орбитального взаимодействия A состав-ляют 10−3 эВ для атомов из начала таблицы Менделеева и 1 эВ дляатома йода, что уже порядка характерных расщеплений, связанныхс взаимодействием электронов между собой.

Правила Гунда также имеют иерархию и применяются в порядке1 → 2 → 3. Строго они были установлены для основных электронныхконфигураций атомов, однако в 90% случаев работают и для невысоковозбужденных конфигураций и потому очень полезны как ориентиры.

В случае атома водорода в основной электронной конфигурации унас имеется всего один терм, 2S1/2 , а для возбужденного атома H∗ —уже два, и, согласно третьему правилу Гунда, нижним из них будеттерм 2P1/2. Основное состояние атома принято обозначать буквой X , пер-вое возбужденное — буквой A, второе возбужденное — B и так дальше полатинскому алфавиту, например, для атома водорода последовательностьуровней будет иметь вид X 2S1/2, A 2P1/2, B 2P3/2, . . .

Мы пропустили еще один терм атома водорода: конфигурация 2s1

порождает терм 2S1/2, который по энергии находится между нашими Sи P термами. Это так называемое метастабильное состояние с временем

Page 63: КВАНТОВАЯ МЕХАНИКА МОЛЕКУЛ · PDF file2 ББК В22.31 УДК 544.11+539.183 isbn 978-5-94356-719-3 Стась Д.В., Плюснин В.Ф. Квантовая

3.2 АТОМ В ПРИБЛИЖЕНИИ LS-СВЯЗИ 63

жизни порядка нескольких миллисекунд, что примерно в 108 раз большевремени жизни состояния 2P1/2. Столь большое время жизни связано сзапретом самого эффективного электрического дипольного перехода изметастабильного состояния в основное. Заметим, что формально термыосновного и метастабильного состояния оказались одинаковыми (2S1/2)хотя это конечно совершенно разные состояния. Поэтому для полнойидентификации терма атома нужно еще указывать порождающую егоэлектронную конфигурацию. Подобные «невидимые» состояния тоже по-лучают свои обозначения, но обычно не нарушают линейку A,B,C . . .

Атомы гелия и бериллия в основных электронных конфигурацияхдают термы 1S0 (1s2) и 1S0 (1s22s2), соответственно, что иллюстрируетобщее правило: электроны полностью заполненных оболочек и подоболо-чек дают «нулевые» термы (L = S = J = 0), которые можно не учитыватьпри анализе. По этой причине при классификации термов мы можемрассматривать только электроны внешних, не полностью заполненных(«валентных») оболочек.

При однократном возбуждении атома бериллия мы можем получитьэлектронную конфигурацию 1s22s12p1, которая порождает термы 1,3P.Согласно правилам Гунда, основным из них будет терм 3P, а при учетеспин-орбитального взаимодействия — терм 3P0.

Построение термов атомов в приближении LS-связи есть один из ва-риантов процедуры сложения моментов. Здесь мы переходим из базисаиндивидуальных моментов l, lz и спинов электронов s, sz сначала кбазису полных орбитального момента L,Lz и спина S,Sz, а затем кбазису полного момента J,Jz. Чтобы при таком переходе не потерятьтермы, очень полезно подсчитывать число состояний в исходном базисеиндивидуальных моментов (называемое кратностью вырождения элек-тронной конфигурации) и числа состояний в результирующих термах(кратности вырождения термов): сумма кратностей вырождения термовдолжна быть равна кратности вырождения электронной конфигурации.Для нашего примера с возбужденным атомом бериллия:

• валентная оболочка: 2s12p1;

• кратность вырождения конфигурации:

2 (один s электрон)×6 (один p электрон) = 12

при подсчете кратности вырождения конфигурации кратности вы-рождения для отдельных электронов перемножаются, посколькуони относятся к независимым степеням свободы — часто базис

Page 64: КВАНТОВАЯ МЕХАНИКА МОЛЕКУЛ · PDF file2 ББК В22.31 УДК 544.11+539.183 isbn 978-5-94356-719-3 Стась Д.В., Плюснин В.Ф. Квантовая

64 ГЛАВА 3. КЛАССИФИКАЦИЯ ТЕРМОВ АТОМА

индивидуальных моментов называют также «мультипликативным»базисом;

• получили термы:1P,кратность = 1 (по спину)×3 (по орбитальному моменту) = 3;3P,кратность = 3 (по спину)×3 (по орбитальному моменту) = 9;

• полная кратность = 3+9= 12.

При подсчете кратности вырождения терма орбитальные и спиновыекратности (соответственно 2L + 1 и 2S + 1) перемножаются как относя-щиеся к независимым степеням свободы. Если теперь еще учесть спин-орбитальное взаимодействие, получим:

• терм 1P не расщепляется, кратность = 3;

• терм 3P расщепляется на 3P0, 3P1, 3P2, в каждом из них по 2J + 1состояний, кратность = 1+3+5= 9.

3.3 Группа эквивалентных электронов

Мы рассмотрели ситуации одного электрона и двух электронов в раз-ных состояниях. Давайте теперь посмотрим, что получится для группыэквивалентных электронов — электронов с одинаковыми квантовымичислами n, l. В качестве простейшего содержательного примера здесьможно взять валентную конфигурацию p2 (например, атома углерода).

Возможные состояния электрона np индексируются двумя кванто-выми числами, проекциями его орбитального момента ml и спина ms,давая 3×2 = 6 различных комбинаций. На первый взгляд кажется, чтоэта электронная конфигурация имеет кратность (3×2)× (3×2) = 36, нопринцип Паули запрещает двум электронам находиться в одинаковомсостоянии, и число возможностей размещения электронов существенносокращается. Наша задача превращается в комбинаторную задачу о чис-ле выборок без повторения: нужно найти, сколькими способами можноразместить 2 одинаковых шара по 6 ящикам без повтора. Ответ даетсячислом сочетаний C2

6 = 5·62 = 15 — именно такова кратность вырождения

электронной конфигурации p2.Для регулярного поиска термов, которые порождает группа эквива-

лентных электронов, существует удобная процедура — построение такназываемой таблицы Рассел—Саундерса. Давайте нарисуем таблицу,

Page 65: КВАНТОВАЯ МЕХАНИКА МОЛЕКУЛ · PDF file2 ББК В22.31 УДК 544.11+539.183 isbn 978-5-94356-719-3 Стась Д.В., Плюснин В.Ф. Квантовая

3.3 ГРУППА ЭКВИВАЛЕНТНЫХ ЭЛЕКТРОНОВ 65

$%$& ' ( )'* + ','- +

' ',(, ',(-'-(, '-(-

( ',)', ',(-'-(,(,(- '-)'-

)' )',(, )',(-)'-(, )'-(-

)* + )',)'- +

Рис. 3.3. Таблица Рассел—Саундерса для электронной конфигурации p2.

столбцы который нумеруются всеми возможными значениями проекцииполного спина MS по убыванию (для конфигурации p2: 1,0,−1), а стро-ки — всеми возможными значениями проекции полного орбитальногомомента ML по убыванию (2,1,0,−1,−2). В клетки таблицы, начиная слевого верхнего угла, вписываем возможные электронные конфигурациис соответствующей парой значений ML/MS в виде «1+0−», что означа-ет «один электрон находится в состоянии с орбитальной проекцией +1и спиновой проекцией +1

2 (это позиция 1+), а другой — в состояниис ml = 0, ms = −1

2 (позиция 0−)». Обратите внимание, что не «первый»и «второй», а «один» и «другой»: поскольку наши электроны неразли-чимы, состояние 0−1+ формально будет тем же самым состоянием, чтои 1+0−. На самом деле из этих двух состояний нам нужно будет по-строить одно антисимметризованное, но этим мы займемся чуть позже.Сейчас нам важно только количество разных способов, которым мож-но набрать состояния с определенными значениями ML/MS. Состояниятипа 0+0− запрещены принципом Паули. Заполненная таблица Рассел—Саундерса для конфигурации p2 приведена рис. 3.3.

Теперь используем построенную таблицу для нахождения термов,порождаемых нашей электронной конфигурацией, при этом процедуравполне аналогична стандартной процедуре сложения моментов и осно-вана на анализе числа состояний с определенными значениями полныхпроекций орбитального момента и спина. Двигаемся по таблице сверху

Page 66: КВАНТОВАЯ МЕХАНИКА МОЛЕКУЛ · PDF file2 ББК В22.31 УДК 544.11+539.183 isbn 978-5-94356-719-3 Стась Д.В., Плюснин В.Ф. Квантовая

66 ГЛАВА 3. КЛАССИФИКАЦИЯ ТЕРМОВ АТОМА

./ 01 / 02./3/ 024 2/ 01 4 2/ 024 1/ 01 4 1/ 002/ 0201 4 2024 14 1 4 201 / 014024 201 4 2024 101 4 10.01 02.3

5 6 / 0405 7

./ 01 / 02./3/ 024 2/ 01 4 2/ 024 1/ 01 4 1/ 002/ 0201 4 2024 14 1 4 201 / 014024 201 4 2024 101 4 10.01 02.3

5 6 / 0405 7

./ 01 / 02./3/ 024 2/ 01 4 2/ 024 1/ 01 4 1/ 002/ 0201 4 2024 14 1 4 201 / 014024 201 4 2024 101 4 10.01 02.3

5 6 / 0405 7

./ 01 / 02./3/ 024 2/ 01 4 2/ 024 1/ 01 4 1/ 002/ 0201 4 2024 14 1 4 201 / 014024 201 4 2024 101 4 10.01 02.3

5 6 / 0405 7

Рис. 3.4. Две последовательные итерации поиска термов из таблицы Рассел—Саундерса для электронной конфигурации p2.

вниз и слева направо и находим первое из вписанных состояний: 1+1− вклетке с ML = 2 и MS = 0, самое «старшее» состояние в рассматриваемойсистеме. Поскольку мы имеем состояние с (максимальной) орбитальнойпроекцией ML = 2, мы должны иметь и момент L = 2, а значит и всевозможные для него проекции от ML = 2 до ML = −2 и (в нашем случае)с MS = 0. Этот набор из пяти состояний представляет собой 1D терм.Вычеркиваем из каждой соответствующей этому терму клетки таблицыпо одному состоянию и получаем таблицу, показанную на рис. 3.4 сле-ва. Какие состояния вычеркивать сейчас неважно, мы просто считаем ихколичества в каждой клетке.

Снова идем по таблице сверху вниз и слева направо и находим самоестаршее невычеркнутое состояние: 1+0+ в клетке с ML = 1 и MS = 1. Этосамое старшее (по проекции спина и орбитального момента) состояниетерма с S = 1 и L = 1, то есть терма 3P, к которому принадлежат поодному состоянию с MS = −1,0,1×ML =−1,0,1. Опять вычеркиваемпо одному состоянию из соответствующих клеток таблицы и получаемтаблицу, показанную рис. 3.4 справа. В таблице теперь осталось всегоодно состояние в клетке с ML = 0 и MS = 0, отвечающее терму 1S. Такимобразом, наша конфигурация порождает термы 1D (1×5= 5 состояний),3P (3×3 = 9 состояний) и 1S (1×1 = 1 состояние), которые исчерпываютвсе 15 состояний конфигурации p2. По правилам Гунда основным будеттерм 3P, а с учетом возможного ненулевого спин-орбитального взаимо-действия — терм 3P0.

Page 67: КВАНТОВАЯ МЕХАНИКА МОЛЕКУЛ · PDF file2 ББК В22.31 УДК 544.11+539.183 isbn 978-5-94356-719-3 Стась Д.В., Плюснин В.Ф. Квантовая

3.4 НЕСКОЛЬКО ГРУПП ЭКВИВАЛЕНТНЫХ ЭЛЕКТРОНОВ 67

3.4 Несколько групп эквивалентных

электронов

Давайте теперь найдем термы гипотетической конфигурации n1p2n2p3.Пользуясь свободой в группировке складываемых моментов, эту задачуудобно решать в два этапа: сначала независимо обработать две груп-пы эквивалентных электронов p2 и p3, найдя для них возможные пол-ные орбитальные моменты и спины с использованием таблицы, а затемсложить получившиеся моменты и спины между собой уже без всякихограничений, как абстрактные моменты. Как мы только что выяснили,конфигурация p2 порождает термы 1D, 3P и 1S. Для конфигурации p3 за-дача решается полностью аналогично и дает термы 2D, 2P и 4S, в которыхсодержится 2×5+2×6+4×1= 20=C3

6 состояний, исчерпывающих крат-ность вырождения конфигурации p3. Отметим для дальнейшего, что припостроении таблицы Рассел—Саундерса в действительности нужен толь-ко левый верхний угол таблицы, покрывающий положительные (и еслиесть нулевые) возможные значения ML и MS, поскольку ячейки с отри-цательными проекциями ничего нового не дают. Для конфигурации p3

возможны значения ML = 3,2,1,0,−1,−2,−3 и MS = 32, 1

2,−12,−3

2, новместо построения таблицы размером 7× 4 можно обойтись укорочен-ным вариантом размером 4×2 для ML = 3,2,1,0 и MS = 3

2, 12, что

существенно упрощает работу.Теперь складываем полученные наборы 1D, 3P, 1S⊕2D, 2P, 4S:

• 1D⊕ 2D: по спинам S: ~0+~12 =~1

2; по моментам L: ~2+~2 =~4,~3,~2,~1,~0;получаем термы 2S+1L: 2G, 2F, 2D, 2P, 2S;

• 1D⊕ 2P = 2F, 2D, 2P;

• 1D⊕ 4S = 4D;

• 3P⊕ 2D = 4F, 4D, 4P, 2F, 2D, 2P;

• 3P⊕ 2P = 4,2D, 4,2P, 4,2S;

• 3P⊕ 4S = 6,4,2P;

• 1S⊕2D, 2P, 4S,= 2D, 2P, 4S.

Можно заметить, что при таком сложении формально одинаковыетермы могут получаться больше чем по одному разу, например, терм 2P

Page 68: КВАНТОВАЯ МЕХАНИКА МОЛЕКУЛ · PDF file2 ББК В22.31 УДК 544.11+539.183 isbn 978-5-94356-719-3 Стась Д.В., Плюснин В.Ф. Квантовая

68 ГЛАВА 3. КЛАССИФИКАЦИЯ ТЕРМОВ АТОМА

получился шестью разными способами. Такая ситуация нередко встре-чается в задачах о нахождении термов, и причина ее заключается в воз-можной «потере» степеней свободы при сложении более двух моментов,когда выносимых в обозначение терма квантовых чисел уже не хватаетдля его однозначной идентификации.

В качестве примера давайте рассмотрим простейший содержательныйслучай, сложение двух и трех спинов 1

2. Сложение двух спинов 12 дает

полные моменты 1,0, в исходном базисе индивидуальных моментовбыло 2× 2 = 4 состояния, в конечном базисе полных моментов будет3+1 = 4 состояний, все сходится. При сложении трех спинов 1

2 получимполные моменты 3

2, 12, в исходном базисе 2×2×2 = 8, а в конечном —

только 4+2 = 6 состояний, два состояния потерялось. Причина конечнов том, что складывать моменты, получая полный момент от суммы домодуля разности, мы умеем только для двух спинов. Чтобы сложить тримомента, нужно сначала сложить два, а потом к результату добавить

третий: ~12 +~1

2 +~12 =

(~12 +~1

2

)

+~12 =

~1;~0

+~12 =

~32 ,~12;~12

, и мы два раза

получаем полный момент 12.

Формально задача о сложении двух спинов состоит в переходе измультипликативного базиса |s1,ms1〉|s2ms2〉, параметризуемого четырьмячислами s1, s2,ms1,ms2, в базис полного момента |s1, s2,S,MS〉, в которомпроекции отдельных моментов заменены на полный момент и его проек-цию, но при этом в задаче все равно осталось четыре степени свободы.При сложении же трех моментов нужно из базиса |s1,ms1〉|s2ms2〉|s3ms3〉 сшестью степенями свободы перейти к базису |s1, s2, s3,S,MS〉, параметри-зуемому всего пятью числами. При таком переходе одна степень свободыоказывается потерянной, точнее, скрытой — в нашем примере про триспина 1

2 в качестве нее можно взять, например, результат сложения пер-вой пары спинов (1,0), к которому затем прибавляется третий спинс получением полного момента 1

2 . При добавлении четвертого спина внашем примере появится вторая скрытая степень свободы, и так далее.Эта проблема хорошо известна в теории углового момента.

Отметим также, что при рассмотрении электронной конфигурации p3

мы не получили терма с формально возможным максимальным момен-том L = 3. Это одно из следствий принципа Паули: поскольку по спинудля электрона имеется всего две возможности, ms = 1

2 и ms = −12, при

сложении орбитальных проекций электронов только две из них можновзять максимальными. Поэтому для трех эквивалентных p электроновмаксимальная возможная величина ML = 1+1+0= 2, откуда и Lmax = 2.

Page 69: КВАНТОВАЯ МЕХАНИКА МОЛЕКУЛ · PDF file2 ББК В22.31 УДК 544.11+539.183 isbn 978-5-94356-719-3 Стась Д.В., Плюснин В.Ф. Квантовая

3.5 БОЛЕЕ ГРОМОЗДКИЕ КОНФИГУРАЦИИ 69

3.5 Более громоздкие конфигурации

Давайте найдем термы, порождаемые электронной конфигурацией d3.Всего в ней C3

10 = 120 состояний, и чтобы не запутаться и ничего непотерять, нужно действовать очень аккуратно. Прежде всего давайтепосмотрим, какими регулярными способами можно набрать возможныезначения полной проекции ML из индивидуальных проекций:

ML = 6 : 2,2,2ML = 5 : 2,2,1ML = 4 : 2,2,0, 2,1,1ML = 3 : 2,2,−1, 2,1,0, 1,1,1ML = 2 : 2,2,−2, 2,1,−1, 2,0,0, 1,1,0ML = 1 : 2,1,−2, 2,0,−1, 1,1,−1, 1,0,0ML = 0 : 2,0,−2, 2,−1,−1, 1,1,−2, 1,0,−1, 1,0,−1.

Все комбинации построены так, что последовательные проекции слеванаправо упорядочены по невозрастанию. Регулярность в построении та-ких комбинаций в какой-то степени защищает от потери возможных ва-риантов, практически неизбежных при случайном наборе комбинаций.

Теперь для построения таблицы Рассел—Саундерса нам необходиморасставить спиновые индексы «±» у орбитальных проекций. Договорим-ся, что если для данного орбитального набора существует более чемодна комбинация спиновых индексов, то мы их не будем выписыватьвсе явно, а просто поставим в соответствующей клетке таблицы нужноечисло галочек

√. Так, например, вместо 1+0+−1−, 1+0−−1+, 1−0+−1+

будем писать 1+0+−1−,√

,√. Заполненная укороченная таблица Рассел—

Саундерса для конфигурации d3 приведена на рис. 3.5.Обратите внимание, что при движении сверху вниз по столбцу или

слева направо по строке укороченной таблицы число состояний можеттолько возрастать или оставаться постоянным — наличие состояний вы-ше или левее по таблице говорит о наличии соответствующих мультип-летов, в каждый из которых обязательно попадет и по одному состояниюиз каждой клетки ниже/правей по таблице. Это еще один защитный ме-ханизм при построении сложной таблицы: если Вы получили состоянийменьше, чем в предыдущей клетке — ищите ошибку.

Снова ищем самое старшее состояние в таблице, которое окажется вклетке с ML = 5 и MS = 1

2 и порождает терм 2H (L = 5,S = 12), содержа-

щий 22 состояния. Вычеркиваем по одному состоянию из соответствую-щих клеток и ищем следующий терм — им будет терм 2G (L = 4,S = 1

2,

Page 70: КВАНТОВАЯ МЕХАНИКА МОЛЕКУЛ · PDF file2 ББК В22.31 УДК 544.11+539.183 isbn 978-5-94356-719-3 Стась Д.В., Плюснин В.Ф. Квантовая

70 ГЛАВА 3. КЛАССИФИКАЦИЯ ТЕРМОВ АТОМА

898: ;<= ><=? @ @A @ =B=C >BD @ =B=CEB=B >B >C; =B >BEB =B=CF>B=B >BECGHGH= =B >BF>B =B=CF=BG =B >BF>CGHGH=BEBECG >B >CEB> =B >BF=B=BEBF>B =B >BF=CGHGHG >B >CF>B=BEBF>CGHGHG >BEBECE =BEBF=B>BEBF>B =BEBF=CGHGHG =BF >BF>C>B >CF=BG >BEBF>CGHGHG

Рис. 3.5. Таблица Рассел—Саундерса для электронной конфигурации d3.

18 состояний), затем получается терм 4F (L = 3,S = 32 , 28 состояний),

затем терм 2F (L = 3,S = 12, 14 состояний), затем 2D (L = 2,S = 1

2, 10состояний), затем . . . — опять 2D (10 состояний), так как при переходеот клетки ML = 3,MS = 1

2 к клетке ML = 2,MS = 12 число состояний

увеличилось сразу на два. Затем получим 4P терм (12 состояний), ипоследним — 2P терм (6 состояний). Проверяем, что полное число состо-яний в выписанных термах равно 120, т. е. мы исчерпали все состоянияисходной конфигурации и ничего не потеряли. Можно отметить, что мыснова не получили формально максимальный момент L = 6, посколькув конфигурации имеется больше двух эквивалентных электронов, и по-лучили формально повторяющиеся термы 2D — это разные состоянияодинаковой симметрии, имеющие разную энергию и разные волновыефункции. Ситуация здесь в чем-то аналогична наличию многих s состо-яний в одноэлектронном атоме. Для конфигурации lN повторяющиесятермы возникают при условии l > 1 и 2< N < 4l (d3−d7, f 3− f 11 и т. д.).

Page 71: КВАНТОВАЯ МЕХАНИКА МОЛЕКУЛ · PDF file2 ББК В22.31 УДК 544.11+539.183 isbn 978-5-94356-719-3 Стась Д.В., Плюснин В.Ф. Квантовая

3.6 БОЛЕЕ ЧЕМ НАПОЛОВИНУ ЗАПОЛНЕННЫЕ ОБОЛОЧКИ 71

Кроме того, рассмотренная конфигурация также не порождает и термас формально минимально возможным моментом L = 0 — при наложе-нии ограничений, связанных с принципом Паули, возможные значенияполного момента терма могут обрезаться не только сверху, но и снизу.

Давайте теперь обсудим вопрос о нахождении только основного со-стояния атома или иона. Возьмем, например, ион Nd3+ с электроннойконфигурацией f 3 — активный ион в лазере на неодимовом стекле. Вы-писывать для него таблицу Рассел—Саундерса будет занятием довольноутомительным, поскольку кратность вырождения электронной конфигу-рации f 3 C3

14 = 364. Однако если требуется найти только терм основно-го состояния, удобно просто обратить правила Гунда: из всех возмож-ных термов, порождаемых электронной конфигурацией, основным будеттерм с (последовательно) максимальным полным спином S, максималь-ным полным орбитальным моментом L и минимальным или максималь-ным полным моментом J. Обращение правил Гунда дает облегченнуюпроцедуру поиска терма основного состояния:

1. Набираем максимально возможную спиновую проекцию, а значит,и полный спин S: в нашем примере получим S = 3

2;

2. Для найденного S набираем максимально возможную проекцию ор-битального момента, а значит, и сам момент L: в нашем примереполучим 3+2+1= 6, L = 6 (I терм);

3. Оболочка заполнена менее чем наполовину — собираем из найден-ных S и L минимально возможный полный момент J = L−S = 9

2.Итого терм основного состояния иона Nd3+( f 3) — 4I9/2.

3.6 Более чем наполовину заполненные

оболочки

Давайте теперь найдем термы, порождаемые конфигурацией p4. Все-го конфигурация содержит C4

6 = 15 состояний, и построенная для нее повсем правилам таблица Рассел—Саундерса показана на рис. 3.6. Частьклеток в таблице можно вычеркнуть сразу: поскольку у нас имеется 4электрона, но только две возможности выбрать спиновые индексы, мак-симальная разрешенная проекция ML будет на 2 меньше формально мак-симальной проекции ML = 4, и верхние две строки в таблице будут заве-домо пустыми. Кроме того, у нас есть только три возможности выбрать

Page 72: КВАНТОВАЯ МЕХАНИКА МОЛЕКУЛ · PDF file2 ББК В22.31 УДК 544.11+539.183 isbn 978-5-94356-719-3 Стась Д.В., Плюснин В.Ф. Квантовая

72 ГЛАВА 3. КЛАССИФИКАЦИЯ ТЕРМОВ АТОМА

IJIK L M NO P P PQ P P P

L P P MRMSNRNS

M P MRMSNRTMR MRMSNRTMSMRMSNSTMR

N P MRNRNSTMR MRMSTMRTMSMRNRNSTMSMSNRNSTMR

Рис. 3.6. Таблица Рассел—Саундерса для электронной конфигурации p4.

орбитальные индексы (для p электрона ml = −1,0,1), так что хотя быодна из спиновых проекций обязательно будет другого знака, поэтому имаксимальная разрешенная спиновая проекция оказывается на единицуменьше формального максимума MS = 2, и крайний левый столбец так-же оказывается пустым. Эти простые рассуждения позволяют нам сразувычеркнуть 9 клеток из 15 и таким образом решить две трети задачи.Остальные клетки таблицы заполняются по обычным правилам и даюттермы 1D, 3P и 1S, исчерпывающие все 15 состояний, имевшихся в исход-ной конфигурации. Можно заметить, что термы получились те же самые,что и для конфигурации p2, а в каждой выписанной нами конфигурации-четверке из таблицы обязательно присутствует пара «спаренных» элек-тронов, имеющих одинаковые ml и разные ms, например, 1+1−0+−1+.Этот простой пример иллюстрирует очень полезный общий вывод: дляболее чем наполовину заполненных конфигураций удобнее раскладыватьпо ячейкам не электроны, а дырки, при этом получаются те же самыетермы. Единственное отличие — в знаке константы спин-орбитальноговзаимодействия, которая для положительно заряженных дырок оказыва-ется меньше нуля, и при учете спин-орбитального взаимодействия самымнижним по энергии оказывается терм с максимальным полным моментомэлектронной оболочки J. Так, в нашем случае конфигурации p4 основнымокажется терм 3P2.

Page 73: КВАНТОВАЯ МЕХАНИКА МОЛЕКУЛ · PDF file2 ББК В22.31 УДК 544.11+539.183 isbn 978-5-94356-719-3 Стась Д.В., Плюснин В.Ф. Квантовая

3.7 АТОМ В ПРИБЛИЖЕНИИ JJ-СВЯЗИ 73

3.7 Атом в приближении j j-связи

Теперь давайте разберем систему классификации термов в противо-положном предельном случае иерархии взаимодействий в атоме, ког-да взаимодействие орбитального момента электрона и его спина можносчитать большим по сравнению с межэлектронным взаимодействием. Какуже упоминалось ранее, это приводит к необходимости ввести сначалаполные моменты индивидуальных электронов ~ji =~li +~si, а затем сложитьих в полный момент многоэлектронной системы J. Термы в этом случаеобозначаются ( j1, j2, . . ., jn)J , а само приближение называется приближе-нием j j-связи.

Мы опять начнем с самого простого случая и постепенно доведемзадачу до содержательной. Для электронной конфигурации s1 имеем:s = 1

2 , l = 0, j = 12, J = 1

2, единственный терм(

12

)

1/2. Для конфигурации p1:

s = 12, l = 1, j = 1

2, 12, J = j = 1

2, 12, получаем термы

(12

)

1/2 и(3

2

)

3/2,

В первом из них содержится два состояния (Jz = 12,−1

2), во втором —четыре, всего получается шесть состояний, исчерпывающих кратностьвырождения электронной конфигурации p1. Можно ли указать, как издвух термов окажется ниже по энергии? В этом простом случае этосделать несложно: для одного электрона константа спин-орбитальноговзаимодействия λ всегда положительна, поэтому нижним по энергииоказывается состояние с минимальным полным моментом, т. е. основнымдля электронной конфигурации p1 в случае j j-связи будет терм

(12

)

1/2.

Теперь рассмотрим конфигурацию s1p1. Прежде всего находим воз-можные значения индивидуальных полных моментов двух электронов:для фрагментов конфигурации s1 и p1 имеем соответственно j1 = 1

2 иj2 = 1

2, 12. Затем комбинируем j1, j2 в возможные суммарные моменты

и получаем следующие термы (первым в записи всегда идет s электрон):(1

2, 12

)

0 (содержит 1 состояние),(1

2, 12

)

1 (3 состояния),(1

2, 32

)

1 (3 состо-

яния) и(

12 , 3

2

)

2 (5 состояний). Всего получаем 12 состояний, что равнократности вырождения исходной электронной конфигурации s1p1.

Можно ли среди полученных термов указать основной? Поскольку до-минирующим взаимодействием в этом предельном случае является спин-орбитальное, терм с минимальной энергией должен прежде всего бытьнабран из состояний с минимальными индивидуальными полными мо-ментами. В нашем случае пара термов

(12, 1

2

)

0,(1

2, 12

)

1, будет заведомониже по энергии, чем пара

(12, 3

2

)

1,(1

2 , 32

)

2. Чтобы упорядочить термыв парах по величине полного момента J, можно попытаться построить

Page 74: КВАНТОВАЯ МЕХАНИКА МОЛЕКУЛ · PDF file2 ББК В22.31 УДК 544.11+539.183 isbn 978-5-94356-719-3 Стась Д.В., Плюснин В.Ф. Квантовая

74 ГЛАВА 3. КЛАССИФИКАЦИЯ ТЕРМОВ АТОМА

корреляцию с термами противоположного предельного случая, прибли-жения LS-связи, для которого имеются правила Гунда относительногорасположения термов по энергии.

Конфигурация s1p1 в пределе LS-связи порождает термы 1P1 и 3P0,1,2,которые упорядочатся по энергии следующим образом (по возрастанию):3P0, 3P1, 3P2, и затем через достаточно большой промежуток 1P1. Фор-мально, чтобы перейти из одного предельного случая в другой, нам нуж-но увеличить константу спин-орбитального взаимодействия λ . Так какспин-орбитальное взаимодействие является для рассматриваемой сис-темы внутренним, его изменение не влияет на классификацию состо-яний по внешнему индексу J, которая просто отражает сферическуюсимметрию электронной системы атома как «черного ящика». Поэтомупри (мысленном) изменении λ состояния предельного случая LS-связис определенной величиной J перейдут в состояния предела j j-связи стеми же J. Поскольку в наших наборах имеется всего по одному термус J = 0 и J = 2, для них ситуация будет полностью определена: термосновного состояния случая LS-связи 3P0 перейдет в терм

(12, 1

2

)

0, атерм 3P2 — в терм

(12 , 3

2

)

2. Чтобы построить соответствие между терма-ми с J = 1, вспомним, что при при изменении параметров взаимодейст-вия термы системы с одной симметрией, как правило, не пересекаются1 ,поэтому при увеличении λ нижнее состояние с J = 1 случая LS-связи(терм 3P1) перейдет в нижнее состояние с J = 1 предела j j-связи — терм(1

2, 12

)

1. Разумно также предположить, что самый нижний по энергиитерм в случае LS-связи, терм 3P0, останется и самым нижним в пределеj j-связи, и искомым термом основного состояния в этом случае будеттерм

(12, 1

2

)

0, хотя для данной системы предсказать порядок термов впарах

(12, 1

2

)

0,1 и(1

2, 32

)

1,2 только из качественных соображений нельзя.Обсуждаемая корреляционная диаграмма качественно будет иметь вид,показанный на рис. 3.7.

Реальный расчет таких диаграмм требует весьма больших усилий ивыполняется только компьютерными методами квантовой химии, однакоинтуитивное их построение, как мы сейчас сделали, является разум-ным приближением для качественного анализа задачи. Для проверки жеправильности таких построений или расчетов все равно приходится об-ращаться к экспериментальным данным.

1Л. Д. Ландау, Е. М. Лифшиц, Теоретическая физика, Т.3: Квантовая механика,нерелятивистская теория, М. Физматлит, 2004, §79.

Page 75: КВАНТОВАЯ МЕХАНИКА МОЛЕКУЛ · PDF file2 ББК В22.31 УДК 544.11+539.183 isbn 978-5-94356-719-3 Стась Д.В., Плюснин В.Ф. Квантовая

3.8 ЭКВИВАЛЕНТНЫЕ ЭЛЕКТРОНЫ В ПРЕДЕЛЕ JJ-СВЯЗИ 75

U

VW XVW YVW Z

YW Y

[ \]^ _\]^`X[ \]^ _\]^`Y

[ \]^ _a]^`Z[ \]^ _a]^`Y

Рис. 3.7. Корреляционная диаграмма перехода LS → j j для электронной конфи-гурации s1p1.

3.8 Эквивалентные электроны в пределе

j j-связи

Давайте теперь рассмотрим термы, порождаемые в пределе j j-связиэлектронной конфигурацией p2. Кратность нашей конфигурации рав-на 15, столько же состояний мы должны получить и в найденных термах.В рассматриваемом случае j1,2 = 1

2, 32, и возможны комбинации

(12 , 1

2

)

J,(12, 3

2

)

J ,(3

2, 32

)

J. Как и при построении таблицы Рассел—Саундерса дляэквивалентных электронов, мы учитаем только разные комбинации элек-тронов (поэтому не рассматривается возможная комбинация

(32, 1

2

)

J) иупорядочиваем их регулярным образом, в данном случае по неубываниюиндивидуальных полных моментов. Давайте подсчитаем, сколько всегосостояний содержится в предполагаемых термах:

(12, 1

2

)

J, J = 0,1, все-го 1+3 = 4 состояния;

(12, 3

2

)

J , J = 1,2, 3+5 = 8 состояний;(3

2 , 32

)

J,J = 0,1,2,3, 1+3+5+7= 16 состояний. Итого у нас получилось 28состояний, вместо требуемых 15. Причина конечно заключается в экви-валентности электронов, в результате чего часть состояний оказываетсязапрещенными принципом Паули.

Давайте разберемся, какие электроны считать эквивалентными в пре-деле j j-связи. В случае LS-связи эквивалентными мы называли электро-ны с одинаковым n (которое мы опускали), s (который всегда равен 1

2)

Page 76: КВАНТОВАЯ МЕХАНИКА МОЛЕКУЛ · PDF file2 ББК В22.31 УДК 544.11+539.183 isbn 978-5-94356-719-3 Стась Д.В., Плюснин В.Ф. Квантовая

76 ГЛАВА 3. КЛАССИФИКАЦИЯ ТЕРМОВ АТОМА

b cdc efg h efg

i

Рис. 3.8. Нахождение термов, порождаемых в пределе j j-связи конфигурациейиндивидуальных моментов

( 12, 1

2

).

и l. Различали мы их по орбитальным/спиновым проекциям ml/ms. Так,для p электрона возможны 6 различных комбинаций ml,ms, и зада-ча о числе состояний для k эквивалентных p электронов сводилась кзадаче о размещении k шаров в 6 ящиках без повторения. Каждый«ящик» здесь представляет из себя возможную уникальную комбинациюn, s, l−одинаковые×ml,ms−различные, а принцип Паули удовлетво-рялся императивом «без повторения». В случае j j-связи состояния элек-тронов описываются квантовыми числами n, s, l (одинаковыми внутригруппы) плюс парой чисел j, jz, заменяющих здесь проекции ml,ms.Теперь эквивалентными в том же смысле нужно считать электроны содинаковыми индексами n, s, l, j, при этом различаться они будут бегу-щей проекцией jz. На состояния электронов с разными j принцип Паулиуже никаких ограничений не накладывает — требуемое различие хотябы в одном квантовом числе обеспечивается разными j.

Теперь последовательно рассмотрим все три наши одноэлектронныекомбинации. У конфигурации

(12, 1

2

)имеем два электрона с одинаковым

моментами j1,2, т. е. эквивалентные. Для нахождения порождаемых этойконфигурацией термов будем следовать процедуре, сходной с построени-ем таблицы Рассел—Саундерса. Поскольку электроны имеют j = 1

2 и двавозможных значения проекции jz = 1

2,−12, нам нужно подготовить два

ящика, в которые мы будем по одному раскладывать шары-электроны.Положить два шара в два ящика без повторов можно C2

2 = 1 способом,поэтому готовим один комплект ящиков, нумеруем их возможными про-екциями jz и раскладываем шары, как показано на рис. 3.8.

Максимальная суммарная проекция Jz в данном случае оказалосьравной 0. Таким образом, мы имеем всего один мультиплет с полныммоментом J = 0, который и исчерпывает наше единственное состояние.Получаем, что конфигурация

(12 , 1

2

)порождает единственный терм

(12 , 1

2

)

0,который содержит всего одно состояние.

Page 77: КВАНТОВАЯ МЕХАНИКА МОЛЕКУЛ · PDF file2 ББК В22.31 УДК 544.11+539.183 isbn 978-5-94356-719-3 Стась Д.В., Плюснин В.Ф. Квантовая

3.8 ЭКВИВАЛЕНТНЫЕ ЭЛЕКТРОНЫ В ПРЕДЕЛЕ JJ-СВЯЗИ 77

jklk mno pno qpno qmnoo p r r

qp qo

Рис. 3.9. Нахождение термов предела j j-связи для конфигурации( 3

2 , 32

).

Переходим к конфигурации(

12 , 3

2

): поскольку мы теперь имеем два

электрона с разными j1,2, на сложение моментов нет никаких ограниче-ний, и допустимы любые в принципе возможные комбинации, в данномслучае J = 1,2. Таким образом, конфигурация

(12, 3

2

)порождает термы

(12, 3

2

)

1,(

12, 3

2

)

2, в которых содержится 3+5= 8 состояний.

В последней конфигурации(3

2 , 32

)мы снова имеем два эквивалентных

электрона с j1,2 = 32, для которых возможны четыре разных проекции jz.

Таким образом, нам нужно приготовить четыре ящика и разложить вних без повторения два шара. Сделать это можно C2

4 = 6 способами,поэтому готовим 6 комплектов по четыре ящика, регулярным образомнумеруем ящики, например, по убыванию проекции jz, и регулярным жеобразом, например, слева направо, раскладываем в них шары-электроны,как показано на рис. 3.9. Рядом к каждым комплектом ящиков пишемдаваемую им проекцию полного момента всей системы Jz. Затем идемсверху вниз и ищем среди полученных значений Jz максимальное — внашем случае им оказывается Jz = 2. Это значит, что в системе имеетсямультиплет с J = 2, в котором содержится по одному состоянию со всемивозможными Jz от −2 до 2. Вычеркиваем по одному такому состоянию

Page 78: КВАНТОВАЯ МЕХАНИКА МОЛЕКУЛ · PDF file2 ББК В22.31 УДК 544.11+539.183 isbn 978-5-94356-719-3 Стась Д.В., Плюснин В.Ф. Квантовая

78 ГЛАВА 3. КЛАССИФИКАЦИЯ ТЕРМОВ АТОМА

stu

vw

xy

xz |

~ |

~

|

¡¢

£

Рис. 3.10. Корреляционная диаграмма для электронной конфигурации p2.

из левого столбца таблицы и снова ищем максимальную из оставшихсяпроекцию Jz. Ей оказывается Jz = 0, соответствующая мультиплету J = 0с единственным состоянием Jz = 0, последним из оставшихся в нашейтаблице. Таким образом, конфигурация

(32, 3

2

)порождает термы

(32, 3

2

)

2и(3

2, 32

)

0, в которых содержится 5+1= 6 состояний.

Итого электронная конфигурация p2 в предельном случае j j-связи по-рождает термы

(12, 1

2

)

0,(1

2 , 32

)

1,(1

2 , 32

)

2,(3

2 , 32

)

0,(3

2 , 32

)

2, которые и исчер-

пывают все 15 возможных состояний. Основным из них будет терм(

12 , 1

2

)

0с минимальной конфигурацией индивидуальных моментов. На рис. 3.10опять качественно показана корреляция термов пределов LS и j j-связидля этого случая. Термы левой и правой частей диаграммы могут со-ответствовать, например, атомам углерода и свинца. Таким же образомможно рассмотреть термы, порождаемые любым числом эквивалентныхэлектронов. В таблице ниже перечислены возможные значения полно-го момента J для группы lN

j из N электронов с одинаковыми n, l, jдля j = 1

2, 32, 5

2, 72, что достаточно для рассмотрения электронов с l ≤ 3.

Отметим, что для группы с самым высоким моментом l47/2 здесь также

Page 79: КВАНТОВАЯ МЕХАНИКА МОЛЕКУЛ · PDF file2 ББК В22.31 УДК 544.11+539.183 isbn 978-5-94356-719-3 Стась Д.В., Плюснин В.Ф. Квантовая

3.9 РАЗБИЕНИЕ НА ЭКВИВАЛЕНТНЫЕ ГРУППЫ 79

появляются повторяющиеся термы с J = 2 и J = 4.

lNj возможные J

l1/212

l21/2 0

l3/2 и l33/2

32

l23/2 0,2

l43/2 0

l5/2 и l53/2

52

l25/2 и l4

3/2 0,2,4

l35/2

32, 5

2, 92

l65/2 0

l7/2 и l77/2

72

l27/2 и l6

7/2 0,2,4,6

l37/2 и l5

7/232, 5

2, 72, 9

2 , 112 , 15

2

l47/2 0,2×2,2×4,5,6,8

l87/2 0

3.9 Разбиение на группы эквивалентных

электронов

Давайте теперь найдем термы конфигурации p3 в случае j j-связи.Всего в задаче имеется C3

6 = 20 состояний, возможны следующие конфи-гурации:

(12, 1

2 , 12

),(1

2, 12, 3

2

),(1

2, 32, 3

2

),(3

2, 32 , 3

2

).

•(1

2 , 12, 1

2

): имеем три эквивалентных электрона с j = 1

2, т. е. задачу оразмещении трех шаров по двум ящикам без повторений. Так какрешения она не имеет, такие термы не существуют;

Page 80: КВАНТОВАЯ МЕХАНИКА МОЛЕКУЛ · PDF file2 ББК В22.31 УДК 544.11+539.183 isbn 978-5-94356-719-3 Стась Д.В., Плюснин В.Ф. Квантовая

80 ГЛАВА 3. КЛАССИФИКАЦИЯ ТЕРМОВ АТОМА

•(

12 , 1

2, 32

): два электрона с j = 1

2 плюс один электрон с j = 32 . Как

и в случае LS-связи, сначала обрабатываем группы эквивалентныхэлектронов, а затем складываем результирующие моменты группмежду собой. Поскольку конфигурация

(12, 1

2

)дает только терм

(12 , 1

2

)

0 с J = 0, конфигурация(

12 , 1

2, 32

)порождает единственный

терм(1

2, 12 , 3

2

)

3/2, в котором содержится 4 состояния;

•(1

2 , 32, 3

2

): имеем два эквивалентных электрона с j = 3

2, которые да-ют термы

(32, 3

2

)

0,(3

2, 32

)

2, плюс еще один электрон с j = 12, по-

лучаем возможные полные моменты 12, 3

2, 52 . Итого конфигурация

порождает термы(1

2 , 32, 3

2

)

1/2,(1

2, 32, 3

2

)

3/2,(1

2, 32, 3

2

)

5/2, содержащих2+4+6 = 12 состояний;

•(3

2 , 32, 3

2

): три эквивалентных электрона с j = 3

2, получаем задачу оразмещении трех шаров в четырех ящиках без повторения, котораяэквивалентна задаче об одном пустом ящике. Анализ показывает,что в этом случае возможно единственное значение полного момен-та J = 3

2, и конфигурация порождает единственный терм(

32, 3

2, 32

)

3/2с четырьмя состояниями.

Итого рассматриваемая электронная конфигурация p3 в случае j j-связипорождает термы

(12, 1

2 , 32

)

3/2 (4 состояния),(1

2 , 32, 3

2

)

1/2 (2 состояния),(1

2, 32, 3

2

)

3/2 (4 состояния),(1

2, 32, 3

2

)

5/2 (6 состояний) и наконец(3

2, 32, 3

2

)

3/2(4 состояния), исчерпывающие все 20 состояний исходной конфигурации,основным будет терм

(12, 1

2, 32

)

3/2.При увеличении сложности подобных задач громоздкость построе-

ния таблицы катастрофически возрастает. Так, например, при нахожде-нии термов уже для конфигурации d3 придется обрабатывать группу изтрех эквивалентных электронов с j = 5

2 — размещать три шара по шестиящикам, что можно сделать 20 способами. Как и при построении табли-цы Рассел—Саундерса, можно немного сэкономить, заметив, что наборыящиков с отрицательными полными проекциями Jz уже не дают ничегонового, поэтому построение таблицы можно оборвать после нахождениявсех неотрицательных комбинаций. И все-таки ключ к успеху здесь кро-ется в аккуратности и регулярности, с обязательной проверкой балансачисла состояний после нахождения всех термов.

Page 81: КВАНТОВАЯ МЕХАНИКА МОЛЕКУЛ · PDF file2 ББК В22.31 УДК 544.11+539.183 isbn 978-5-94356-719-3 Стась Д.В., Плюснин В.Ф. Квантовая

Глава 4

Волновые функции

многоэлектронного атома

Пока что мы занимались только классификацией состояний много-электронного атома, находя количество и тип его электронных состоя-ний, но при этом не вдаваясь в подробности их внутреннего устройства.Теперь давайте рассмотрим задачу о построении в явном виде электрон-ной волновой функции многоэлектронного атома с определенным зна-чением суммарного орбитального, спинового и полного моментов и ихпроекций в виде линейных комбинаций произведений одноэлектронныхфункций, причем мы ограничимся рассмотрением конфигураций эквива-лентных электронов в предельных случаях LS и j j-связи.

4.1 Процедура сложения моментов

Поскольку построение многоэлектронных волновых функций ведет-ся аналогично стандартной процедуре сложения моментов и основанона анализе суммарных проекций, использовании лестничных операто-ров и соотношений ортогональности, давайте прежде всего вспомним,как строятся функции полного момента. В качестве конкретного при-мера возьмем пару моментов j1 = 3

2, j2 = 1, коллективное состояниекоторых в мультипликативном базисе описывается 4×3 = 12 функциями∣∣ 3

2,m1⟩ ∣∣1,m2

⟩. Нам нужно будет построить из них 12 функций

∣∣3

2 ,1,J,M⟩

с определенными значениями суммарного момента J и его проекции M.

Page 82: КВАНТОВАЯ МЕХАНИКА МОЛЕКУЛ · PDF file2 ББК В22.31 УДК 544.11+539.183 isbn 978-5-94356-719-3 Стась Д.В., Плюснин В.Ф. Квантовая

82 ГЛАВА 4. МНОГОЭЛЕКТРОННЫЕ ВОЛНОВЫЕ ФУНКЦИИ

Прежде всего упростим нотацию, опустив для сокращения записификсированные индексы j1, j2: наши базисы будем обозначать |m1〉|m2〉и |J,M〉, соответственно. Вспомним также, что функции одного базиса схотя бы одним различающимся индексом ортогональны:

〈m1i|m1 j〉〈m2k|m2l〉 = δi jδkl, 〈Ji,Mk|J j,Ml〉 = δi jδkl. (4.1)

Кроме того, нам также понадобятся лестничные операторы, пониженияl− = lx − ily и повышения l+ = lx + ily, действующих следующим образом:

l−|l,m〉=√

(l +m)(l −m+1) |l,m−1〉,

l+|l,m〉=√

(l −m)(l +m+1) |l,m+1〉.(4.2)

Запомнить возникающие в (4.2) корни помогает следующее рассуж-дение. Оба корня имеют вид

(l ·m)(l ·m+1), причем знаки в двухскобках противоположны: либо

(+)(−), либо√

(−)(+). Оператор l−должен давать 0 на самой младшей функции мультиплета, |l,m = −l〉,которую дальше некуда понижать, поэтому для него коэффициент имеетвид

(+)(−) (ноль дает первая скобка (l +(−l))). Аналогично операторповышения дает 0 на самой старшей функции |l,m = l〉, и таким образомего коэффициент имеет знаки

(−)(+). Лестничные операторы позволя-ют по одной функции регулярным образом восстановить весь мультиплет.Действие операторов l+ и l− определяется с точностью до фазового мно-жителя (в (4.2) использован один из стандартных вариантов), а вместо l±(и соответственно чисел l,m) могут стоять любой оператор момента исоответствующие ему квантовые числа.

Процедура сложения моментов j1,2 основана на анализе собствен-ных чисел M = m1 + m2 оператора суммарной проекции Jz = j1z + j2z иопределении момента как максимальной возможной проекции.В мультипликативном базисе можно получить максимальную суммарнуюпроекцию Mmax = j1 + j2, причем единственным способом, взяв функ-цию |m1 = j1〉|m2 = j2〉. Соответственно ровно одно состояние с проекци-ей Mmax = j1 + j2 будет и в базисе суммарного момента, и ему отвечает исуммарный момент Jmax = j1+ j2. Таким образом, состояние с максималь-ным суммарным моментом и его проекцией всегда строится однозначно:

|J = j1 + j2,M = J〉 = |m1 = j1〉|m2 = j2〉. (4.3)

Для нашего случая получим:∣∣5

2 , 52

⟩=∣∣ 3

2

⟩∣∣1⟩. (4.4)

Page 83: КВАНТОВАЯ МЕХАНИКА МОЛЕКУЛ · PDF file2 ББК В22.31 УДК 544.11+539.183 isbn 978-5-94356-719-3 Стась Д.В., Плюснин В.Ф. Квантовая

4.1 ПРОЦЕДУРА СЛОЖЕНИЯ МОМЕНТОВ 83

Теперь, имя самую старшую функцию мультиплета, оператором J−можно «прорастить» из нее и весь мультиплет. На практике делать этоудобно в два этапа: сначала, действуя оператором в виде J− = j1− + j2−,получить относительные коэффициенты в разложении:

|J,M〉= ∑M=m1+m2

C J,Mm1,m2

|m1〉|m2〉, (4.5)

а затем нормировать полученную линейную комбинацию. Для нашегоконкретного случая имеем:

J−∣∣ 5

2, 52

⟩= ( j1− + j2−)

∣∣3

2

⟩∣∣1⟩

= j1−∣∣ 3

2

⟩∣∣1⟩+ j2−

∣∣ 3

2

⟩∣∣1⟩

=

=√

3∣∣ 1

2

⟩∣∣1⟩+√

2∣∣ 3

2

⟩∣∣0⟩,

∣∣ 5

2, 32

⟩=

√3∣∣ 1

2

⟩∣∣1⟩+√

2∣∣ 3

2

⟩∣∣0⟩

√5

, (4.6)

где в первой строке жирным шрифтом показано, на какие функции впроизведении действуют соответствующие операторы индивидуальныхмоментов. Далее,

J−∣∣ 5

2, 32

⟩∼ ( j1− + j2−)

(√3∣∣ 1

2

⟩∣∣1⟩+√

2∣∣ 3

2

⟩ ∣∣0⟩)

=

= 2√

3∣∣−1

2

⟩∣∣1⟩+2

√6∣∣ 1

2

⟩∣∣0⟩+2∣∣3

2

⟩∣∣−1

⟩,

∣∣ 5

2, 12

⟩=

√3∣∣−1

2

⟩∣∣1⟩+√

6∣∣ 1

2

⟩∣∣0⟩+∣∣ 3

2

⟩ ∣∣−1

√10

. (4.7)

Чтобы получить следующую функцию∣∣ 5

2 ,−12

⟩, можно в принципе еще

раз выполнить понижение J−∣∣ 5

2, 12

⟩. Однако видно, что с продвижением

по мультиплету сложность вычислений быстро нарастает. Кроме того, наэтом шаге впервые появится «холостое» понижение: так, j2−

∣∣3

2

⟩∣∣−1

⟩≡ 0.

В этой ситуации проще будет зайти с другой стороны мультиплета и,рассуждая аналогично, сразу выписать функцию

∣∣ 5

2,−52

⟩:

∣∣ 5

2,−52

⟩=∣∣−3

2

⟩∣∣−1

⟩, (4.8)

а затем оператором J+ прорастить из нее∣∣ 5

2,−32

⟩и затем

∣∣ 5

2,−12

⟩:

J+

∣∣ 5

2,−52

⟩= ( j1+ + j2+)

∣∣− 3

2

⟩∣∣−1

⟩=√

3∣∣−1

2

⟩∣∣−1

⟩+√

2∣∣−3

2

⟩ ∣∣0⟩,

∣∣ 5

2,−32

⟩=

√3∣∣− 1

2

⟩∣∣−1

⟩+√

2∣∣−3

2

⟩∣∣0⟩

√5

, (4.9)

Page 84: КВАНТОВАЯ МЕХАНИКА МОЛЕКУЛ · PDF file2 ББК В22.31 УДК 544.11+539.183 isbn 978-5-94356-719-3 Стась Д.В., Плюснин В.Ф. Квантовая

84 ГЛАВА 4. МНОГОЭЛЕКТРОННЫЕ ВОЛНОВЫЕ ФУНКЦИИ

J+

∣∣5

2 ,−32

⟩∼ ( j1+ + j2+)

(√3∣∣−1

2

⟩∣∣−1

⟩+√

2∣∣− 3

2

⟩∣∣0⟩)

=

= 2√

3∣∣ 1

2

⟩∣∣−1

⟩+2

√6∣∣−1

2

⟩∣∣0⟩+2∣∣−3

2

⟩∣∣⟩,

∣∣5

2 ,−12

⟩=

√3∣∣ 1

2

⟩ ∣∣−1

⟩+√

6∣∣−1

2

⟩∣∣0⟩+∣∣−3

2

⟩∣∣1⟩

√10

. (4.10)

Давайте выпишем рядом все полученные нами функции мультиплетас суммарным моментом J = 5

2 :

∣∣ 5

2, 52

⟩=

∣∣ 3

2

⟩∣∣1⟩,

∣∣ 5

2, 32

⟩=

√3∣∣ 1

2

⟩∣∣1⟩+√

2∣∣ 3

2

⟩∣∣0⟩

√5

,

∣∣ 5

2, 12

⟩=

√3∣∣−1

2

⟩∣∣1⟩+√

6∣∣ 1

2

⟩ ∣∣0⟩+∣∣3

2

⟩∣∣−1

√10

,

∣∣ 5

2,−12

⟩=

√3∣∣ 1

2

⟩∣∣−1

⟩+√

6∣∣−1

2

⟩∣∣0⟩+∣∣−3

2

⟩∣∣1⟩

√10

,

∣∣ 5

2,−32

⟩=

√3∣∣−1

2

⟩∣∣−1

⟩+√

2∣∣− 3

2

⟩∣∣0⟩

√5

,

∣∣ 5

2,−52

⟩=

∣∣−3

2

⟩∣∣−1

⟩.

(4.11)

Можно заметить симметричное устройство функций |J,M〉, отличаю-щихся только знаком проекции: если |J,M〉= a1|m1

1〉|m12〉+a2|m2

1〉|m22〉+ . . .,

то |J,−M〉 = a1|−m11〉|−m1

2〉+ a2|−m21〉|−m2

2〉+ . . . с теми же коэффициен-тами. Следующий из этого наблюдения вывод позволяет практическивдвое сократить вычисления: чтобы из функции |J,M〉 получить функцию|J,−M〉, нужно в разложении первой функции просто перевернуть зна-ки у всех проекций индивидуальных моментов (m →−m,−m → m,0→ 0).Как мы скоро увидим, причина заключается во вполне определенномповедении функций с определенными значениями момента и его проек-ции на некоторую ось z относительно операции отражения в плоскости,содержащей ось.

Таким образом, построив состояние с максимальной проекцией сум-марного момента, мы тем самым породили самый старший мультиплет, азатем лестничными операторами прорастили и все его функции. Теперьдавайте возьмем следующую проекцию, на единицу меньше максималь-ной: M = j1 + j2−1. В мультипликативном базисе имется два состояния

Page 85: КВАНТОВАЯ МЕХАНИКА МОЛЕКУЛ · PDF file2 ББК В22.31 УДК 544.11+539.183 isbn 978-5-94356-719-3 Стась Д.В., Плюснин В.Ф. Квантовая

4.1 ПРОЦЕДУРА СЛОЖЕНИЯ МОМЕНТОВ 85

с такой проекцией, φ1 =∣∣ 3

2

⟩∣∣0⟩

и φ2 =∣∣ 1

2

⟩∣∣1⟩. Столько же состояний

с M = 32 должно быть и в базисе полного момента, причем связь между

функциями двух базисов линейна: Φ1 = aφ1+bφ2, Φ2 = cφ1+dφ2. Одна изфункций |K,M = 3

2〉 попадает в мультиплет с J = 52 — это уже полученное

нами состояние | 52, 3

2〉. Второе же осталось свободным, и оно опять имеетмаксимальную полную проекцию в системе, а значит и порождает новыймультиплет с J = 3

2. Итого имеем:

∣∣3

2 , 32

⟩= a

∣∣ 3

2

⟩∣∣0⟩+b∣∣1

2

⟩∣∣1⟩. (4.12)

Найти коэффициенты a и b помогают соотношения ортогональности.Функция | 3

2, 32〉 должна быть ортогональна всем остальным функциям

базиса полного момента, в том числе и уже найденным функциям | 52,M〉:

⟨52,M

∣∣ 3

2, 32

⟩= 0. (4.13)

Однако из шести таких выражений полезным будет только соотношениеортогональности с функцией | 5

2, 32〉, которая разложена по тем же самым

функциям мультипликативного базиса. Имеем:

0 =⟨ 5

2, 32

∣∣ 3

2, 32

⟩=

=(√

3⟨

12

∣∣⟨1∣∣+

√2⟨

32

∣∣⟨0∣∣

)(

a∣∣3

2

⟩∣∣0⟩+b∣∣1

2

⟩ ∣∣1⟩)

=

= a√

2+b√

3. (4.14)

Второе уравнение на a,b дает условие нормировки, и мы получаемследующую систему:

a√

2+b√

3 = 0;

a2+b2 = 1.(4.15)

откуда (с точностью до общего фазового множителя) a =√

35 , b = −

√25,

и в итоге получаем:

∣∣ 3

2, 32

⟩=

√3∣∣ 3

2

⟩∣∣0⟩−√

2∣∣ 1

2

⟩∣∣1⟩

√5

. (4.16)

Полезно заметить, что если некоторая функция записана в видеразложения по двум ортонормированным функциям: Φ1 = αφ1 +β φ2, тонормированная и ортогональная ей функция обязательно имеет вид

Page 86: КВАНТОВАЯ МЕХАНИКА МОЛЕКУЛ · PDF file2 ББК В22.31 УДК 544.11+539.183 isbn 978-5-94356-719-3 Стась Д.В., Плюснин В.Ф. Квантовая

86 ГЛАВА 4. МНОГОЭЛЕКТРОННЫЕ ВОЛНОВЫЕ ФУНКЦИИ

Φ2 = β φ1−αφ2 — нужно просто переставить местами коэффициенты и уодного из них поменять знак (в случае комплексных коэффициентов ещенужно выполнить комплексное сопряжение).

Имея теперь функцию | 32,

32〉, оператором J− опять проращиваем из

нее функцию | 32, 1

2〉, а затем, перевернув знаки у всех проекций, сразувыписываем и функции | 3

2,−32〉, | 3

2,−12〉. Итого мультиплет с J = 3

2 состоитиз следующих нормированных функций:

∣∣ 3

2, 32

⟩=

√3∣∣3

2

⟩∣∣0⟩−√

2∣∣ 1

2

⟩∣∣1⟩

√5

,

∣∣ 3

2, 12

⟩=

√6∣∣3

2

⟩∣∣−1

⟩+∣∣ 1

2

⟩∣∣0⟩−2

√2∣∣−1

2

⟩ ∣∣1⟩

√15

,

∣∣ 3

2,−12

⟩=

√6∣∣− 3

2

⟩∣∣1⟩+∣∣−1

2

⟩ ∣∣0⟩−2

√2∣∣ 1

2

⟩ ∣∣−1

√15

,

∣∣ 3

2,−32

⟩=

√3∣∣− 3

2

⟩∣∣0⟩−√

2∣∣−1

2

⟩ ∣∣−1

√5

.

(4.17)

Следующей идет полная проекция M = 12, которую в мультипликатив-

ном базисе можно набрать тремя способами:∣∣ 3

2

⟩∣∣−1

⟩,∣∣1

2

⟩∣∣0⟩, и∣∣− 1

2

⟩∣∣1⟩.

Соответственно в базисе полного момента также будет три функции спроекцией M = 1

2 . Две из них обязательно попадут в уже полученныемультиплеты (это функции | 5

2, 12〉 и | 3

2,12〉), а одна останется свободной и

породит новый мультиплет с J = 12 :

∣∣ 1

2, 12

⟩= a

∣∣3

2

⟩∣∣−1

⟩+b∣∣1

2

⟩∣∣0⟩+∣∣−1

2

⟩∣∣1⟩. (4.18)

Теперь нам нужны три уравнения, чтобы найти три неизвестных ко-эффициента a,b,c. Этими уравнениями будут два соотношения ортого-нальности с функциями | 5

2, 12〉 и | 3

2,12〉, которые разложены по тем же

функциями мультипликативного базиса, и всегда имеющееся условиенормировки. Итого имеем:

⟨52, 1

2

∣∣ 1

2 , 12

⟩= 0 : a +b

√6+ c

√3 = 0;

⟨ 32, 1

2

∣∣ 1

2 , 12

⟩= 0 : a

√6+b−2c

√2 = 0;

⟨ 12, 1

2

∣∣ 1

2 , 12

⟩= 1 : a2 +b2 + c2 = 1.

(4.19)

Решение этой системы дает a =√

36 , b = −

√26, c =

√16 , так что функции

Page 87: КВАНТОВАЯ МЕХАНИКА МОЛЕКУЛ · PDF file2 ББК В22.31 УДК 544.11+539.183 isbn 978-5-94356-719-3 Стась Д.В., Плюснин В.Ф. Квантовая

4.1 ПРОЦЕДУРА СЛОЖЕНИЯ МОМЕНТОВ 87

¤ ¥ ¦§ ¥ ¦

¨ § ¥ ¦¨ ¤ ¥ ¦

§©

¨ §

ª ¥ ¦¤ ¥ ¦ ¤ ¥ ¦§ ¥ ¦ § ¥ ¦ § ¥ ¦

¨ § ¥ ¦ ¨ § ¥ ¦ ¨ § ¥ ¦¨ ¤ ¥ ¦ ¨ ¤ ¥ ¦¨ ª ¥ ¦

« ¬ ­ ®

¯ ° ± ¯ ²Рис. 4.1. Процедура сложения моментов j1 = 3

2 и j2 = 1.

мультиплета с J = 12 имеют вид:

∣∣ 1

2, 12

⟩=

√3∣∣ 3

2

⟩∣∣−1

⟩−√

2∣∣ 1

2

⟩ ∣∣0⟩+∣∣− 1

2

⟩∣∣1⟩

√6

,

∣∣ 1

2,−12

⟩=

√3∣∣−3

2

⟩∣∣1⟩−√

2∣∣−1

2

⟩∣∣0⟩+∣∣ 1

2

⟩∣∣−1

√6

.

(4.20)

На этом решение нашей задачи заканчивается: мы построили все 12искомых функций базиса полного момента. Общая процедуры решениядля случая сложения двух моментов показана схематически на рис. 4.1:построение начинается с левого верхнего угла треугольной схемы, в ко-тором находится состояние с максимальной суммарной проекцией, столб-цы схемы отвечают мультиплетам с определенным значением суммарногомомента J, строки — состояниям с одной и той же проекцией суммар-ного момента M. Для перемещения по столбцам используются лестнич-ные операторы, переход в следующий столбец выполняется в его самойкрайней (верхней или нижней) клетке с помощью соотношений ортого-нальности для всех функций данной строки из предыдущих мультипле-тов. Число функций мультипликативного базиса, входящих в разложениифункции полного момента с некоторым значением M, равно числу клетокв соответствующей строке, хотя некоторые из коэффициентов разложе-ния могут оказаться и нулевыми. Новые мультиплеты порождаются дотех пор, пока при уменьшении полной проекции увеличивается числоспособов, которыми данную проекцию можно набрать, до есть до ис-черпания короткого столбца в правой части схемы. Так, например, при

Page 88: КВАНТОВАЯ МЕХАНИКА МОЛЕКУЛ · PDF file2 ББК В22.31 УДК 544.11+539.183 isbn 978-5-94356-719-3 Стась Д.В., Плюснин В.Ф. Квантовая

88 ГЛАВА 4. МНОГОЭЛЕКТРОННЫЕ ВОЛНОВЫЕ ФУНКЦИИ

сложении моментов 52 и 1 мы бы все равно получили три мультиплета

(с полным моментом J = 72, 5

2, 32).

При сложении больше чем двух моментов задача принципиальноусложняется, поскольку теперь при уменьшении суммарной проекциина 1 число способов ее набрать может увеличиваться больше чем на 1,и однозначно написать старшие функции для всех новых порождаемыхмультиплетов описанным способом уже не получится. Здесь на помощьприходят либо методы теории групп, либо попарное сложение моментов.Отметим также для дальнейшего, что для часто встречающихся момен-тов 1

2 и 1 все ненулевые матричные элементы лестничных операторов(корни в (4.2)) равны соответственно 1 и

√2, что позволяет экономить

вычисления. Для момента же 32 и выше они оказываются разными для

разных проекций — именно поэтому в качестве примера и был взят«неудобный» момент 3

2.Отметим также следующий подводный камень, который создает опи-

санная упрощенная процедура получения функций с отрицательнымипроекциями путем смены знака всех индивидуальных проекций. Приэтом действительно получается базис как максимальный набор линей-но независимых функций в данном пространстве (с определенным пол-ным моментом). Однако функции базиса всегда определены с точностьюдо произвольной фазы, и иногда возникает необходимость их взаимногосогласования. Так, иногда нужно, чтобы лестничные операторы, опреде-ленные соотношением (4.2), переводили все функции мультиплета другв друга без смены знака. Как можно проверить, в нашем примере дляэтого придется поменять знаки у выписанных автоматически функций сотрицательной проекцией с моментом 3

2 (4.17), в мультиплетах с J = 52

и j = 12 (4.11, 4.20) такой проблемы в данном случае не возникает.

4.2 Детерминанты Слэйтера и спин-орбитали

Волновая функция многоэлектронной системы обязательно должнабыть антисимметрична относительно перестановки любой пары электро-нов — таков теоретический эквивалент принципа Паули. Требование ан-тисимметричности должно обязательно удовлетворяться при построениилюбых многоэлектронной волновой функции. До сих пор мы от негоабстрагировались, беря в качестве исходного базиса для описания сис-темы просто мультипликативный базис |φ1〉|φ2〉 . . .|φN〉, где сомножителиотносились к индивидуальным электронам. Теперь нам нужно научить-

Page 89: КВАНТОВАЯ МЕХАНИКА МОЛЕКУЛ · PDF file2 ББК В22.31 УДК 544.11+539.183 isbn 978-5-94356-719-3 Стась Д.В., Плюснин В.Ф. Квантовая

4.2 ДЕТЕРМИНАНТЫ СЛЭЙТЕРА И СПИН-ОРБИТАЛИ 89

ся антисимметризовать функции полученного таким образом базиса поперестановкам пар электронов, точнее, по перестановкам описывающихэлектроны комплектов переменных, пространственных плюс спиновых.При этом число линейно независимых функций — размер нашего бази-са — существенно сокращается. Так, для системы из двух p электроновналожение связанных с принципом Паули ограничений сокращает числодопустимых состояний с формально возможных 6×6 = 36 до C2

6 = 15.Регулярную процедуру антисимметризации многоэлектронной волно-

вой функции относительно перестановок электронов дает построение такназываемого детерминанта Слэйтера (ДС): для N электронов 1,2, . . .N,находящихся в состояниях φ1,φ2 . . .φN, нормированная антисимметри-зованная волновая функция дается символическим определителем:

Φ(φ1,φ2, . . .,φN) =1√N!

∑(−1)PPφ1(1) . . .φN(N) =

=1√N!

∣∣∣∣∣∣∣∣∣∣∣

φ1(1) φ1(2) · · · φ1(N)φ2(1) φ2(2) · · · φ2(N)φ3(1) φ3(2) · · · φ3(N)...

.... . .

...φN(1) φN(2) · · · φN(N)

∣∣∣∣∣∣∣∣∣∣∣

, (4.21)

где столбцы отличаются друг от друга только номером электрона, а стро-ки — только номером функции. P и (−1)P дают оператор перестановкии ее четность, соответственно, суммирование ведется по всем переста-новкам N чисел. Запись φk(m) означает, что из нашего набора беретсяфункция φk, формальными аргументами которой будут координаты m-гоэлектрона. Антисимметричность построенной таким образом функцииотносительно перестановок электронов автоматически обеспечивается ан-тисимметричностью определителя относительно перестановки столбцов,а нормировочный множитель 1/

√N! появился из-за того, что при

раскрытии определителя размером N ×N получится ровно N! линейнонезависимых слагаемых.

Поскольку раскрытие определителя большого размера требует оченьгромоздких выкладок, мы в дальнейшем будем ограничиваться в основ-ном только определителями 2× 2, уже достаточными для построениясодержательных задач. Явный вид функции (4.21) для двух электроновприведен ниже:

Φ(φ1,φ2) =1√2

∣∣∣∣

φ1(1) φ1(2)φ2(1) φ2(2)

∣∣∣∣=

φ1(1)φ2(2)−φ1(2)φ2(1)√2

. (4.22)

Page 90: КВАНТОВАЯ МЕХАНИКА МОЛЕКУЛ · PDF file2 ББК В22.31 УДК 544.11+539.183 isbn 978-5-94356-719-3 Стась Д.В., Плюснин В.Ф. Квантовая

90 ГЛАВА 4. МНОГОЭЛЕКТРОННЫЕ ВОЛНОВЫЕ ФУНКЦИИ

Используемую для записи антисимметризованной функции нотациюможно значительно упростить, если договориться, что при раскрытииопределителя в произведениях одноэлектронных функций первой всегдабудет записываться функция, зависящая от координат первого электрона(то есть φk(1)), второй — от второго (φl(2)) и так далее по порядку.Тогда номер электрона можно просто опустить, и получим стандартныйвид раскрытого двухэлектронного детерминанта Слэйтера:

Φ(φ1,φ2) =φ1(1)φ2(2)−φ2(1)φ1(2)√

2≡ φ1φ2−φ2φ1√

2. (4.23)

Раскрытый детерминант Слэйтера для трехэлектронной функции будетиметь вид

Φ(φ1,φ2,φ3) =φ1φ2φ3−φ1φ3φ2−φ2φ1φ3 +φ3φ1φ2 +φ2φ3φ1−φ3φ2φ1√

6, (4.24)

и так далее. Можно заметить, что отдельные слагаемые в выраженияхдля функций (4.23) и (4.24) отличаются друг от друга только переста-новкой индексов (соответственно 1,2 и 1,2,3), а знак при слагаемомдается четностью соответствующей ему перестановки. Все это конеч-но является отражением общих алгебраических свойств определителя(4.21), порождающего функцию, и обобщается на случай функции дляпроизвольного числа электронов. В принципе теперь можно опустить изнотации и буквы φ и сохранить только сами индексы:

Φ(1,2,3) =(1,2,3)− (1,3,2)− (2,1,3)+(3,1,2)+(2,3,1)− (3,2,1)√

6,

Φ(1,2) =(1,2)− (2,1)√

2, (4.25)

но для решения конкретных задач это уже слишком абстрактная запись.Обсуждая многоэлектронные волновые функции, например, при по-

строении таблицы Рассел—Саундерса, мы говорили о состояниях вида«1+0−», что означало «один (p) электрон в состоянии с ml = 1,ms = 1

2,другой — в состоянии с ml = 0,ms = −1

2». Давайте адаптируем записьфункции (4.22) к описанию таких состояний. Выделим в функциях φk(n)в явном виде орбитальную и спиновые части:

φk(n) ≡ φk(~rn)ξk(msn), (4.26)

Page 91: КВАНТОВАЯ МЕХАНИКА МОЛЕКУЛ · PDF file2 ББК В22.31 УДК 544.11+539.183 isbn 978-5-94356-719-3 Стась Д.В., Плюснин В.Ф. Квантовая

4.2 ДЕТЕРМИНАНТЫ СЛЭЙТЕРА И СПИН-ОРБИТАЛИ 91

для спиновых функций введем стандартные обозначения α (ms = 12) и

β (ms = −12), а для орбитальных функций в качестве индекса поставим

проекцию ml (соответственно φ−, φ0 и φ+ для p электрона). Примемтакже, что координатная часть одноэлектронной функции не зависит отспиновой, и, например, состояния 1+ и 1− соответствуют функциям видаφ+α и φ+β с одной и той же орбитальной частью φ+. Такое приближениесоответствует используемому в квантовой химии «ограниченному методуХартри-Фока» (ОХФ) и тоже вносит свои ошибки, особенно серьезныедля систем с открытой электронной оболочкой. Однако, как мы уже ви-дели на простейшем примере атома водорода, что-либо рассчитать точнодля реальных систем только руками и головой — задача практическиневозможная, анализировать можно только идеализованные модели, априближение ОХФ значительно упрощает такой анализ для многоэлек-тронных функций.

В наших новых обозначениях состояние 1+0−, которое мы теперьбудем обозначать Φ(1+0−), принимает вид

Φ(1+0−) =φ+α φ0β −φ0β φ+α√

2, (4.27)

или, перегруппировав орбитальные и спиновые сомножители, в болееудобном виде:

Φ(1+0−) =φ+φ0 αβ −φ0φ+ β α√

2, (4.28)

где и в орбитальной, и в спиновый частях первый сомножитель всегдаотносится к формально первому электрону, второй — ко второму и т. д.Запись вида (4.28) удобна тем, что иногда позволяет привести подобныеи записать многоэлектронную функцию в более обозримом виде. Так,например, для состояния 1+1−

Φ(1+1−) =φ+φ+ αβ −φ+φ+ β α√

2= φ+φ+

αβ −β α√2

, (4.29)

и функция распалась на произведение нормированных координатной испиновых функции. Это, конечно, тоже следствие приближения (4.26).Наконец, отметим, что одноэлектронные функции вида (4.26) называютспин-орбиталями — одноэлектронными состояниями в многоэлектроннойсистеме с явно выделенными спиновыми переменными.

Page 92: КВАНТОВАЯ МЕХАНИКА МОЛЕКУЛ · PDF file2 ББК В22.31 УДК 544.11+539.183 isbn 978-5-94356-719-3 Стась Д.В., Плюснин В.Ф. Квантовая

92 ГЛАВА 4. МНОГОЭЛЕКТРОННЫЕ ВОЛНОВЫЕ ФУНКЦИИ

4.3 Функции |L,ML;S,MS〉 в пределе LS-связи

Давайте опять обратимся к нашей модельной задаче о конфигурацииp2 и вспомним, как в приближении LS-связи для нее выглядит табли-ца Рассел—Саундерса (рис. 4.2). До сих пор мы пользовались таблицейтолько для подсчета числа состояний с определенными значениями ML

и MS, даваемого числом комбинаций, которые можно было построитьдля соответствующей клетки. Однако теперь можно заметить, что в ис-пользованной при построении таблицы символике закодирована еще иполезная информация о явном виде соответствующей функции — этоопределители вида (4.22), построенные на указанных функциях. Такимобразом, каждый элемент в таблице Рассел—Саундерса символизируетДС, т. е. вполне определенную многоэлектронную волновую функцию,приведенную по перестановочной симметрии входящих в нее электронов.Построением таблицы мы переходим от исходного мультипликативногобазиса к правильным образом симметризованному мультипликативномубазису, пусть пока чисто символически. Базис этот ортонормированный:ДС нормированы по построению, а разные ДС ортогональны, посколькуразличаются хотя бы одной одноэлектронной функцией, входящей со-множителем в каждое из слагаемых даваемой ДС линейной комбинации.

Теперь вспомним, как мы по таблице строили термы атома, и найдетсамое старшее (верхнее левое) состояние — им будет состояние «1+1−»в клетке ML = 2, MS = 0. Это состояние порождало у нас терм 1D сопределенным значением орбитального момента L = 2 и спина S = 0, пятьфункций которого различались проекцией ML. Соответствующий опреде-литель мы уже раскрывали в (4.29):

Φ(1+1−) = φ+φ+αβ −β α√

2. (4.30)

Можно заметить, что орбитальная часть в (4.30) соответствует функциис L = 2, ML = 2, получаемой при сложении двух моментов l = 1, а спиноваячасть отвечает синглетному состоянию для пары спинов 1

2. Таким обра-зом, мы получили состояние с определенными значениями L,ML,S,MS —старшую функцию старшего терма 1D:

|L = 2,ML = 2;S = 0,MS = 0〉 = Φ(1+1−) = φ+φ+αβ −β α√

2, (4.31)

причем построили мы ее сразу, так же как самую старшую функциюбазиса полного момента в задаче о сложении моментов.

Page 93: КВАНТОВАЯ МЕХАНИКА МОЛЕКУЛ · PDF file2 ББК В22.31 УДК 544.11+539.183 isbn 978-5-94356-719-3 Стась Д.В., Плюснин В.Ф. Квантовая

4.3 ФУНКЦИИ |L,ML;S,MS〉 В ПРЕДЕЛЕ LS-СВЯЗИ 93

³ ´³µ ¶ · ¸ ¶¹ º ¶» ¶¼ º

¶ ¶»·» ¶»·¼¶¼·» ¶¼·¼

· ¶»¸ ¶» ¶»·¼¶¼·»·»·¼ ¶¼¸ ¶¼

¸ ¶ ¸ ¶»·» ¸ ¶»·¼¸ ¶¼·» ¸ ¶¼·¼

¸¹ º ¸ ¶»¸ ¶¼ º

Рис. 4.2. Таблица Рассел—Саундерса для электронной конфигурации p2.

Давайте теперь запишем ДС из следующей клетки ML = 1, MS = 0:

Φ(1+0−) =1√2

∣∣∣∣

φ+α φ+αφ0β φ0β

∣∣∣∣

=φ+φ0 αβ −φ0φ+ β α√

2,

Φ(1−0+) =1√2

∣∣∣∣

φ+β φ+βφ0α φ0α

∣∣∣∣

=φ+φ0 β α −φ0φ+ αβ√

2.

(4.32)

Можно заметить, что это уже не есть функции с определенными L, S,так же как при сложении моментов две функции мультипликативногобазиса с проекцией, на единицу меньше максимальной, уже не былисостояниями с определенными значениями суммарного момента. Однакоих линейные комбинации давали функции с определенными значениямиполного момента, попадающие в разные мультиплеты. Взяв по аналогиисумму и разность функций (4.32), после группировки получим:

Φ1 =Φ(1+0−)+Φ(1−0+)√

2=

φ+φ0 −φ0φ+√2

αβ +β α√2

,

Φ2 =Φ(1+0−)−Φ(1−0+)√

2=

φ+φ0 +φ0φ+√2

αβ −β α√2

.

(4.33)

Page 94: КВАНТОВАЯ МЕХАНИКА МОЛЕКУЛ · PDF file2 ББК В22.31 УДК 544.11+539.183 isbn 978-5-94356-719-3 Стась Д.В., Плюснин В.Ф. Квантовая

94 ГЛАВА 4. МНОГОЭЛЕКТРОННЫЕ ВОЛНОВЫЕ ФУНКЦИИ

В координатной части второй функции опять можно узнать состоянияс L = 2, ML = 1 из задачи о сложении двух моментов l = 1, а в спиновойчасти — состояние с S = 0,MS = 0 для задачи о сложении спинов 1

2. Дляпервой функции имеем координатную часть |L = 1, ML = 1〉 и спиновуючасть |S = 1, MS = 0〉. Таким образом:

|2,1;0,0〉= Φ(1+0−)−Φ(1−0+)√2

=φ+φ0 +φ0φ+√

2

αβ −β α√2

,

|1,1;1,0〉= Φ(1+0−)+Φ(1−0+)√2

=φ+φ0 −φ0φ+√

2

αβ +β α√2

,

(4.34)

мы получили следующую функцию из терма 1D (с ML = 1, MS = 0) иодну из функций терма 3P (с ML = 1, MS = 0), причем получили их ввиде, не вызывающем сомнения в их антисимметричности относительноперестановки электронов.

В клетке ML = 1, MS = 1 имеем одно состояние «1+0+». Раскрываясоответствующий определитель, получим:

Φ(1+0+) =φ+φ0 −φ0φ+√

2αα = |1,1;1,1〉, (4.35)

старшую из функций терма 3P. Для клетки ML = 0, MS = 1 имеем:

Φ(1+−1+) =φ+φ−−φ−φ+√

2αα = |1,0;1,1〉. (4.36)

Можно заметить, что две функции |2,1;0,0〉 (4.34) и |2,2;0,0〉 (4.31)отличаются только формальным понижением орбитальной части от φ+φ+

до (φ+φ0 +φ0φ+)/√

2, т. е. от |2,2〉 до |2,1〉 для задачи о сложении двухмоментов l = 1, а функция |1,1;1,0〉 (4.34) отличается от |1,1;1,1〉 (4.35)понижением спиновой части от αα до (αβ + β α)/

√2, т. е. от |1,1〉

до |1,0〉 для задачи о двух спинах s = 12. Так оно конечно и должно быть,

так как имея одну из функций терма, все остальные можно прораститьиз нее соответствующими лестничными операторами L±,S±. Ситуацияполностью аналогична обычной задаче о сложении моментов и дает намрегулярный способ по одной функции восстановить весь терм, только те-перь двигаться нужно по двум независимым проекциям, орбитальной ML

и спиновой MS.

Page 95: КВАНТОВАЯ МЕХАНИКА МОЛЕКУЛ · PDF file2 ББК В22.31 УДК 544.11+539.183 isbn 978-5-94356-719-3 Стась Д.В., Плюснин В.Ф. Квантовая

4.3 ФУНКЦИИ |L,ML;S,MS〉 В ПРЕДЕЛЕ LS-СВЯЗИ 95

Заметим также, что если в клетке имеется всего одно состояние,то оно автоматически будет и состоянием с определенными значениямиполного орбитального момента и спина, т. е. одним из состояний терма,причем функцию для него можно выписать сразу. Ситуация здесь вполнеаналогична выписыванию старшей функции в cтандартной задаче о сло-жении моментов, которая порождает весь старший мультиплет. В нашейзадаче такими будут клетки ML = 2,MS = 0 и ML = 1,MS = 1, которыепорождают соответственно термы 1D и 3P. Если в клетке имеется большеодного состояния, то функции термов будут линейными комбинациямисоответствующих ДС. В нашей простейшей задаче мы эти комбинацииугадали (4.34), однако несложно построить и регулярную процедуру ихполучения, опять по аналогии с задачей о сложении моментов.

Давайте возьмем клетку ML = 1,MS = 0, которой соответствует двасостояния, по одному из термов 1D и 3P. Допустим, что мы уже имеемсостояние из терма 1D в виде:

|2,1;0,0〉 = aΦ(1+0−)+bΦ(1−0+). (4.37)

Поскольку ДС образуют ортонормированный базис, то вторая искомаяфункция получается автоматически из соображений ортогональности:

|2,1;0,0〉 = bΦ(1+0−)−aΦ(1−0+). (4.38)

Чтобы получить разложение (4.37), необходимо пройти следующуюцепочку: выписать функцию |2,2;0,0〉 в виде разложения по функцияммультипликативного базиса (4.31), подействовать оператором L− и по-лучить функцию |2,1;0,0〉 в виде разложения по функциям мультипли-кативного базиса (4.34), выписать соответствующие ДС через функциимультипликативного базиса в виде (4.32), и затем скалярно перемножитьзаписанные в одном базисе функции |2,1;0,0〉 и ДС:

a = 〈Φ(1+0−)|2,1;0,0〉=

=

⟨φ+φ0 αβ −φ0φ+ β α√

2

∣∣∣∣

φ+φ0 +φ0φ+√2

αβ −β α√2

=1√2

,

b = 〈Φ(1−0+)|2,1;0,0〉=

=

⟨φ+φ0 β α −φ0φ+ αβ√

2

∣∣∣∣

φ+φ0 +φ0φ+√2

αβ −β α√2

= − 1√2,

(4.39)

Page 96: КВАНТОВАЯ МЕХАНИКА МОЛЕКУЛ · PDF file2 ББК В22.31 УДК 544.11+539.183 isbn 978-5-94356-719-3 Стась Д.В., Плюснин В.Ф. Квантовая

96 ГЛАВА 4. МНОГОЭЛЕКТРОННЫЕ ВОЛНОВЫЕ ФУНКЦИИ

На первый взгляд кажется, что приходится делать много лишнейработы, однако реально с некоторой практикой выписывать раскрытыедетерминанты Слэйтера и выполнять умножение типа (4.39) можно оченьбыстро, а преимущество регулярности такого подхода становится опре-деляющим уже для чуть более сложных задач.

Давайте таким образом обработаем клетку ML = 0, MS = 0 с тремясостояниями. Соответствующие раскрытые ДС имеют вид:

Φ(1+−1−) =φ+φ−αβ −φ−φ+ β α√

2,

Φ(1−−1+) =φ+φ−β α −φ−φ+ αβ√

2,

Φ(0+0−) = φ0φ0αβ −β α√

2.

(4.40)

В клетку попадет по одной функции из термов 1D и 3P. Функцию |2,0;0,0〉получим действием L− на уже имеющуюся функцию |2,1;0,0〉 (4.34), афункцию |1,0;1,0〉 — действием S− на функцию |1,0;1,1〉 (4.36):

|2,0;0,0〉 =φ+φ−+2φ0φ0 +φ−φ+√

6

αβ −β α√2

,

|1,0;1,0〉 =φ+φ−−φ−φ+√

2

αβ +β α√2

.

(4.41)

Раскладывая функции (4.41) по базису (4.40), получаем:

|2,0;0,0〉 =Φ(1+−1−)+2Φ(0+0−)−Φ(1−−1+)√

6,

|1,0;1,0〉 =Φ(1+−1−)+Φ(1−−1+)√

2.

(4.42)

Третью функцию с ML = MS = 0, функцию |0,0;0,0〉 терма 1S, ищем ввиде разложения

|0,0;0,0〉 = aΦ(1+−1−)+bΦ(0+0−)+ cΦ(1−−1+). (4.43)

Требуемые три уравнения на коэффициенты a,b,c даются двумя со-отношениями ортогональности с функциями (4.42) и всегда имеющимся

Page 97: КВАНТОВАЯ МЕХАНИКА МОЛЕКУЛ · PDF file2 ББК В22.31 УДК 544.11+539.183 isbn 978-5-94356-719-3 Стась Д.В., Плюснин В.Ф. Квантовая

4.3 ФУНКЦИИ |L,ML;S,MS〉 В ПРЕДЕЛЕ LS-СВЯЗИ 97

условием нормировки:

〈2,0;0,0|0,0;0,0〉 = 0 : a +2b− c = 0

〈1,0;1,0|0,0;0,0〉 = 0 : a + c = 0

〈0,0;0,0|0,0;0,0〉 = 1 : a2 +b2 + c2 = 1

, (4.44)

откуда (с точностью до произвольного фазового множителя) получаем

для коэффициентов a =√

13 , b = −

√13, c = −

√13 , и

|0,0;0,0〉 =Φ(1+−1−)−Φ(0+0−)−Φ(1−−1+)√

3. (4.45)

Подставляя теперь ДС в виде (4.40) и приводя подобные, окончательнополучим:

|0,0;0,0〉 =φ+φ−−φ0φ0 +φ−φ+√

3

αβ −β α√2

. (4.46)

На этом мы завершили построение всех функций с неотрицатель-ными проекциями ML и MS. Как и раньше, функции с отрицательнымипроекциями ML и/или MS получаются из уже имеющихся сменой знака увсех соответственно индивидуальных орбитальных и/или спиновых про-екций. Итого все 15 функций термов конфигурации p2 в приближенииLS-связи имеют вид:

• для терма 1D:

|2,2;0,0〉 = φ+φ+αβ −β α√

2,

|2,1;0,0〉 =φ+φ0 +φ0φ+√

2

αβ −β α√2

,

|2,0;0,0〉 =φ+φ−+2φ0φ0 +φ−φ+√

6

αβ −β α√2

,

|2,−1;0,0〉 =φ−φ0 +φ0φ−√

2

αβ −β α√2

,

|2,−2;0,0〉 = φ−φ−αβ −β α√

2,

(4.47)

Page 98: КВАНТОВАЯ МЕХАНИКА МОЛЕКУЛ · PDF file2 ББК В22.31 УДК 544.11+539.183 isbn 978-5-94356-719-3 Стась Д.В., Плюснин В.Ф. Квантовая

98 ГЛАВА 4. МНОГОЭЛЕКТРОННЫЕ ВОЛНОВЫЕ ФУНКЦИИ

• для терма 3P:

|1,1;1,1〉 =φ+φ0 −φ0φ+√

2αα,

|1,1;1,0〉 =φ+φ0 −φ0φ+√

2

αβ +β α√2

,

|1,1;1,−1〉 =φ+φ0 −φ0φ+√

2β β ,

|1,0;1,1〉 =φ+φ−−φ−φ+√

2αα,

|1,0;1,0〉 =φ+φ−−φ−φ+√

2

αβ +β α√2

,

|1,0;1,−1〉 =φ+φ−−φ−φ+√

2β β ,

|1,−1;1,1〉 =φ−φ0 −φ0φ−√

2αα,

|1,−1;1,0〉 =φ−φ0 −φ0φ−√

2

αβ +β α√2

,

|1,−1;1,−1〉 =φ−φ0 −φ0φ−√

2β β .

(4.48)

Отметим, что если требуется сквозное согласование всех функцийбазиса по фазе (см. комментарий на стр. 88), то у последних трехфункций нужно поменять знак, поскольку, как можно проверитьпрямым вычислением, действие определенным в (4.2) оператором l−на функцию (φ+φ−−φ−φ+)/

√2 даст (φ0φ−−φ−φ0)/

√2, тогда как

использованный при построении функций (4.48) автоматическийпереворот проекций дает (φ−φ0 −φ0φ−)/

√2.

• для терма 1S:

|0,0;0,0〉 =φ+φ−−φ0φ0 +φ−φ+√

3

αβ −β α√2

. (4.49)

Можно отметить, что во всех выписанных функциях координатные испиновые части разделились, и они выглядят как произведение коорди-натной функции с определенным значением L,M и спиновой функции сопределенным значением S,M — это есть следствие простоты рассмот-ренной задачи, и в общем случае не всегда возможно.

Page 99: КВАНТОВАЯ МЕХАНИКА МОЛЕКУЛ · PDF file2 ББК В22.31 УДК 544.11+539.183 isbn 978-5-94356-719-3 Стась Д.В., Плюснин В.Ф. Квантовая

4.4 ФУНКЦИИ |L,S,J,JZ〉 В ПРЕДЕЛЕ LS-СВЯЗИ 99

ДС дают удобный базис для расчетов, поэтому часто функции термовне доводят до явного разложения вида (4.47–4.49), а оставляют в видеразложений по ДС вида (4.42), рассчитывая затем матричные элементывида 〈Φ1 |V |Φ2〉. Такое разложение для части функций терма основногосостояния 4F более сложной конфигурации d3 приведено ниже:

∣∣3,3; 3

2 , 32

⟩= Φ(2+1+0+),

∣∣3,2; 3

2 , 32

⟩= Φ(2+1+−1+),

∣∣3,1; 3

2 , 32

⟩=

√3Φ(2+1+−2+)+

√2Φ(2+0+−1+)√

5,

∣∣3,0; 3

2 , 32

⟩=

2Φ(2+0+−2+)+Φ(1+0+−1+)√5

,

∣∣3,−1; 3

2, 32

⟩=

√3Φ(−2+−1+2+)+

√2Φ(−2+0+1+)√

5,

∣∣3,−2; 3

2, 32

⟩= Φ(−2+−1+1+),

∣∣3,−3; 3

2, 32

⟩= Φ(−2+−1+0+).

(4.50)

Остальные функции терма проращиваются из приведенных действиемоператора S−.

4.4 Функции |L,S,J,Jz〉 в пределе LS-связи

Давайте теперь в рамках приближения LS-связи учтем (слабое) спин-орбитальное взаимодействие и построим для модельной конфигурации p2

функции термов с определенными значениями полного момента J и егопроекции Jz. Для этого нужно на пространстве функций каждого терма2S+1L решить задачу о сложении орбитального и спинового моментов,т. е. выполнить еще одну итерацию процедуры сложения моментов, пе-рейдя от функций |L,ML;S,MS〉 к функциями |L,S,J,Jz〉∗.

Электронная конфигурация p2 порождает следующие три терма: 1D,3P, 1S. Поскольку в термах 1D и 1S спин равен 0, уже имеющиеся у насфункции (4.47, 4.49) с точностью до переобозначений и будут искомыми

Page 100: КВАНТОВАЯ МЕХАНИКА МОЛЕКУЛ · PDF file2 ББК В22.31 УДК 544.11+539.183 isbn 978-5-94356-719-3 Стась Д.В., Плюснин В.Ф. Квантовая

100 ГЛАВА 4. МНОГОЭЛЕКТРОННЫЕ ВОЛНОВЫЕ ФУНКЦИИ

функциями полного момента J:

|2,0,2,JZ〉∗ ≡ |2,ML = Jz; 0,0〉,|0,0,0,0〉∗ ≡ |0,0;0,0〉

(4.51)

для термов 1D и 1S, соответственно. Для терма 3P получим расщеплениена термы 3P0, 3P1, 3P2, функции которых придется искать.

Прежде всего опять упростим обозначения и оставим в символахфункций только бегущие индексы, ML;MS и J,Jz, соответственно:

|L,ML;S,MS〉 → |ML;MS〉, |L,S,J,Jz〉∗ → |J,Jz〉∗. (4.52)

Имеем задачу о сложении двух моментов L = S = 1 и следуем обычнойпроцедуре. Старшая функция выписывается сразу:

|2,2〉∗ = |1;1〉. (4.53)

Затем дважды действуем оператором J− и получаем:

|2,1〉∗ =|1;0〉+ |0;1〉√

2,

|2,0〉∗ =|1;−1〉+2|0;0〉+ |−1;1〉√

2.

(4.54)

Переворачиваем проекции в функциях |2,2〉∗ и |2,1〉∗ и получаем

|2,−1〉∗ =|−1;0〉+ |0;−1〉√

2,

|2,−2〉∗ = |−1;−1〉.(4.55)

По ортогональности к функции |2,1〉∗ строим функцию |1,1〉∗ и затем изнее получаем весь мультиплет с J = 1:

|1,1〉∗ =|1;0〉− |0;1〉√

2,

|1,0〉∗ =|1;−1〉− |−1;1〉√

2,

|1,−1〉∗ =|−1;0〉− |0;−1〉√

2.

(4.56)

Page 101: КВАНТОВАЯ МЕХАНИКА МОЛЕКУЛ · PDF file2 ББК В22.31 УДК 544.11+539.183 isbn 978-5-94356-719-3 Стась Д.В., Плюснин В.Ф. Квантовая

4.4 ФУНКЦИИ |L,S,J,JZ〉 В ПРЕДЕЛЕ LS-СВЯЗИ 101

Наконец, по ортогональности строим функцию |0,0〉∗:

|0,0〉∗ =|1;−1〉− |0;0〉+ |−1;1〉√

3. (4.57)

Чтобы теперь в явном виде выразить полученные функции |J,Jz〉∗ черезодноэлектронные, нужно в полученные выражения (4.53–4.57) подста-вить соответствующие функции терма 3P из (4.48), не забыв согласоватьфазы внутри набора (см. комментарий на стр. 88). Так, например, дляфункции основного состояния (терм 3P0, 4.57) подстановка дает:

|0,0〉∗ =1√3

φ+φ0 −φ0φ+√

2β β − φ−φ0 −φ0φ−√

2αα −

− φ+φ−−φ−φ+√2

αβ +β α√2

.

(4.58)

Функция получена в виде, явно отражающем ее антисимметричностьотносительно перестановки электронов.

Подводя итог, можно сказать, что для получения в регулярном видефункции основного состояния конфигурации p2 в приближении LS-связис учетом слабого спин-орбитального взаимодействия мы последовательнои регулярно прошли через четыре базиса, постепенно учитывая все болеетонкие взаимодействия:

1. мультипликативный базис |l1m1〉|l2m2〉 — исходная формулировказадачи;

2. базис детерминантов Слэйтера Φ(lms11 lms2

2 ) — учет перестановочнойсимметрии многоэлектронной волной функции;

3. базис полного орбитального момента и спина |L,ML;S,MS〉 — учетмежэлектронных взаимодействий;

4. базис полного момента |L,S,J,Jz〉 — учет спин-орбитальных взаи-модействий.

В таком последовательном переходе из базиса в базис часто и со-стоит процедура аналитического решения задач по квантовой механикеатома или молекулы, причем правильный выбор базиса, соответствую-щего симметрии задачи, может катастрофически облегчить решение. Яр-кий пример тому дает переход в базис полного момента для нахождениясобственных чисел оператора вида a(~l1~l2), что позволяет сразу выписатьответ при сколь угодно большой размерности исходной задачи.

Page 102: КВАНТОВАЯ МЕХАНИКА МОЛЕКУЛ · PDF file2 ББК В22.31 УДК 544.11+539.183 isbn 978-5-94356-719-3 Стась Д.В., Плюснин В.Ф. Квантовая

102 ГЛАВА 4. МНОГОЭЛЕКТРОННЫЕ ВОЛНОВЫЕ ФУНКЦИИ

½¾ ¾¿ ¿ ÀÁ Á Á  Âà Á à Á à Áÿ à ¿ ÿà ¾ à ¾Ã½

Ä ÅÄ Æ Á ½¾ Ç È ÉÈ¿ÁÂ

Рис. 4.3. Фрагменты схемы сложения моментов и таблица Рассел—Саундерса,требуемые для нахождения волновой функции основного состояния электроннойконфигурации d2 в случае LS-связи с учетом слабого спин-орбитального взаимо-

действия и еще более слабого внешнего постоянного магнитного поля.

4.5 Сведение вычислений к минимуму

Задачи на построение многоэлектронных волновых функций, хотя иполностью алгоритмизуемые, тем не менее очень громоздки, поэтому прирешении конкретных задач лучше стараться сводить вычисления толькок реально необходимому минимуму. В качестве показательного примерадавайте рассмотрим задачу о нахождении волновой функции основногосостояния электронной конфигурации d2 в случае LS-связи с учетом сла-бого спин-орбитального взаимодействия и еще более слабого внешнегопостоянного магнитного поля. Кратность вырождения конфигурации d2

равна C210 = 45, и задача о последовательном преобразовании набора из 45

функций через три базиса требует огромного объема вычислений привесьма большой вероятности ошибки. Однако реально нам нужно най-ти одну функция — давайте посмотрим, что для этого действительнопридется сделать.

Последовательно пойдем по условию. Терм основного состояния кон-фигурации d2 в случае LS-связи можно найти из обращенных правилГунда (см. стр. 71): максимальный спин будет равен 1

2 + 12 = 1, при этом

максимальный орбитальный момент равен 2+ 1 = 3, получаем терм 3F.Поскольку наша оболочка заполнена менее чем наполовину, при уче-те слабого спин-орбитального взаимодействия основным окажется термс минимальным J = |L− S| = 2, получаем терм 3F2. Гамильтониан взаи-модействия со слабым внешним магнитным полем H = gβ BJz, поэтомунижним по энергии окажется состояние с Jz = −2. Таким образом, намнужно найти функцию |L = 3,S = 1,J = 2,Jz = −2〉∗ для конфигурации d2.

Page 103: КВАНТОВАЯ МЕХАНИКА МОЛЕКУЛ · PDF file2 ББК В22.31 УДК 544.11+539.183 isbn 978-5-94356-719-3 Стась Д.В., Плюснин В.Ф. Квантовая

4.5 СВЕДЕНИЕ ВЫЧИСЛЕНИЙ К МИНИМУМУ 103

Давайте будем разворачивать задачу с конца. Прежде всего заметим,что нужная нам функция |3,1,2,−2〉∗ — это почти функция |3,1,2,2〉∗(с положительной проекцией) и отличается от нее только сменой знакову всех индивидуальных проекций. Поскольку психологически работать сположительными проекциями значительно проще, чем с отрицательны-ми, нашим первым шагом будет замена

|3,1,2,−2〉∗ → |3,1,2,2〉∗. (4.59)

При сложении моментов L = 3 и S = 1 в полный момент J возможныJ = 4,3,2. Достаточный фрагмент схемы сложения моментов в этомслучае приведена на рис. 4.3 слева. Чтобы добраться то требуемой намфункции |J = 2,Jz = 2〉∗, нам придется построить функции |4,4〉∗, |4,3〉∗,|4,2〉∗, |3,3〉∗ и |3,2〉∗. Следуя стандартной процедуре сложения моментови сохраняя сокращенные обозначения предыдущего раздела, имеем:Старшая функция:

|4,4〉∗ = |3;1〉. (4.60)

Затем понижением получаем:

|4,3〉∗ =

√6|2;1〉+

√2|3;0〉√

2,

|4,2〉∗ =

√15|1;1〉+

√12|2;0〉+ |3;−1〉√28

.

(4.61)

По ортогональности строим функцию |3,3〉∗ из |4,3〉∗:

|3,3〉∗ =

√2|2;1〉−

√6|3;0〉√

8. (4.62)

Понижаем полученную функцию до |3,2〉∗:

|3,2〉∗ =

√5|1;1〉−2|2;0〉−

√3|3;−1〉√

12. (4.63)

По ортогональности строим искомую функцию |2,2〉∗:

|2,2〉∗ =|1;1〉−

√5|2;0〉+

√15|3;−1〉√

21. (4.64)

Итого нам понадобятся функции |1;1〉, |2;0〉 и |3;−1〉 терма 3F . Опятьвременно заменим функцию |3;−1〉 с отрицательной спиновой проекци-ей на парную ей функцию |3;1〉, отличающейся только сменой знака

Page 104: КВАНТОВАЯ МЕХАНИКА МОЛЕКУЛ · PDF file2 ББК В22.31 УДК 544.11+539.183 isbn 978-5-94356-719-3 Стась Д.В., Плюснин В.Ф. Квантовая

104 ГЛАВА 4. МНОГОЭЛЕКТРОННЫЕ ВОЛНОВЫЕ ФУНКЦИИ

всех спиновых проекций. На рис. 4.3 справа показан фрагмент таблицыРассел—Саундерса для конфигурации d2 с единственной заполненнойклеткой ML = 3,MS = 1, порождающей терм 3F . Соответствующая ейфункция |3;1〉 выписывается сразу через единственный ДС:

|3;1〉 = Φ(2+1+) =1√2

∣∣∣∣

φ2α φ2αφ1α φ1α

∣∣∣∣

=φ2φ1−φ1φ2√

2αα. (4.65)

Одна из искомых функций |3;−1〉 получается из нее сразу прозрачнойзаменой αα → β β :

|3;−1〉 =φ2φ1−φ1φ2√

2β β . (4.66)

Функцию |1;1〉 получим из |3;1〉 двухкратным действием оператора L−:

|2;1〉 =φ2φ0−φ0φ2√

2αα,

|1;1〉 =

√2(φ1φ0−φ0φ1)+

√3(φ2φ−1−φ−1φ2)√

10αα.

(4.67)

Функцию |2;0〉 получим из |2;1〉 действием оператора S−:

|2;0〉 =φ2φ0−φ0φ2√

2

αβ +β α√2

. (4.68)

Теперь собираем функцию |2,2〉∗ из (4.64):

|2,2〉∗ =1√21

√2(φ1φ0−φ0φ1)√

10αα +

√3(φ2φ−1−φ−1φ2)√

10αα −

−√

5(φ2φ0−φ0φ2)√10

αβ +β α√2

+

√15(φ2φ1−φ1φ2)√

10β β

. (4.69)

И, наконец, из (4.69) заменой α → β , β → α, 2→ −2, 1→ −1, −1→ 1получаем искомую функцию |2,−2〉∗:

|2,−2〉∗ =1√21

√2(φ−1φ0−φ0φ−1)√

10β β +

√3(φ−2φ1−φ1φ−2)√

10β β −

−√

5(φ−2φ0−φ0φ−2)√10

β α +αβ√2

+

√15(φ−2φ−1−φ−1φ−2)√

10αα

.(4.70)

Page 105: КВАНТОВАЯ МЕХАНИКА МОЛЕКУЛ · PDF file2 ББК В22.31 УДК 544.11+539.183 isbn 978-5-94356-719-3 Стась Д.В., Плюснин В.Ф. Квантовая

4.5 СВЕДЕНИЕ ВЫЧИСЛЕНИЙ К МИНИМУМУ 105

Подобная техника решения задачи «с конца» в некоторой степени за-щищает от лишних вычислений и помогает свести задачу к обозримой.Особенно полезно это становится при усложнении задачи, например, приувеличении числа электронов. В качестве примера давайте найдем функ-ции терма основного состояния электронной конфигурации p3 в пределеLS-связи с учетом слабого спин-орбитального взаимодействия. В прин-ципе в этой задаче имеется C3

6 = 20 трехэлектронных функций, не за-путаться в которых весьма непросто. Однако простые рассуждения по-казывают, что термом основного состояния будет терм 4S3/2 с нулевыморбитальным моментом, функции которого с точностью до переобозна-чений совпадают с функциями терма 4S до «учета» спин-орбитальноговзаимодействия, которое здесь тождественно равно нулю из-за нулево-го орбитального момента. Всего в терме будет 4 функции, причем однуиз них написать можно сразу — это функция с максимальной спиновойпроекцией, которая и порождает весь терм:

∣∣0; 3

2

⟩= Φ(1+0+−1+) =

1√6

∣∣∣∣∣∣

φ+ φ+ φ+

φ0 φ0 φ0

φ− φ− φ−

∣∣∣∣∣∣

ααα. (4.71)

Функция распалась в произведение координатной части Φc, одинаковойдля всего терма, и простой спиновой функции, понижение которой идаст функции терма. Координатную функцию Φc мы фактически ужевыписали в (4.24):

Φc =φ+φ0φ−−φ+φ−φ0 −φ0φ+φ− +φ−φ+φ0 +φ0φ−φ+−φ−φ0φ+√

6. (4.72)

Таким образом, искомые функции терма основного состояния 4S3/2 дляконфигурации p3 имеют вид:

∣∣0, 3

2 , 32, 3

2

⟩= Φc ααα,

∣∣0, 3

2 , 32, 1

2

⟩= Φc

ααβ +αβ α +β αα√3

,

∣∣0, 3

2 , 32,−1

2

⟩= Φc

αβ β +β β α +β αβ√3

,

∣∣0, 3

2 , 32,−3

2

⟩= Φc β β β .

(4.73)

Page 106: КВАНТОВАЯ МЕХАНИКА МОЛЕКУЛ · PDF file2 ББК В22.31 УДК 544.11+539.183 isbn 978-5-94356-719-3 Стась Д.В., Плюснин В.Ф. Квантовая

106 ГЛАВА 4. МНОГОЭЛЕКТРОННЫЕ ВОЛНОВЫЕ ФУНКЦИИ

4.6 Функции для термов предела j j-связи

В предельном случае j j-связи термы, а с ними и функции, ищутсянесколько по-другому, хотя подход в целом остается тем же самым —последовательный переход из базиса в базис с учетом все более тонкихвзаимодействий. Давайте в качестве примера опять построим функциидля нашей модельной конфигурации p2.

Возможные значения полного момента для p электрона j1,2 =1

2 , 32

,

и, как мы уже разбирались, конфигурация p2 порождает термы(1

2, 12

)

0,(12, 3

2

)

1,2 и(

32, 3

2

)

0,2. Терм(

12, 1

2

)

0 получался следующим образом: имеемдва электрона с одинаковыми орбитальными моментами l = 1, спинамиs = 1

2 и полными моментами j = 12 , состояния которых могут и в силу

принципа Паули обязаны различаться только проекцией jz = ±12 , откуда

максимальная суммарная проекция системы j1z + j2z = 0, а потому и еесуммарный момент J = 0. Вводя теперь для одноэлектронных состоянийобозначения

∣∣l = 1, s = 1

2, j = 12, jz = ±1

2

⟩= ϕ± 1

2, (4.74)

получаем, что один из наших электронов должен обязательно находитьсяв состоянии ϕ 1

2, а другой — в состоянии ϕ−1

2. Соответствующая волновая

функция в этом случае будет даваться детерминантом Слэйтера, постро-енным на функциях ϕ± 1

2:

∣∣ j1 = 1

2, j2 = 12,J = 0,Jz = 0

∗ =

=1√2

∣∣∣∣∣

ϕ 12

ϕ 12

ϕ− 12

ϕ−12

∣∣∣∣∣=

ϕ 12ϕ− 1

2−ϕ−1

2ϕ 1

2√2

. (4.75)

Чтобы отличать эти функции от функций предела LS-связи с учетомспин-орбитального взаимодействия, мы будем опускать вниз звездочкув их обозначениях. Теперь нам осталось найти функции ϕ± 1

2, то есть

решить задачу о сложении моментов l = 1 и s = 12. Заодно мы получим

и функции c j = 32 , для которых введем аналогичные (4.74) обозначения

ψ jz. Исходные функции мультипликативного одноэлектронного базиса

Page 107: КВАНТОВАЯ МЕХАНИКА МОЛЕКУЛ · PDF file2 ББК В22.31 УДК 544.11+539.183 isbn 978-5-94356-719-3 Стась Д.В., Плюснин В.Ф. Квантовая

4.6 ФУНКЦИИ ДЛЯ ТЕРМОВ ПРЕДЕЛА JJ-СВЯЗИ 107

будем, как и раньше, обозначать φ lz · α/β:ψ 3

2= φ+ α,

ψ 12

=

√2φ0 α +φ+ β√

3, ϕ 1

2=

φ0 α −√

2φ+ β√3

,

ψ− 12

=

√2φ0 β +φ−α√

3, ϕ− 1

2=

φ0 β −√

2φ−α√3

,

ψ− 32

= φ−β.

(4.76)

Для искомой функции (4.75) после подстановки и перегруппировкислагаемых получим:

∣∣ 1

2 , 12,0,0

∗ =1√18

φ0φ0(αβ −β α)−√

2(φ0φ−−φ−φ0)αα −

−√

2(φ+φ0 −φ0φ+)β β +2(φ+φ−β α −φ−φ+ αβ )

. (4.77)

Как и раньше, слагаемые здесь сгруппированы в блоки, явным образомантисимметричные относительно перестановки электронов.

Теперь давайте разберемся с функциями термов(3

2, 32

)

0,2. Состояние смаксимально возможной суммарной проекцией Jz = 2, порождающее терм(

32, 3

2

)

2, дается детерминантом Слэйтера Φ(ψ 32,ψ 1

2):

∣∣3

2 , 32,2,2

∗ =ψ 3

2ψ 1

2−ψ 1

2ψ 3

2√2

=

=1√6

√2(φ+φ0 −φ0φ+)αα +φ+φ+(αβ −β α)

. (4.78)

Теперь действием на функцию (4.78) оператора J− = l1− + l2− + s1− + s2−получаем следующие две функции терма:

∣∣ 3

2, 32,2,1

∗ =(φ+φ−−φ−φ+)αα +

√2(φ+φ0 αβ −φ0φ+β α)√6

(4.79)

и затем∣∣ 3

2, 32 ,2,0

∗ =16

2φ0φ0(αβ −β α)+

+√

2(φ0φ−−φ−φ0)αα +√

2(φ+φ0 −φ0φ+)β β + (4.80)

+ 3(φ+φ−αβ −φ−φ+ β α)+(φ+φ− β α −φ−φ+ αβ )

.

Page 108: КВАНТОВАЯ МЕХАНИКА МОЛЕКУЛ · PDF file2 ББК В22.31 УДК 544.11+539.183 isbn 978-5-94356-719-3 Стась Д.В., Плюснин В.Ф. Квантовая

108 ГЛАВА 4. МНОГОЭЛЕКТРОННЫЕ ВОЛНОВЫЕ ФУНКЦИИ

Функции с отрицательной проекцией опять получаются переворотом всехиндивидуальных орбитальных и спиновых проекций:

∣∣ 3

2, 32,2,−1

∗ =(φ−φ+−φ+φ−)β β +

√2(φ−φ0 β α −φ0φ−αβ )√6

, (4.81)

∣∣ 3

2, 32,2,−2

∗ =

√2(φ−φ0 −φ0φ−)β β +φ−φ−(β α −αβ )√

6. (4.82)

Чтобы найти функцию терма(3

2, 32

)

0, выпишем имеющиеся в задаче двадетерминанта Слэйтера с нулевой суммарной проекцией:

Φ(ψ 32,ψ−3

2) =

ψ 32ψ− 3

2−ψ− 3

2ψ 3

2√2

=φ+φ−αβ −φ−φ+ β α√

2, (4.83)

Φ(ψ 12,ψ−1

2) =

1√18

2φ0φ0(αβ −β α)+(φ+φ−β α −φ−φ+ αβ )+

+√

2(φ0φ−−φ−φ0)αα +√

2(φ+φ0 −φ0φ+)β β

(4.84)

и разложим по ним функцию (4.80). Несмотря на то, что полученныенами функции (4.80–4.84) выглядят достаточно громоздко, в их струк-туре видны одинаковые блоки, так что проектирование функции (4.80)на антисимметризованные функции (4.83, 4.84) выполняется достаточнопросто. Так, например,

⟨Φ(ψ 3

2,ψ−3

2)∣∣ 3

2 , 32,2,0

∗ =

=16· 1√

2·3〈φ+φ−αβ −φ−φ+ β α|φ+φ−αβ −φ−φ+ β α〉 =

1√2, (4.85)

все остальные блоки в функции (4.80) дадут нулевой вклад. В итогеполучаем разложение:

∣∣ 3

2 , 32,2,0

∗ =1√2

Φ(ψ 12,ψ−1

2)+

1√2

Φ(ψ 32,ψ−3

2), (4.86)

Page 109: КВАНТОВАЯ МЕХАНИКА МОЛЕКУЛ · PDF file2 ББК В22.31 УДК 544.11+539.183 isbn 978-5-94356-719-3 Стась Д.В., Плюснин В.Ф. Квантовая

4.6 ФУНКЦИИ ДЛЯ ТЕРМОВ ПРЕДЕЛА JJ-СВЯЗИ 109

из которого затем по ортогональности строим разложение для искомойфункции:

∣∣ 3

2 , 32,0,0

∗ =1√2

Φ(ψ 12,ψ−1

2)− 1√

2Φ(ψ 3

2,ψ−3

2). (4.87)

Подставляя теперь обратно выражения для антисимметризованных функ-ций из (4.83, 4.84), получаем для искомой функции полного момента∣∣ 3

2, 32,0,0

∗ окончательное выражение через одноэлектронные функции:

∣∣ 3

2, 32 ,0,0

∗ =16

2φ0φ0(αβ −β α)+

+√

2(φ0φ−−φ−φ0)αα +√

2(φ+φ0 −φ0φ+)β β + (4.88)

− 3(φ+φ−αβ −φ−φ+ β α)+(φ+φ− β α −φ−φ+ αβ )

,

которое отличается от выражения (4.80) только сменой знака у предпос-леднего слагаемого, пришедшего из детерминанта Слэйтера Φ(ψ 3

2,ψ− 3

2).

Отметим, что функции полного момента в пределе j j-связи, вообще го-воря, не совпадают с функциями полного момента в пределе LS-связи стеми же J,Jz, если для данной комбинации J,Jz имеется больше однойвозможности. Так, в нашем случае мы имеем два состояния с J = 0,Jz = 0:функции (4.77) и (4.88) в пределе j j-связи и функции (4.49) и (4.58)в пределе LS-связи. Непосредственная проверка показывает, что междудвумя комплектами функций имеется следующая связь:

|0,0,0,0〉∗ = −√

13

∣∣ 1

2, 12,0,0

∗−√

23

∣∣ 3

2, 32,0,0

∗ ,

|1,1,0,0〉∗ = −√

23

∣∣ 1

2, 12,0,0

∗ +√

13

∣∣3

2 , 32,0,0

∗ .(4.89)

Если бы мы не согласовали фазы внутри мультиплета в (4.48) при записифункции (4.58), мы бы не получили правильного разложения. Получен-ные нами функции попарно образуют базисы в двумерном пространствеJ = 0, Jz = 0 и, как положено, переводятся друг в друга ортогональнымпреобразованием.

И давайте кратко обсудим, как построить функции термов(1

2, 32

)

1,2,порождаемых электронами с разными моментами j1,2. В этом случае за-дача решается просто как задача о сложении моментов, в данном случаеj1 = 3

2 , j2 = 12, и, используя наши обозначения ϕ jz,ψ jz, для мультиплетов

Page 110: КВАНТОВАЯ МЕХАНИКА МОЛЕКУЛ · PDF file2 ББК В22.31 УДК 544.11+539.183 isbn 978-5-94356-719-3 Стась Д.В., Плюснин В.Ф. Квантовая

110 ГЛАВА 4. МНОГОЭЛЕКТРОННЫЕ ВОЛНОВЫЕ ФУНКЦИИ

с согласованным выбором фаз получаем:

|2,2〉 = ψ 32ϕ 1

2,

|2,1〉 =

√3ψ 1

2ϕ 1

2+ψ 3

2ϕ− 1

2

2, |1,1〉 =

ψ 12ϕ 1

2−√

3ψ 32ϕ− 1

2

2,

|2,0〉 =ψ− 1

2ϕ 1

2+ψ 1

2ϕ− 1

2√2

, |1,0〉 =ψ− 1

2ϕ 1

2−ψ 1

2ϕ−1

2√2

,

|2,−1〉 =ψ− 3

2ϕ 1

2+√

3ψ− 12ϕ− 1

2

2, |1,−1〉 =

ψ− 32ϕ 1

2−√

3ψ− 12ϕ− 1

2

2,

|2,−2〉 = ψ− 32ϕ− 1

2.

(4.90)

Однако это еще не есть искомые функции, поскольку они не обладаюттребуемой антисимметричностью относительно перестановок электронов.Так, например, функция |2,2〉 из (4.90) имеет вид «первый электрон всостоянии ψ 3

2, второй электрон в состоянии ϕ 1

2». Нам же вместо «пер-

вый и второй» нужно иметь «один и другой». Для этого нужно каждоепарное произведение вида ψ 3

2ϕ 1

2заменить на построенных на этих же

функциях детерминант Слэйтера вида Φ(ψ 32,ϕ 1

2), и затем подставить

функции ϕ jz,ψ jz через одноэлектронные, используя выражения (4.76).Для функции

∣∣3

2 , 12,2,2

∗ получим:

∣∣ 3

2, 12 ,2,2

∗ = Φ(ψ 32,ϕ 1

2) =

ψ 32ϕ 1

2−ϕ 1

2ψ 3

2√2

=

=1√6

(φ+φ0 −φ0φ+)αα −√

2φ+φ+(αβ −β α)

, (4.91)

причем слагаемые сгруппированы так, чтобы в явном виде показать анти-симметричность функции относительно перестановки электронов. Анало-гичным образом можно получить и остальные функции из набора (4.90)для термов

(12, 3

2

)

1,2, причем реально искать нужно будет только функ-ции с неотрицательными величинами Jz — функции с отрицательнымипроекциями получатся опять просто переворотом всех индивидуальных

Page 111: КВАНТОВАЯ МЕХАНИКА МОЛЕКУЛ · PDF file2 ББК В22.31 УДК 544.11+539.183 isbn 978-5-94356-719-3 Стась Д.В., Плюснин В.Ф. Квантовая

4.6 ФУНКЦИИ ДЛЯ ТЕРМОВ ПРЕДЕЛА JJ-СВЯЗИ 111

проекций в разложении. Итого получаем:

∣∣ 3

2, 12,2,2

∗ =(φ+φ0 −φ0φ+)αα −

√2φ+φ+(αβ −β α)√

6,

∣∣ 3

2, 12,2,1

∗ =

√3(φ+φ0 β α −φ0φ+ αβ )

2√

2+

+φ+φ0 αβ −φ0φ+ β α

2√

6− φ+φ−−φ−φ+

2√

3αα,

∣∣ 3

2, 12,2,0

∗ =φ−φ0 −φ0φ−

2αα +

φ+φ0 −φ0φ+

2β β ,

∣∣ 3

2, 12,2,−1

∗ =

√3(φ−φ0 αβ −φ0φ−β α)

2√

2+

+φ−φ0 β α −φ0φ−αβ

2√

6− φ−φ+−φ+φ−

2√

3β β ,

∣∣ 3

2, 12,2,−2

∗ =(φ−φ0 −φ0φ−)β β −

√2φ−φ−(β α −αβ )√

6,

∣∣ 3

2, 12,1,1

∗ =(φ+φ0 β α −φ0φ+ αβ )

2√

2−

− φ+φ0 αβ −φ0φ+ β α2√

2+

φ+φ−−φ−φ+

2αα,

∣∣ 3

2, 12,1,0

∗ = φ0φ0

√2(β α −αβ )

3+

φ0φ−−φ−φ0

6αα +

+

√2(φ+φ− β α −φ−φ+ αβ )

3+

φ+φ0 −φ0φ+

6β β ,

∣∣ 3

2, 12,1,−1

∗ =(φ−φ0 αβ −φ0φ−β α)

2√

2−

− φ−φ0 β α −φ0φ−αβ2√

2+

φ−φ+−φ+φ−2

β β .

(4.92)

Несмотря на кажущуюся сложность функций (4.92), как впрочем идругих полученных нами функций, процедура их получения полностьюалгоритмизуема, и при аккуратном пошаговом выполнении гарантиро-ванно приводит к получению требуемого результата. С другой стороны,совершенно понятно, что сразу написать такие функции, не проходя избазиса в базис, если и можно, то достаточно сложно.

Page 112: КВАНТОВАЯ МЕХАНИКА МОЛЕКУЛ · PDF file2 ББК В22.31 УДК 544.11+539.183 isbn 978-5-94356-719-3 Стась Д.В., Плюснин В.Ф. Квантовая
Page 113: КВАНТОВАЯ МЕХАНИКА МОЛЕКУЛ · PDF file2 ББК В22.31 УДК 544.11+539.183 isbn 978-5-94356-719-3 Стась Д.В., Плюснин В.Ф. Квантовая

Глава 5

Пространственные

преобразования волновой

функции

Как мы уже не раз обсуждали, при «ручном» анализе волновой функ-ции главную ценность имеют симметрийные соображения и вытекающиеиз них правила отбора. В этой главе мы рассмотрим поведение волновойфункции многоэлектронного атома в пределе LS-связи при пространст-венных преобразованиях — поворотах, отражениях и инверсии, и какестественное следствие из них получим правила нахождения термов ге-тероядерной двухатомной молекулы, образующейся при сведении парыатомов в определенных электронных состояния.

5.1 Пространственные преобразования

изолированного атома

Давайте рассмотрим, какие пространственные преобразования элек-тронной волновой функции допускает изолированный атом. Всего име-ется три типа таких преобразований: поворот на произвольный угол αвокруг оси, проходящей через ядро и задаваемой ортом ~n (Cn(α)), отра-жение в плоскости, проходящей через ядро и задаваемой парой некол-линеарных ортов ~n, ~m (σnm), и инверсия координат в центре атома (I).

Page 114: КВАНТОВАЯ МЕХАНИКА МОЛЕКУЛ · PDF file2 ББК В22.31 УДК 544.11+539.183 isbn 978-5-94356-719-3 Стась Д.В., Плюснин В.Ф. Квантовая

114 ГЛАВА 5. ПРЕОБРАЗОВАНИЯ ВОЛНОВОЙ ФУНКЦИИ

ÊË

Ì

ÊË

Ì

ÊË

Ì

Í Î ÏÐ ÑÒ

Ó

Ô

ÒÓ

Ô

ÒÓ

Ô

Õ

Ö ×

Ø Ù ÚÛ

Ü

Ý

ÛÜ

Ý

ÛÜ

Ý

Рис. 5.1. Связь между операциями отражения σxy, поворота Cz(π) и инверсии I.

Речь пока идет об операциях, затрагивающих только пространственныекоординаты электронов атома.

Указанные три типа преобразований не являются независимыми: какпоказывает рис. 5.1, отражение в плоскости xy сводится к последователь-но выполненному повороту на угол π вокруг оси z и инверсии:

σxy = I ·Cz(π). (5.1)

Двухкратное применение каждой из входящих в (5.1) операций (σ , C(π)и I) опять возвращает систему в исходное состояние, т. е. их квадратыдают единичное преобразование E:

σ2 = C2(π) = I2 = E. (5.2)

Все такие операции второго порядка коммутируют между собой — ихможно выполнять в любой последовательности, и поэтому из (5.1) имеем:

σnm = Ck(π) · I; Ck(π) = σnm · I; I = Ck(π) ·σnm, (5.3)

где

~n, ~m,~k

— тройка любых взаимно перпендикулярных ортов, а опе-

рации в произведениях могут следовать в любом порядке. Поскольку мыхотим описывать поведение волновой функции терма с определенныморбитальным моментом, т. е. с определенным поведением относительновращений, самой удобной на практике оказывается редукция вида (5.1),сводящая отражение в некоторой плоскости к композиции инверсии иповорота вокруг перпендикулярной плоскости оси. Для анализа прост-ранственных преобразований волной функции достаточно рассмотреть

Page 115: КВАНТОВАЯ МЕХАНИКА МОЛЕКУЛ · PDF file2 ББК В22.31 УДК 544.11+539.183 isbn 978-5-94356-719-3 Стась Д.В., Плюснин В.Ф. Квантовая

5.2 ПРЕОБРАЗОВАНИЯ КООРДИНАТ 115

Þ ß

à

á

â

ãä

åæ ç è ç é ä

êë ì êí

ë î í

Рис. 5.2. Операция инверсии.

только поведение ее угловой части, поскольку все такие преобразованияне затрагивают радиальную переменную r, а значит, и радиальную частьволновой функции.

5.2 Преобразования координат

Поскольку волновая функция естественным образом параметризует-ся через сферические координаты, давайте прежде всего найдем законыпреобразования сферических углов ϑ , ϕ при пространственных пре-образованиях, причем мы ограничимся только операциями инверсии I,поворотами на угол π вокруг трех декартовых осей Cx(π), Cy(π), Cz(π), иотражениями в трех декартовых плоскостях σxy, σxz, σyz. Все такие пре-образования удобно строить сначала в декартовых координатах, а затемуже, нарисовав картинку, получать законы преобразования углов ϑ , ϕ.Именно так мы и будем делать, начав с операции инверсии, которую ужебыла кратко рассмотрена на стр. 18:

• Инверсия I.

При инверсии декартовы координаты точки меняют знак:

I: x →−x; y →−y; z →−z. (5.4)

На рис. 5.2 слева преобразование инверсии схематически показанов пространстве. Изображающая точка при инверсии перемещается

Page 116: КВАНТОВАЯ МЕХАНИКА МОЛЕКУЛ · PDF file2 ББК В22.31 УДК 544.11+539.183 isbn 978-5-94356-719-3 Стась Д.В., Плюснин В.Ф. Квантовая

116 ГЛАВА 5. ПРЕОБРАЗОВАНИЯ ВОЛНОВОЙ ФУНКЦИИ

с одного полуконуса, задаваемого углом ϑ , на другой полуконус,задаваемый углом π −ϑ , и смещается на половину окружности поуглу ϕ. Более явно результат преобразования можно увидеть надвух сечениях, приведенных на рис. 5.2 справа: сечение проходя-щей через ось z вертикальной плоскостью σv показывает, как из-меняется угол ϑ , а из проекции на плоскость σxy легче увидеть,что угол ϕ увеличивается на π. Подобные картинки очень полезныи помогают представить себе, что происходит при тех или иныхпреобразованиях координат. Углы ϑ и ϕ ограничены диапазона-ми 0,π и 0,2π, соответственно, и при выходе за их пределыциклически замыкаются. Таким образом, преобразование сферичес-ких координат при инверсии выполняются по закону:

I: ϑ → π −ϑ ; ϕ → ϕ +π. (5.5)

• Поворот Cz(π).

При повороте на угол π вокруг оси z координата z не меняется, акоординаты x и y меняют знаки:

Cz(π): x →−x; y →−y; z → z. (5.6)

Преобразование показано на рис. 5.3 в первом ряду. Как можноувидеть из рисунка, изображающая точка после преобразованияостается на том же полуконусе и смещается на половину окруж-ности по азимутальному углу ϕ:

Cz(π): ϑ → ϑ ; ϕ → ϕ +π. (5.7)

• Поворот Cx(π).

При этом преобразовании не меняется координата x, а y и z меняютзнак:

Cx(π): x → x; y →−y; z →−z. (5.8)

Как показывает второй ряд на рис. 5.3, изображающая точка приэтом переходит с одного полуконуса на другой, а в проекции xyотражается относительно оси x. Результирующее преобразованиедля сферических углов имеет вид:

Cx(π): ϑ → π −ϑ ; ϕ →−ϕ. (5.9)

Page 117: КВАНТОВАЯ МЕХАНИКА МОЛЕКУЛ · PDF file2 ББК В22.31 УДК 544.11+539.183 isbn 978-5-94356-719-3 Стась Д.В., Плюснин В.Ф. Квантовая

5.2 ПРЕОБРАЗОВАНИЯ КООРДИНАТ 117

ï ð

ñ

ò

ó

ôõ

ö÷ ö øù ú û ü û ý õ

þÿ þ

ï ð

ñ

ò

ó

ôõ

ö÷ ý øù ú û ü û ý õ

þþ

ÿ

ï ð

ñ

ò

ó

ôõ

ö÷ õ øù ú û ü û ý õ

þ

ÿ

Рис. 5.3. Операции поворота вокруг трех декартовых осей.

Page 118: КВАНТОВАЯ МЕХАНИКА МОЛЕКУЛ · PDF file2 ББК В22.31 УДК 544.11+539.183 isbn 978-5-94356-719-3 Стась Д.В., Плюснин В.Ф. Квантовая

118 ГЛАВА 5. ПРЕОБРАЗОВАНИЯ ВОЛНОВОЙ ФУНКЦИИ

• Поворот Cy(π).

Имеем для декартовых координат:

Cy(π): x →−x; y → y; z →−z. (5.10)

Изображающая точка при этом меняет полуконус, а в плоскостиxy отражается относительно оси y (третий ряд на рис. 5.3). Здесьзакон преобразования ϕ не столь прозрачен, как в предыдущихслучаях, и чтобы получить его регулярным путем, мы поступимследующим образом. Искомое преобразование должно быть линей-ным по ϕ, поэтому его самый общий вид:

ϕ → aϕ +b, (5.11)

где a и b — неизвестные пока константы, для нахождения которыхнужны два уравнения. Давайте выберем какой-нибудь удобный дляанализа угол, например, ϕ = 0 — как видно из рисунка, при этомпосле преобразования мы должны получить ϕ = π, откуда b = π.В качестве второго реперного угла удобно взять, например, ϕ = π

2 ,который после преобразования перейдет сам в себя: π

2 = a π2 +π, и

отсюда a = −1. Такого рода рассуждения помогают в ситуациях,когда не хватает пространственного воображения, чтобы сразу вы-писать ответ. Итого для закона преобразования сферических угловпри повороте Cy(π) получаем:

Cy(π): ϑ → π −ϑ ; ϕ → π −ϕ. (5.12)

• Отражение σxy.

Это отражение оставляет на месте координаты x, y и меняет знаккоординаты z:

σxy: x → x; y → y; z →−z. (5.13)

Верхний ряд рис. 5.4 показывает, что сферические углы при этомпреобразуются по закону:

σxy: ϑ → π −ϑ ; ϕ → ϕ. (5.14)

Отражение σxy можно представить в виде композиции операцийCz(π) и I, при этом мы должны получить те же самые законы

Page 119: КВАНТОВАЯ МЕХАНИКА МОЛЕКУЛ · PDF file2 ББК В22.31 УДК 544.11+539.183 isbn 978-5-94356-719-3 Стась Д.В., Плюснин В.Ф. Квантовая

5.2 ПРЕОБРАЗОВАНИЯ КООРДИНАТ 119

Рис. 5.4. Операции отражения в трех декартовых плоскостях.

Page 120: КВАНТОВАЯ МЕХАНИКА МОЛЕКУЛ · PDF file2 ББК В22.31 УДК 544.11+539.183 isbn 978-5-94356-719-3 Стась Д.В., Плюснин В.Ф. Квантовая

120 ГЛАВА 5. ПРЕОБРАЗОВАНИЯ ВОЛНОВОЙ ФУНКЦИИ

преобразования координат:

σxy = Cz(π) · I: ϑ I→ π −ϑ Cz(π)→ π −ϑ;

ϕ I→ ϕ +π Cz(π)→ (ϕ +π)+π = ϕ.

(5.15)

Напомним, что операторы в композиции всегда действуют справаналево.

• Отражение σxz.

Для декартовых координат имеем:

σxz: x → x; y →−y; z → z. (5.16)

Для сферических координат получим (второй ряд рис. 5.4):

σxz: ϑ → ϑ ; ϕ →−ϕ. (5.17)

Снова собирая отражение σxz как композицию, имеем:

σxz = Cy(π) · I: ϑ I→ π −ϑCy(π)→ π − (π −ϑ) = ϑ;

ϕ I→ ϕ +πCy(π)→ π − (ϕ +π) = −ϕ.

(5.18)

• Отражение σyz.

Декартовы координаты преобразуются следующим образом:

σyz: x →−x; y → y; z → z. (5.19)

Либо композицией σyz = Cx(π) · I, либо из рис. 5.4 (последний ряд)получим для сферических координат:

σyz: ϑ → ϑ ; ϕ → π −ϕ. (5.20)

5.3 Преобразование сферических гармоник

Имея законы преобразования сферических координат для интересу-ющих нас операций пространственных преобразований, теперь можнопостроить и соответствующие законы преобразования для зависящих от

Page 121: КВАНТОВАЯ МЕХАНИКА МОЛЕКУЛ · PDF file2 ББК В22.31 УДК 544.11+539.183 isbn 978-5-94356-719-3 Стась Д.В., Плюснин В.Ф. Квантовая

5.3 ПРЕОБРАЗОВАНИЕ СФЕРИЧЕСКИХ ГАРМОНИК 121

них сферических гармоник Ylm(ϑ ,ϕ). Таким образом мы получим зако-ны преобразования одноэлектронных волновых функций с определенныммоментом l и его проекцией m |l,m〉. Вспоминая общий вид сферическихгармоник (1.34):

Ylm(ϑ ,ϕ) ∼ sin|m| ϑ P l−|m|g/u (cosϑ)e imϕ , (5.21)

последовательно получаем:

• Инверсия I.

ϑ → π −ϑ ⇒ sinϑ → sinϑ ; cosϑ →−cosϑ;

P l−|m|g/u (cosϑ) → (−1)l−|m|P l−|m|

g/u (cosϑ);

ϕ → ϕ +π ⇒ e imϕ → e im(ϕ+π) = (−1)me imϕ ;

Ylm(ϑ ,ϕ) → (−1)l−|m|+mYlm(ϑ ,ϕ) = (−1)l Ylm(ϑ ,ϕ).

(5.22)

• Поворот Cz(π).

ϑ → ϑ;

ϕ → ϕ +π ⇒ e imϕ → (−1)me imϕ ;

Ylm(ϑ ,ϕ) → (−1)m Ylm(ϑ ,ϕ).

(5.23)

• Поворот Cx(π).

ϑ → π −ϑ ⇒ P l−|m|g/u (cosϑ) → (−1)l−|m|P l−|m|

g/u (cosϑ);

ϕ →−ϕ ⇒ e imϕ → e−imϕ ;

Ylm(ϑ ,ϕ) → (−1)l−|m|Yl,−m(ϑ ,ϕ).

(5.24)

Отметим, что смена знака у ϕ в выражении для преобразованнойфункции означает смену знака у индекса m — поворот на уголπ вокруг оси x, помимо добавления фазового множителя, еще именяет знак проекции момента l на ось z на обратный.

• Поворот Cy(π).

ϑ → π −ϑ ⇒ P l−|m|g/u (cosϑ) → (−1)l−|m|P l−|m|

g/u (cosϑ);

ϕ → π −ϕ ⇒ e imϕ → (−1)m e−imϕ ;

Ylm(ϑ ,ϕ) → (−1)l Yl,−m(ϑ ,ϕ).

(5.25)

Page 122: КВАНТОВАЯ МЕХАНИКА МОЛЕКУЛ · PDF file2 ББК В22.31 УДК 544.11+539.183 isbn 978-5-94356-719-3 Стась Д.В., Плюснин В.Ф. Квантовая

122 ГЛАВА 5. ПРЕОБРАЗОВАНИЯ ВОЛНОВОЙ ФУНКЦИИ

• Отражение σxy.

Операцию отражения здесь можно собрать двумя эквивалентнымиспособами: либо сначала построив полное преобразование коорди-нат и подставив его в выражение для Ylm(ϑ ,ϕ):

ϑ → π −ϑ ⇒ P l−|m|g/u (cosϑ) → (−1)l−|m|P l−|m|

g/u (cosϑ);

ϕ → ϕ;

Ylm(ϑ ,ϕ) → (−1)l−|m|Ylm(ϑ ,ϕ),

(5.26)

либо выполнив композицию операций над Ylm(ϑ ,ϕ):

σxy ·Ylm(ϑ ,ϕ) = Cz(π) · I ·Ylm(ϑ ,ϕ) = (−1)l Cz(π) ·Ylm(ϑ ,ϕ) =

= (−1)l+mYlm(ϑ ,ϕ). (5.27)

Несмотря на формально разный вид, результаты преобразований(5.26) и (5.27) тождественны, поскольку четности выражений l +mи l−|m| совпадают.

• Отражение σxz.

Выполняя композицию операций инверсии и поворота вокруг осиy, получаем:

σxz ·Ylm(ϑ ,ϕ) = Cy(π) · I ·Ylm(ϑ ,ϕ) = (−1)l Cy(π) ·Ylm(ϑ ,ϕ) =

= Yl,−m(ϑ ,ϕ). (5.28)

• Отражение σyz.

σyz ·Ylm(ϑ ,ϕ) = Cx(π) · I ·Ylm(ϑ ,ϕ) = (−1)m Yl,−m(ϑ ,ϕ). (5.29)

Анализируя рассмотренные примеры, можно видеть, что поворот Cz(π)и отражение σxy не меняет знак проекции момента на ось z (индекс m),а повороты Cx(π), Cy(π) и отражения σxz, σyz меняют знак проекциина обратный. Это есть частный случай общей ситуации: поворот вокругнекоторой оси и отражение в перпендикулярной ей плоскости не меняетзнак проекции момента на ось, а отражение в любой содержащей осьплоскости изменяет знак проекции на обратный. В этом можно легкоубедиться, посмотрев в зеркало (рис. 5.5): движение справа налево с

Page 123: КВАНТОВАЯ МЕХАНИКА МОЛЕКУЛ · PDF file2 ББК В22.31 УДК 544.11+539.183 isbn 978-5-94356-719-3 Стась Д.В., Плюснин В.Ф. Квантовая

5.3 ПРЕОБРАЗОВАНИЕ СФЕРИЧЕСКИХ ГАРМОНИК 123

Рис. 5.5. Смена направления вращения при отражении в плоскости, содержащейось вращения.

одной стороны зеркала выглядит как вращение вокруг вертикальной осипо часовой стрелке, а с другой стороны — как вращение против часовойстрелки. А изменение направления вращения и соответствует изменениюзнака проекции момента на ось вращения. Этим обстоятельством мы ипользовались ранее, получая функции полного момента с отрицательнойпроекцией путем смены знака у всех индивидуальных проекций.

Кроме того, из наших примеров видно, что при описании круглогоатома в сферической системе координат ось z оказалась явным образомвыделена. Более того, сравнение выражений (5.28) и (5.29) показывает,что и отражение в двух содержащих ось z декартовых плоскостях σxz

и σyz также оказываются не вполне эквивалентны: хотя оба они изме-няют знак проекции на обратный, выдаваемые ими при этом фазовыемножители равны соответственно 1 и (−1)m и вовсе не совпадают. Отра-

Page 124: КВАНТОВАЯ МЕХАНИКА МОЛЕКУЛ · PDF file2 ББК В22.31 УДК 544.11+539.183 isbn 978-5-94356-719-3 Стась Д.В., Плюснин В.Ф. Квантовая

124 ГЛАВА 5. ПРЕОБРАЗОВАНИЯ ВОЛНОВОЙ ФУНКЦИИ

жение в другой вертикальной плоскости даст, вообще говоря, и другойфазовый множитель. Все это есть прямое следствие выбранного наминесимметричного способа описания круглого атома, с явной привязкойодного из углов (ϑ ) к оси z, а второго (ϕ) — к оси x. Любые физическиосмысленные результаты, конечно, не должны зависеть от выбора осейдля свободного атома. Ситуация здесь в чем-то аналогична необходимос-ти приписывать номера электронам при формальном составлении много-электронной волной функции, которую затем приходится искусственноантисимметризовать, чтобы избавиться от этой привязки. В дальнейшем,если в задаче появится естественное выделенное направление, мы будемвсегда привязывать его к оси z сферической системы координат, а ес-ли потребуется операция отражения в содержащей ось z плоскости, мыбудем для определенности брать отражение σxz.

5.4 Преобразование многоэлектронной

волновой функции

Давайте теперь обсудим законы преобразования многоэлектроннойволновой функции для атома, находящегося в состоянии с определеннымполным орбитальным моментом L и его проекцией M. На первый взглядкажется, что для этого достаточно просто заменить в полученных вышевыражениях одноэлектронные индексы l, m на коллективные L, M, одна-ко это не совсем так. Дело в том, что многоэлектронные функции |L,M〉не есть те же самые функции (5.21) с той же явной зависимостью откоординат.

Проиллюстрировать это можно на следующем простейшем примере.Давайте рассмотрим задачу о сложении двух моментов l = 1. Мы знаем,что в итоге получатся суммарные моменты J = 2,1,0, а самую старшуюфункцию системы можно выписать сразу:

|J = 2,Jz = 2〉 = |1,1〉1|1,1〉2. (5.30)

Вид сферических гармоник Y11, Y22 мы знаем, так что с точностью дочисленных множителей имеем:

|1,1〉1 ∼ sinϑ1 e iϕ1,

|1,1〉2 ∼ sinϑ2 e iϕ2,

|2,2〉 ∼ sin2 ϑ e2iϕ .

(5.31)

Page 125: КВАНТОВАЯ МЕХАНИКА МОЛЕКУЛ · PDF file2 ББК В22.31 УДК 544.11+539.183 isbn 978-5-94356-719-3 Стась Д.В., Плюснин В.Ф. Квантовая

5.4 ПРЕОБРАЗОВАНИЕ МНОГОЭЛЕКТРОННОЙ ФУНКЦИИ 125

Видно, что произведение первых двух функций в (5.31) вовсе не даеттретью. Для пары функций полного момента с проекцией на единицуменьше максимальной аналогично получаем:

|J = 2,Jz = 1〉 ∼ |1,0〉1|1,1〉2+ |1,1〉1|1,0〉2,

|J = 1,Jz = 1〉 ∼ |1,0〉1|1,1〉2−|1,1〉1|1,0〉2,(5.32)

для соответствующих сферических гармоник:

|1,1〉1,2 ∼ sinϑ1,2 e iϕ1,2,

|1,0〉1,2 ∼ cosϑ1,2,

|2,1〉 ∼ sinϑ cosϑ e iϕ ,

|1,1〉 ∼ sinϑ e iϕ ,

(5.33)

и опять подстановка функций из (5.33) вовсе не превращает (5.32) втождества. Более того, даже в этом простом примере не очень понят-но, как превратить четыре независимые переменные ϑ1,ϑ2,ϕ1,ϕ2 в две(ϑ ,ϕ). С математической точки зрения задача о сложении моментовпредставляет из себя задачу о разложении прямого произведения непри-водимых представлений группы непрерывных вращений, базисом кото-рых и служат сферические гармоники, по неприводимым представлениямгруппы вращений, и регулярные процедуры для этого хорошо известны втеории углового момента. Нам же сейчас важно, что в рассматриваемойзадаче о сложении моментов функции полного момента ведут себя какпростые сферические гармоники Ylm(ϑ ,ϕ) только относительно операций

вращения, причем вращения собственного — без отражений.Таким образом, из рассмотренных выше семи примеров операций

пространственных преобразований мы можем пока использовать толькорезультаты для трех операций вращения вокруг декартовых осей. Намже нужны еще операции инверсии и отражений. Получить законы ихдействия можно следующим образом. Мы уже знаем, что многоэлек-тронная волновая функция всегда есть некоторая линейная комбинацияпроизведений одноэлектронных функций, причем в каждое слагаемое та-кой линейной комбинации всегда входит ровно по одной функции откаждого индивидуального момента (см. предыдущую главу). Это даетвозможность найти результат действия на многоэлектронную функциюоперации инверсии I: поскольку пространственная инверсия по опреде-лению есть операция обращения знака всех (декартовых) координат в

Page 126: КВАНТОВАЯ МЕХАНИКА МОЛЕКУЛ · PDF file2 ББК В22.31 УДК 544.11+539.183 isbn 978-5-94356-719-3 Стась Д.В., Плюснин В.Ф. Квантовая

126 ГЛАВА 5. ПРЕОБРАЗОВАНИЯ ВОЛНОВОЙ ФУНКЦИИ

системе, а входящие в полную функцию подсистемы описываются неза-висимыми комплектами координат, действие операции инверсии I прос-то распадается в композицию одноэлектронных операторов инверсии Ik,действующих только на одноэлектронную функцию φq(~rk), зависящую отk-го комплекта координат, каждый из которых в соответствии с (5.22)выдаст (−1)lq . Итого получаем, что

I Φ(φ1,φ2, . . .,φN) = I1 · I2 · . . .· IN (φ1(1)φ2(2) . . .φN(N)+ · · ·) =

= (−1)l1 · (−1)l2 · · · · · (−1)lN φ1(1)φ2(2) . . .φN(N)+ · · ·=

= (−1)∑ li Φ(φ1,φ2, . . .,φN). (5.34)

Сомножители (−1)lq в коэффициентах перед произведениями одноэлек-тронных функций для разных членов линейной комбинации будут идтив разном порядке, но при этом все они дадут один и тот же коэффи-циент (−1)∑ li, где в показателе стоит алгебраическая сумма всех инди-видуальных моментов системы, который можно вынести и опять сгруп-пировать все члены линейной комбинации в исходную функцию. Этотмножитель, который одинаков для всех порождаемых электронной кон-фигурацией состояний, обозначается буквой P (от английского «parity»)и называется четностью электронной конфигурации. Если бы мы попы-тались перенести результаты для операции инверсии с одноэлектронныхфункций — сферических гармоник, мы бы вместо четности P написа-ли (−1)L, где L — одно из возможных значений полного орбитальногомомента системы, т. е. векторной суммы индивидуальных моментов.

Таким образом, для операции инверсии имеем:

I |L,M; l1, l2, . . .lN〉 = P |L,M; l1, l2, . . .lN〉. (5.35)

А теперь, имея операции поворота и инверсии, их композицией можнополучить и оставшиеся операции отражения. Так, например,

σxy |L,M〉= Cz(π) · I |L,M〉= PCz(π) · |L,M〉= P(−1)M |L,M〉. (5.36)

В приведенной ниже таблице суммированы результаты действий всехрассмотренных операций пространственных преобразований O на одно-

Page 127: КВАНТОВАЯ МЕХАНИКА МОЛЕКУЛ · PDF file2 ББК В22.31 УДК 544.11+539.183 isbn 978-5-94356-719-3 Стась Д.В., Плюснин В.Ф. Квантовая

5.5 ТЕРМЫ ДВУХАТОМНОЙ МОЛЕКУЛЫ AB 127

электронные функции |l,m〉 и многоэлектронные функции |L,M〉:

O |l,m〉 |L,M〉

I (−1)l |l,m〉 P |L,M〉

Cz(π) (−1)m |l,m〉 (−1)M |L,M〉

Cx(π) (−1)l+m |l,−m〉 (−1)L+M |L,−M〉

Cy(π) (−1)l |l,−m〉 (−1)L |L,−M〉

σxy (−1)l+m |l,m〉 P(−1)M |L,M〉

σxz |l,−m〉 P(−1)L |L,−M〉σyz (−1)m |l,−m〉 P(−1)L+M |L,−M〉

Подводя итог, подчеркнем еще раз, что законы преобразования волновыхфункций с определенными значениями полного орбитального моментаи его проекции |L,M〉 — функций термов случая LS связи — для опе-раций инверсии, отражений (и не рассмотренных здесь несобственныхвращений) отличаются от законов преобразования сферических гармо-ник Ylm(ϑ ,ϕ). Впрочем, два набора преобразований можно примирить,если договориться, что результатом действия операции инверсии всегдаслужит четность P = (−1)∑ li, в том числе и для обычных сферическихгармоник, где сумма в показателе вырождается в единственное слагаемое— соответствующий гармонике орбитальный момент.

5.5 Термы двухатомной гетероядерной

молекулы

Давайте теперь обсудим, по каким признакам следует классифици-ровать электронные состояния двухатомной молекулы и каким образомее термы связаны с термами образовавших ее атомов. Как и для ато-мов, любая практически полезная классификация термов молекул требу-ет введения упрощающих предположений, и мы рассмотрим только ана-лог предела LS-связи, в котором спин считается свободным и в первом

Page 128: КВАНТОВАЯ МЕХАНИКА МОЛЕКУЛ · PDF file2 ББК В22.31 УДК 544.11+539.183 isbn 978-5-94356-719-3 Стась Д.В., Плюснин В.Ф. Квантовая

128 ГЛАВА 5. ПРЕОБРАЗОВАНИЯ ВОЛНОВОЙ ФУНКЦИИ

приближении не влияет на классификацию по пространственным степе-ням свободы. Кроме того, мы будем рассматривать только состоящие издвух разных атомов, так называемые гетероядерные, молекулы, чтобыпока отвлечься от осложнений, связанных с наличием дополнительногоцентра инверсии в центре молекулы и необходимости классифицироватьэлектронные состояния по отношению к этой операции.

Давайте посмотрим, какие операции пространственных преобразова-ний поддерживает двухатомная гетероядерная молекула, и на основе это-го составим систему классификации ее термов. Напомним, что для ато-ма в приближении LS-связи термы обозначались 2S+1L и различалисьвеличиной полного момента L, сохраняющегося при поддерживаемымикруглым атомом операциями вращения, состояли из 2L + 1 функций содинаковым L и бегущим M, переводимых друг в друга вращениями,и имели также дополнительное вырождение по спину кратности 2S +1,так как спин считался свободным. В двухатомной молекуле сферическаясимметрия понижается до аксиальной — молекула допускает вращениена произвольный угол вокруг своей оси, поэтому теперь ее электронныесостояния нужно классифицировать уже не по орбитальному моменту L,а по его проекции на ось молекулы M. Однако помимо вращения мо-лекула допускает также и отражение в произвольной плоскости σ , про-ходящей через ее ось (рис. 5.6). Как мы уже знаем, такое отражениеменяет знак проекции на противоположный, т. е. переводит состояние спроекцией M в состояние с проекцией −M и наоборот. Ситуация вполнеаналогична переводу состояний с одинаковым L, но разным M друг вдруга при вращениях в свободном атоме. Поэтому здесь пара состоянийс проекциями M и −M также должна быть объединена в один терм,и электронные состояния двухатомной молекулы следует классифици-ровать по модулю проекции орбитального момента на ось молекулы —величине, сохраняющейся при всех поддерживаемых молекулой операци-ях пространственных преобразований. Это квантовое число по аналогиис орбитальным моментом L для атома обозначается прописной греческойбуквой Λ и может принимать любые целые неотрицательные значения.По аналогии с атомными термами, для обозначения термов двухатомноймолекулы также используется буквенная символика, и для обозначениятермов с Λ = 0,1,2, . . . обозначаются прописные греческие буквы, экви-валентные используемым для атомов: Σ (аналог S, Λ = 0), Π (аналог P,Λ = 1), ∆ (аналог D, Λ=2) и т. д.

Далее, поскольку мы работаем в приближении свободного спина,каждый терм оказывается вырожден 2S + 1 раз по спиновой проекции,

Page 129: КВАНТОВАЯ МЕХАНИКА МОЛЕКУЛ · PDF file2 ББК В22.31 УДК 544.11+539.183 isbn 978-5-94356-719-3 Стась Д.В., Плюснин В.Ф. Квантовая

5.6 ТЕРМЫ АТОМОВ И ТЕРМЫ МОЛЕКУЛ 129

Рис. 5.6. Пространственные преобразования, поддерживаемые двухатомной гете-роядерной молекулой.

что привычным образом отражается в символе терма: 2S+1Λ. И наконец,все термы с Λ > 0 двухкратно вырождены и содержат пару функций спроекцией M = ±Λ. Однако в Σ термах имеется всего одна функция, ипоэтому при операциях отражения σ она должна переходить сама в себяс точностью до фазового множителя:

σ ΨΣ = α ΨΣ. (5.37)

Поскольку повторное применение операции отражения опять возвраща-ет систему в исходное состояние, α2 = 1 и α = ±1, поэтому Σ термыдолжны дополнительно классифицироваться по поведению их функцийотносительно отражений на Σ+ и Σ−. Таким образом, термы двухатомнойгетероядерной молекулы с Λ > 0 обозначаются 2S+1Λ и имеют кратностьвырождения (2S+1)по спину×2по коорд., а для Λ = 0 — 2S+1Σ± с кратностьювырождения (2S +1)по спину.

5.6 Соответствие между термами атомов

и образуемых ими молекул

Теперь давайте посмотрим, в каких состояниях можно получить моле-кулу при свободном сближении двух разных атомов A и B, находящихсяв электронных состояниях 2SA+1LA и 2SB+1LB, соответственно. Посколькуспины считаются свободными и складываются независимо, часть задачирешается сразу: для состояний двухатомной молекулы можно по одному

Page 130: КВАНТОВАЯ МЕХАНИКА МОЛЕКУЛ · PDF file2 ББК В22.31 УДК 544.11+539.183 isbn 978-5-94356-719-3 Стась Д.В., Плюснин В.Ф. Квантовая

130 ГЛАВА 5. ПРЕОБРАЗОВАНИЯ ВОЛНОВОЙ ФУНКЦИИ

Рис. 5.7. Образовании двухатомной гетероядерной молекулы AB при свободномсведении разных атомов A и B.

разу получить все спины от SA + SB до |SA − SB| для каждого из орби-тальных состояний. Что же касается орбитальной части классификациитермов, то здесь нам придется построить некоторый аналог процедурысложения моментов.

Рассмотрим процесс сближения наших атомов. Поскольку в задаченет внешних сил, атомы здесь двигаются только под действием сил вза-имного притяжения или отталкивания, направленных от одного атомак другому (нейтральные атомы на больших расстояниях притягивают-ся силами Ван-дер-Ваальса, на малых — отталкиваются). Кроме того,нас сейчас интересует только внутреннее состояние системы — терм ееэлектронной оболочки, поэтому естественным будет отделить внутрен-ние степени свободы, перейдя в систему ее центра масс. Таким образом,с точки зрения классификации термов мы имеем задачу о сближениидвух тел с неподвижным центром масс под действием силы взаимно-го притяжения. В этих условиях тела будут двигаться друг к другу поодной прямой.

Таким образом, теперь в задаче у нас появилось сохраняющееся на-правление — линия сближения атомов, которая затем перейдет в осьмолекулы — а значит, сохраняющаяся проекция полного орбитальногомомента системы на это направление. Кроме того, в задаче имеется иотражение в плоскости, проходящей через ось, т. е. мы имеем весь наборопераций, на основе которых классифицируются электронные состояниямолекулы (рис. 5.7). Теперь можно строить наш аналог процедуры сло-жения моментов.

Page 131: КВАНТОВАЯ МЕХАНИКА МОЛЕКУЛ · PDF file2 ББК В22.31 УДК 544.11+539.183 isbn 978-5-94356-719-3 Стась Д.В., Плюснин В.Ф. Квантовая

5.6 ТЕРМЫ АТОМОВ И ТЕРМЫ МОЛЕКУЛ 131

В качестве исходного базиса для описания орбитального состояниядвухатомной системы удобно взять мультипликативный базис упоря-доченных парных произведений вида ΨA

MAΨB

MB, составленных из волно-

вых функций атомов A и B с определенными проекциями орбиталь-ных моментов MA и MB, соответственно. Всего таких функций будет(2LA + 1)× (2LB + 1) штук, соответственно ровно столько орбитальныхсостояний молекулы мы и должны получить. В качестве оси z выберемнаше выделенное направление — линию сближения атомов, а в качествеотражения возьмем операцию σxz.

Состояние с максимальной полной проекцией M = LA +LB на ось z вмультипликативном базисе можно получить только одним способом, взявсамую старшую функцию ΨA

LAΨB

LBс максимально возможными индиви-

дуальными проекциями MA и MB. Соответственно мы должны получитьи ровно одно состояние с такой проекцией и для базиса молекулярныйсостояний: возникает один терм с Λ = LA + LB. В этот же терм попадети состояние с проекцией M = −(LA + LB), даваемое в исходном базисефункцией ΨA

−LAΨB

−LB. Таким образом, в обсуждаемой процедуре учет от-

рицательных значений полных проекций получается автоматически, инам нужно будет рассмотреть только неотрицательные проекции M.

Следующую проекцию M = LA +LB −1, на единицу меньшую макси-мально возможной, в мультипликативном базисе можно получить ужедвумя способами, взяв функции ΨA

LAΨB

LB−1 и ΨALA−1ΨB

LB. Соответствен-

но и молекулярных состояний с такой проекцией также должно бытьдва. Однако здесь, в отличие от привычной процедуры сложения момен-тов при поиске атомных термов, оба состояния окажутся свободными:в отличие от атомных термов 2S+1L, содержащих по 2L + 1 состоянийс M = −L,−L+1, . . .,L−1,L, в каждом молекулярном терме 2S+1Λ имеет-ся только два орбитальных состояния, с M = Λ и M = −Λ. Поэтому мыполучим два терма с Λ = LA +LB −1.

Продолжая дальше аналогичным образом, мы получим, что при каж-дом уменьшении анализируемой проекции на единицу количество термовс таким Λ также будет увеличиваться на единицу. Так будет продол-жаться до тех пор, пока не будут исчерпаны все состояния атома с мень-шей величиной орбитального момента, количество которых и определяетчисло возможных комбинаций для набора (неотрицательных) проекций.Пусть, например, LB ≤ LA. Тогда мы получим один терм с Λ = LA + LB,два терма с Λ = LA + LB −1, три терма с Λ = LA + LB − 2, . . . , 2LB + 1термов с Λ = LA−LB, 2LB +1 термов с Λ = LA −LB −1, . . . , 2LB +1 термовс Λ = 0. В приведенной ниже таблице перечислены способы, которыми

Page 132: КВАНТОВАЯ МЕХАНИКА МОЛЕКУЛ · PDF file2 ББК В22.31 УДК 544.11+539.183 isbn 978-5-94356-719-3 Стась Д.В., Плюснин В.Ф. Квантовая

132 ГЛАВА 5. ПРЕОБРАЗОВАНИЯ ВОЛНОВОЙ ФУНКЦИИ

можно набрать соответствующие полные проекции:

Λ = LA +LB : (LA;LB)

Λ = LA +LB −1 : (LA;LB −1), (LA −1;LB)

Λ = LA +LB −2 : (LA;LB −2), (LA −1;LB−1), (LA −2;LB)

Λ = LA +LB −3 : (LA;LB −3), (LA −1;LB−2), . . ., (LA −3;LB)︸ ︷︷ ︸

4 состояния· · ·

Λ = LA −LB : (LA;−LB), (LA −1;−LB +1), . . ., (LA −2LB;LB)︸ ︷︷ ︸

2LB +1 состояний

Λ = LA −LB −1 : (LA −1;−LB), (LA −1−1;−LB +1), . . ., (LA −1−2LB;LB)︸ ︷︷ ︸

2LB +1 состояний· · ·

Λ = 0 : (LB;−LB), (LB −1;−LB +1), . . ., (−LB;LB)︸ ︷︷ ︸

2LB +1 состояний

Теперь нам нужно классифицировать Σ термы (состояния с Λ = 0)по поведению относительно операции отражения. Состояния с Λ = 0 по-рождаются функциями вида ΨA

MΨB−M исходного базиса, однако сами они

не будут функциями Σ термов, поскольку не переходят сами в себя приотражении в плоскости:

σxz ΨAMΨB

−M = σAxz ΨA

M ·σBxz ΨB

−M = PA PB (−1)LA (−1)LB ΨA−MΨB

M. (5.38)

Однако из пары функций ΨAMΨB

−M и ΨA−MΨB

M можно составить две ли-нейные комбинации, которые уже будут переводиться отражением самив себя, т. е. будут функциями Σ термов с определенной четностью:

Ψ+ = ΨAMΨB

−M +ΨA−MΨB

M : σxzΨ+ = PA PB (−1)LA (−1)LB Ψ+

Ψ− = ΨAMΨB

−M −ΨA−MΨB

M : σxzΨ− = −PA PB (−1)LA (−1)LB Ψ−(5.39)

Можно проверить, что такой же результат получится и для отраженияв плоскости xy. На самом деле функции Ψ± ведут себя одинаковым об-разом относительно отражения в любой плоскости, содержащей линию

Page 133: КВАНТОВАЯ МЕХАНИКА МОЛЕКУЛ · PDF file2 ББК В22.31 УДК 544.11+539.183 isbn 978-5-94356-719-3 Стась Д.В., Плюснин В.Ф. Квантовая

5.6 ТЕРМЫ АТОМОВ И ТЕРМЫ МОЛЕКУЛ 133

сближения атомов — это и есть тот самый физически осмысленный, независящий от выбора системы координат результат, о котором шла речьвыше, поведение функции с нулевой проекцией относительно отраже-ния в плоскости. Если же попытаться составить аналогичные функциидля состояния с ненулевой проекцией, взяв пары переходящих друг вдруга при отражениях функций ΨA

MAΨB

−MBи ΨA

−MAΨB

MBс MA 6= MB, отра-

жения σxz и σyz дадут формально разный результат:

Ψ∗+ = ΨA

MAΨB

−MB+ΨA

−MAΨB

MB:

σxzΨ∗+ = PA PB (−1)LA (−1)LB Ψ∗

+

σyzΨ∗+ = PA PB (−1)LA (−1)LB (−1)MA−MBΨ∗

+

Ψ∗− = ΨA

MAΨB

−MB−ΨA

−MAΨB

MB:

σxzΨ∗− = −PA PB (−1)LA (−1)LB Ψ∗

σyzΨ∗− = −PA PB (−1)LA (−1)LB (−1)MA−MBΨ∗

−.

(5.40)

Зависимость результата отражения от выбора плоскости отражаетневозможность составить для молекулы одну функцию с ненулевой про-екцией, которая бы вела себя осмысленным образом при отраженияхотносительно плоскости, содержащей ось молекулы — таких функцийвсегда должна быть пара, и именно поэтому термы с Λ > 0 обязательносодержат пару функций.

Отметим, что четность функций (5.39) относительно отражений σопределяется множителем PA PB (−1)LA (−1)LB и вовсе необязательно рав-на +1 и −1 для функций Ψ+ и Ψ−, соответственно. Тем не менее однаиз этих двух функций обязательно будет четной (σΨ = Ψ), а другая —нечетной (σΨ = −Ψ) относительно операции σ . Эта идея о симметри-зации функций, несмотря на свою простоту, оказывается чрезвычайнополезной, и мы еще не раз будем к ней возвращаться: если в задачеимеется некоторая операция O и пара функций φ1,φ2, переводимыхоперацией O друг в друга, то из них всегда можно составить две ин-вариантные относительно операции O линейные комбинации, одна изкоторых будет четной, а другая — нечетной относительно операции O.Эта процедура называется симметризацией базиса φ1,φ2 относитель-но операции O, а получаемые линейные комбинации — приведенным посимметрии базисом относительно операции O.

Page 134: КВАНТОВАЯ МЕХАНИКА МОЛЕКУЛ · PDF file2 ББК В22.31 УДК 544.11+539.183 isbn 978-5-94356-719-3 Стась Д.В., Плюснин В.Ф. Квантовая

134 ГЛАВА 5. ПРЕОБРАЗОВАНИЯ ВОЛНОВОЙ ФУНКЦИИ

В нашей задаче подобную симметризацию относительно отражения σможно провести для всех функций с нулевой проекцией вида (5.39)с 0 < M ≤ LB, т. е. ровно LB раз. Получаем, что из имеющихся 2LB +1 Σтермов LB термов обязательно будут Σ+ термами и LB термов — Σ− терма-ми. Единственной функцией с Λ = 0, для которой такую симметризациюпровести нельзя, оказывается функция ΨA

0ΨB0 , у которой нет пары, но

которая операцией σ переводится сама в себя по общему правилу (5.38):

σ ΨA0ΨB

0 = PA PB (−1)LA (−1)LB ΨA0ΨB

0 . (5.41)

Соответственно последний оставшийся Σ терм будет Σ+ или Σ− в зависи-мости от четности множителя PA PB (−1)LA (−1)LB . Отметим, что Σ термоввсегда нечетное количество и, в отличие от атомных термов, где можнобыло и не получить состояния с L = 0, для молекул всегда имеется хотябы один Σ терм, поскольку нулевую суммарную проекцию при сложениидвух целых моментов можно получить всегда. Если такой терм всегоодин, то его четность определяется по правилу (5.41).

5.7 Термы молекулы FO

Давайте в качестве примера найдем, в каких состояния можно по-лучить молекулу (радикал) FO при ее сведении из атомов кислорода ифтора, находящихся в своих основных состояниях. Прежде всего нахо-дим электронные конфигурации и термы атомов:

F : p5 → p1, осн. терм 2P, кратность вырождения 2×3= 6,O : p4 → p2, осн. терм 3P, кратность вырождения 3×3= 9,

полная кратность вырождения по спину 2×3= 6,по орб. моменту 3×3= 9,

полная кратность вырождения 6×9= 54.

Собираем возможные значения Λ:

Λ = 2 : (1;1)Λ = 1 : (1;0), (0;1)Λ = 0 : (1;−1), (0;0), (−1;1).

(5.42)

Page 135: КВАНТОВАЯ МЕХАНИКА МОЛЕКУЛ · PDF file2 ББК В22.31 УДК 544.11+539.183 isbn 978-5-94356-719-3 Стась Д.В., Плюснин В.Ф. Квантовая

5.7 ТЕРМЫ МОЛЕКУЛЫ FO 135

По спинам имеем:

SF =12, SO = 1⇒ SFO =

12,32

. (5.43)

Итого имеем для обоих возможных значений SFO:

1 ∆ терм (2 состояния),2 Π терма (4 состояния),3 Σ терма (3 состояния),

всего 9 состояний.

(5.44)

Из трех Σ термов один точно будет Σ+ термом и один — Σ− термом.Четность третьего Σ терма находим из (5.41):

(−1)LF = (−1)1 = −1,(−1)LO = (−1)1 = −1,PF = (−1)5 = −1,PO = (−1)4 = 1,

PF PO (−1)LF (−1)LO = −1.

(5.45)

Таким образом, третий Σ терм будет Σ− термом. Итого имеем следую-щие термы: 2,4∆, 2× 2,4Π, 2× 2,4Σ−, 2,4Σ+, которые исчерпывают все 54искомых состояния. Указать среди них основной (если он попал в этотнабор) мы пока не можем — для этого нужно будет строить и заселятьмолекулярные орбитали.

Давайте теперь посмотрим, в каком состоянии нашу молекулу можнополучить при образовании ее не из нейтральных атомов, а из ионов F− иO+ в своих основных состояниях. В этом случае имеем для иона F− элек-тронную конфигурацию p6 и соответственно терм 1S, а для иона O+ —конфигурацию p3 и терм основного состояния 4S. В результате у обра-зующейся молекулы можно получить только один терм, с Λ = 0,S = 3

2,и анализ четностей показывает, что это будет 4Σ− терм. Наши рассуж-дения можно обратить и сказать, что, чтобы при распаде молекулы FOполучить два иона F− и O+ в своих основных электронных состояниях,молекула должна распадаться из состояния 4Σ− — таковы в данном слу-чае получаются симметрийные ограничения для активации именно этогоканала распада.

Page 136: КВАНТОВАЯ МЕХАНИКА МОЛЕКУЛ · PDF file2 ББК В22.31 УДК 544.11+539.183 isbn 978-5-94356-719-3 Стась Д.В., Плюснин В.Ф. Квантовая

136 ГЛАВА 5. ПРЕОБРАЗОВАНИЯ ВОЛНОВОЙ ФУНКЦИИ

И давайте еще посмотрим на один из возможных каналов реакции сучастием возбужденного атома. В качестве примера можно взять атомкислорода в высореакционноспособном состоянии 1D:

O(1D

)+F

(2P)→ FO. (5.46)

По спину снова имеется единственная возможность SFO = 12. Для класси-

фикации по Λ строим аналог таблицы (5.42), указывая первым проекциюорбитального момента для атома кислорода MO:

Λ = 3 : (2;1)Λ = 2 : (2;0), (1;1)Λ = 1 : (2;−1), (1;0), (0;1)Λ = 0 : (1;−1), (0;0), (−1;1).

(5.47)

Получаем один 2Φ терм, два 2∆ терма, три 2Π терма и три 2Σ терма,один из которых точно 2Σ+, второй — 2Σ−. Четность третьего 2Σ термаотносительно отражения в плоскости определяем по правилу (5.41):

PF PO (−1)LF (−1)LO = (−1)5 (−1)4 (−1)1 (−1)2 = 1. (5.48)

Третий 2Σ терм в этом случае будет 2Σ+ термом. Итого получаем, чтодля реакции (5.46) можно получить следующие термы молекулы FO: 2Φ,2× 2∆, 3× 2Π, 2× 2Σ+, 2Σ−.

Поскольку пока рассматривались только симметрийные ограничения,мы можем говорить только о принципиальной возможности протеканияреакции по тому или иному каналу, реальные же вероятности каждо-го канала будут определяться энергетическими и другими факторами,которые в этом анализе никак не фигурируют. Тем не менее и такаяинформация может оказаться очень полезной: так, например, если средипродуктов реакции атомов кислорода и фтора будет обнаружена молеку-ла FO

(2Φ), то можно с уверенностью сказать, что по крайней один из

атомов-реагентов был в возбужденном и/или ионизованном состоянии,поскольку нейтральные атомы в основных состояниях такой терм датьне могут, и с разумной долей вероятности предположить, что реакцияпрошла по каналу (5.46), хотя последнее требует независимой проверки.

5.8 Учет спин-орбитального взаимодействия

Так же как и в атоме, при наличии у двухатомной молекулы ненулево-го спина возникает дополнительное взаимодействие, связывающее между

Page 137: КВАНТОВАЯ МЕХАНИКА МОЛЕКУЛ · PDF file2 ББК В22.31 УДК 544.11+539.183 isbn 978-5-94356-719-3 Стась Д.В., Плюснин В.Ф. Квантовая

5.8 УЧЕТ СПИН-ОРБИТАЛЬНОГО ВЗАИМОДЕЙСТВИЯ 137

собой спиновые и орбитальные степени свободы системы электронов. Всферически симметричном атоме спин-орбитальное взаимодействие при-водило к расщеплению термов предела LS-связи по величине полногомомента J. В аксиально симметричной молекуле орбитальный моментэлектронной оболочки эффективно уложен на выделенное направление —ось молекулы, и спин-орбитальное взаимодействие здесь превращается втак называемое взаимодействие спин-ось. В атоме классификация состо-яний по полному моменту проводилась в предположении малости спин-орбитального взаимодействия по сравнению с межэлектронными, т. е.малости расщеплений по J (интервалами в мультиплетном расщепле-нии) по сравнению с расстояниями между термами с разным полныморбитальным моментом L. В молекуле аналогичным приближением будетмалость всех расщеплений по сравнением с расстояниями между терма-ми с разной величиной Λ, однако ситуация здесь осложняется наличиемдополнительных степеней свободы, в частности, возможностью враще-ния молекулы как целого, также несущего момент количества движения,поэтому приходится вводить дополнительные оговорки и ограничения.Мы здесь обсудим только простейший случай, так называемый случай aпо Гунду, когда спин-орбитальное взаимодействие считается слабым посравнению с межэлектронным, но при этом его энергия много большехарактерных расстояний между вращательными уровнями. В таком при-ближении о вращении молекулы можно пока просто забыть и вести клас-сификацию при неподвижных ядрах.

В этом случае в задаче фигурируют две проекции на ось молекулы:проекция орбитального момента M, равная ±Λ, и проекция спина, пробе-гающая значения −S,−S+1, . . .,S−1,S и обозначаемая Σ. Поскольку речьидет о классификации внутренних состояний молекул, в качестве поло-жительного направления оси удобно выбрать направление орбитальногомомента молекулы, так что для орбитальной проекции остается единст-венная возможность M = Λ. При учете взаимодействия спин-ось состоя-ния молекулы расщепятся по величине полной проекции момента на осьмолекулы Ω = Λ + Σ, которая пробегает ряд значений от Λ + S до Λ− Sчерез единицу, не симметричный относительно нуля.

В нашем последнем примере с молекулой FO при учете слабого взаи-модействия спин-ось в рамках приближений случая a по Гунду получимследующие термы: терм 2Φ расщепится на термы 2Φ5/2,

2Φ7/2, терм2∆ —

на термы 2∆3/2,2∆5/2, терм 2Π — на 2Π1/2,

2Π3/2, термы 2Σ+, 2Σ− нерасщепятся. Термы 4Π и 4∆, возможные при образовании молекулы изнейтральных атомов в основных состояниях, расщепятся на термы 4Π−1/2,

Page 138: КВАНТОВАЯ МЕХАНИКА МОЛЕКУЛ · PDF file2 ББК В22.31 УДК 544.11+539.183 isbn 978-5-94356-719-3 Стась Д.В., Плюснин В.Ф. Квантовая

138 ГЛАВА 5. ПРЕОБРАЗОВАНИЯ ВОЛНОВОЙ ФУНКЦИИ

4Π1/2,4Π3/2,

4Π5/2 и 4∆1/2,4∆3/2,

4∆5/2,4∆7/2, соответственно, в каждом

из которых будет по два состояния.При увеличении расстояния между ядрами взаимодействие атомов

ослабляется и рано или поздно становится меньше спин-орбитальноговзаимодействия в атомах. Поэтому корректное рассмотрение соответст-вия между электронными термами молекулы и состояниями атомов набольших расстояниях при наличии ненулевого спин-орбитального вза-имодействия выполняется в два этапа и требует рассмотрения другогопредельного случая для иерархии электронных взаимодействий в моле-куле, так называемого случая c по Гунду, а также поиска соответствиямолекулярных термов двух разных предельных случаев. Пример такогоанализа можно найти в работе1. И если первая часть задачи решаетсядостаточно просто, то рассмотрение связи молекулярных термов случа-ев a и c по Гунду требует привлечения дополнительной информации опорядке расположения термов в молекуле. Мы обсудим только связь мо-лекулярных термов с атомными при доминировании спин-орбитальноговзаимодействия, когда атомные и молекулярные состояния описываютсяединственным «хорошим» квантовым числом, полным моментом J и егопроекцией на ось молекулы Ω, соответственно. Как и для числа Λ, вэтом случае классификация ведется по модулю Ω.

Задача о соответствии между Ω для состояний молекулы и J1,2 длясоставляющих ее атомов в этом случая оказывается аналогичной задачео нахождении связи между Λ и орбитальными моментами L1,2. Термымолекулы в этом случае обозначают просто величиной |Ω|, например, 1,и при |Ω| > 0 двухкратно вырождены. При Ω = 0 термы невырождены идополнительно классифицируются по четности относительно отраженийв плоскости как 0±, аналогично рассмотренным ранее Σ термам.

Все основные результаты можно получить, практически повторяя рас-суждения со стр. 129-134 с заменой орбитальных моментов L1,2 на полныемоменты J1,2 и величины Λ на |Ω|. Возможные значения |Ω| при |Ω|> 0находятся через проекции полных моментов атомов как |Ω| = |J1z + J2z |,состояния с |Ω| = 0 можно получить либо при J1z = −J2z 6= 0, либопри J1z = J2z = 0. В первом случае всегда получается пара термов 0+

и 0−, а для комбинации J1z = −J2z 6= 0 четность 0-терма дается факто-ром (−1)J1+J2+∑i l1i+∑ j l2 j , в котором опять узнается коэффициент из (5.41).Отличие заключается только в том, что, в отличие от орбитальных мо-ментов L1,2, полные моменты J1,2, а с ними и сумма J1 + J2, могут быть

1R. S. Mulliken, Phys. Rev. 36, 1440 (1930).

Page 139: КВАНТОВАЯ МЕХАНИКА МОЛЕКУЛ · PDF file2 ББК В22.31 УДК 544.11+539.183 isbn 978-5-94356-719-3 Стась Д.В., Плюснин В.Ф. Квантовая

5.8 УЧЕТ СПИН-ОРБИТАЛЬНОГО ВЗАИМОДЕЙСТВИЯ 139

полуцелыми. Итого при J1 ≥ J2 можно получить следующие термы |Ω|:

J1 + J2,J1+ J2−1,J1 + J2−2, . . . , 12 или 0+

J1 + J2−1,J1 + J2−2, . . . , 12 или 0−

. . .

J1− J2, . . . , 12 или 0±

(5.49)

При полуцелой сумме J1 + J2 наименьшее возможное значение |Ω|равно 1

2, и нулевых термов не будет вовсе. Если J1 и J2 оба полуцелые,то число нулевых термов (равное 2J2+1) будет четным, и термов 0+ и 0−

получится равное количество. Если же моменты J1 и J2 оба целые, мыполучим нечетное число нулевых термов, при этом J2 из них заведомобудут 0+ термами, J2 — 0− термами, а четность оставшегося терма даетсяфактором (−1)J1+J2 P1P2.

В качестве примера давайте найдем термы молекулы CsI, которыеможно получить при сведении атомов цезия и йода в основных состояни-ях. Атомы можно описывать в приближении как LS-, так и j j-связи, приэтом имеем термы 2S1/2/

(12

)

1/2 для Cs (один s электрон) и 2P3/2/(

32

)

3/2

для I (одна p дырка), дающие состояния 2, 1, 1, 0+, 0− молекулы CsI.

Page 140: КВАНТОВАЯ МЕХАНИКА МОЛЕКУЛ · PDF file2 ББК В22.31 УДК 544.11+539.183 isbn 978-5-94356-719-3 Стась Д.В., Плюснин В.Ф. Квантовая
Page 141: КВАНТОВАЯ МЕХАНИКА МОЛЕКУЛ · PDF file2 ББК В22.31 УДК 544.11+539.183 isbn 978-5-94356-719-3 Стась Д.В., Плюснин В.Ф. Квантовая

Глава 6

Термы двухатомной

гомоядерной молекулы

В двухатомной гомоядерной молекуле AA к уже имеющимся операци-ям пространственных преобразований — вращению вокруг оси и отра-жению в содержащей ось плоскости — добавляется еще одна: инверсияв центре соединяющего атом отрезка. Соответственно теперь состояниямолекулы должны классифицироваться по отношению к этой операции, аправила связи атомных термов с молекулярными должны быть уточнены.Именно этим мы сейчас и займемся.

6.1 Инверсия в центре молекулы

На рис. 6.1 схематически показан результат действия новой операциив двухатомной молекуле, состоящей из одинаковых атомов A, которые мывпредь будем обозначать малыми буквами a и b. Операцию инверсии вцентре соединяющего атомы отрезка ab будем обозначать малой буквой i,чтобы не путать ее с операциями инверсии в центрах атомов a и b, длякоторых мы сохраним обозначения Ia и Ib, соответственно. Посколькумолекула под действием операции i переходит сама в себя, с формальнойточки зрения ее гамильтониан коммутирует с оператором i, а это значит,что собственные функции гамильтониана — функции термов молекулы —можно одновременно сделать и собственными функциями оператора ин-версии. Другими словами, функции термов можно и нужно выбрать с

Page 142: КВАНТОВАЯ МЕХАНИКА МОЛЕКУЛ · PDF file2 ББК В22.31 УДК 544.11+539.183 isbn 978-5-94356-719-3 Стась Д.В., Плюснин В.Ф. Квантовая

142 ГЛАВА 6. ТЕРМЫ ГОМОЯДЕРНОЙ МОЛЕКУЛЫ

Рис. 6.1. Операция инверсии в центре соединяющего атомы отрезка i.

определенным поведением относительно операции i:

iψg = +ψg; iψu = −ψu. (6.1)

В (6.1) использована стандартная нотация для обозначения четности Pi

относительно операции i: четные состояния получают индекс g (от немец-кого gerade — четный), а нечетные — индекс u (ungerade — нечетный).Соответственно и термы двухатомной гомоядерной молекулы с Λ > 0 обо-значаются 2S+1Λg/u , а термы с Λ = 0 получают символ 2S+1Σ±

g/u, описываю-щий поведение электронной волновой функции терма относительно всехдопустимых молекулой операций пространственных преобразований: по-ворота вокруг оси (Σ), отражения в плоскости (±) и инверсии (g/u).

Давайте теперь проанализируем, каким образом исходные состоянияатомов соотносятся с состояниями сводимой из них молекулы, причем мыпока ограничимся случаем одинаковых атомов в полностью одинаковыхэлектронных состояниях, т. е. имеющих, помимо одинаковых ядер, оди-наковые электронные конфигурации и находящихся в одинаковых термахэтих конфигураций, в полном пренебрежении спин-орбитальными взаи-модействиями. Впервые эту задачи рассмотрели в 1928 г. Ю. Вигнер иЕ. Витмер1 , и полученные ими правила называют правилами Вигнера—Витмера. Мы будем следовать значительно более простым рассуждениям,

1E. Wigner and E. E. Witmer, Zeits. f. Physik 51, 883 (1928).

Page 143: КВАНТОВАЯ МЕХАНИКА МОЛЕКУЛ · PDF file2 ББК В22.31 УДК 544.11+539.183 isbn 978-5-94356-719-3 Стась Д.В., Плюснин В.Ф. Квантовая

6.2 ДЕЙСТВИЕ ИНВЕРСИИ НА ВОЛНОВУЮ ФУНКЦИЮ 143

предложенным в 1972 г. Р. Заре и П. Печукасом2 . Аналогичную задачудля атомов со значительной спин-орбитальной связью, например, тяже-лых галогенов, рассмотрел в 1930 г. Р. Малликен3.

Задача решается в целом как рассмотренная в предыдущей главеаналогичная задача о гетероядерной молекуле AB, однако теперь ещепоявляется необходимость классифицировать состояния образующейсямолекулы по поведению относительно операции инверсии i, и здесь иначинаются главные трудности. Как и для гетероядерной молекулы, взадаче имеются все требуемые для классификации элементы симмет-рии: выделенная ось (линия сближения, переходящая в ось молекулы),проходящая через нее плоскость и находящийся в центре масс центр ин-версии. Используя их, нам нужно будет построить соответствие междусостояниями пары изолированных атомов и термами образующейся приих сведении молекулы.

6.2 Действие операции инверсии на волновую

функцию системы

В качестве исходного базиса для описания системы здесь опять удоб-но взять произведения многоэлектронных функций атомов a и b, имею-щих определенные значения орбитального момента L, спина s, а такжеих проекций на линию сближения атомов M и m. Будем считать, что накаждом атоме находится по N электронов, причем электроны с услов-ными номерами от 1 до N находятся на атоме a, а с номерами от N +1до 2N — на атоме b.

Поскольку мы собираемся делать преобразования в реальном прост-ранстве, давайте явно разобьем орбитальные и спиновые переменные изапишем коллективную волновую функцию системы в виде

Ψ = Φsa,ma(1,2, . . .,N;La,Ma)a ·Φsb,mb(N +1,N +2, . . .,2N;Lb,Mb)b. (6.2)

В этой записи функции Φ()a и Φ()b центрированы на атомах a и b, соот-ветственно, содержат перечисленные в скобках электроны, которые даюторбитальные состояния с моментами La/b и их проекциями Ma/b, соот-ветственно, а спиновые переменные — спины sa/b и их проекции ma/b —вынесены в верхние индексы. Поскольку речь идет об атомах в одинако-вых состояниях, обязательно имеем sa = sb = s и La = Lb = L, и эти (не

2P. Pechukas and R. N. Zare, Am. J. Phys. 40, 1687 (1972).3R. S. Mulliken, Phys. Rev. 36, 1440 (1930).

Page 144: КВАНТОВАЯ МЕХАНИКА МОЛЕКУЛ · PDF file2 ББК В22.31 УДК 544.11+539.183 isbn 978-5-94356-719-3 Стась Д.В., Плюснин В.Ф. Квантовая

144 ГЛАВА 6. ТЕРМЫ ГОМОЯДЕРНОЙ МОЛЕКУЛЫ

Рис. 6.2. Представление операции инверсии в центре соединяющего два атомаотрезка ia как композиции инверсии в центре атома Ia и трансляции на межа-

томное расстояние τab.

меняющиеся) индексы можно опустить из достаточно громоздкой запи-си (6.2), что дает

Ψ = Φma(1,2, . . .,N;Ma)a ·Φmb(N +1,N +2, . . .,2N;Mb)b. (6.3)

Поскольку функция (6.2) записана через одноцентровые функции Φ()a

и Φ()b, было бы очень удобно и операцию инверсии представить в видекомпозиции операций, простым образом действующих на одноцентро-вые функции. Во-первых, поскольку центрированные на атомах a и bфункции описываются независимыми комплектами переменных, опера-ция i распадается в композицию операций ia и ib, действующих толькона пространственные переменные функций Φ()a и Φ()b, соответственно:

i = ia · ib. (6.4)

Во-вторых, как показывает рис. 6.2, одноцентровую операцию инверсии,например, ia, можно представить в виде композиции обычной операцииинверсии в центре атома Ia и трансляции электронов в пространстве навектор ab τab:

ia = τab · Ia; ib = τba · Ib. (6.5)

Операторы Ia, Ib, действуя на соответствующие функции Φ()a и Φ()b,выдадут четности электронных состояний для центров a и b:

Ia Φ()a = PaΦ()a; Ib Φ()b = PbΦ()b, (6.6)

Page 145: КВАНТОВАЯ МЕХАНИКА МОЛЕКУЛ · PDF file2 ББК В22.31 УДК 544.11+539.183 isbn 978-5-94356-719-3 Стась Д.В., Плюснин В.Ф. Квантовая

6.2 ДЕЙСТВИЕ ИНВЕРСИИ НА ВОЛНОВУЮ ФУНКЦИЮ 145

а оператор трансляции переместит в пространстве электроны, а с ними иорбитальные состояния, с центра a на центр b и обратно, не затрагиваяспиновых переменных:

τab Φma(1,2, . . .,N;Ma)a = Φma(1,2, . . .,N;Ma)b,

τba Φmb(N +1,N +2, . . .,2N;Mb)b = Φmb(N +1,N +2, . . .,2N;Mb)a.(6.7)

Итого операция инверсии i действует на определенную в (6.3) функциюследующим образом:

i Ψ = τab Ia Φma(1,2, . . .,N;Ma)a τba Ib Φmb(N +1,N +2, . . .,2N;Mb)b =

= PaPb Φma(1,2, . . .,N;Ma)b Φmb(N +1,N +2, . . .,2N;Mb)a. (6.8)

Электроны оказались перемещены в пространстве на другой центр, ис ними же переместились и орбитальные состояния атомов. Получаем,что в результате применения операции инверсии i атомы a и b обменя-лись своими орбитальными электронными состояниями, а коллективнаяволновая функция системы умножилась на коэффициент PaPb. Однакопоскольку речь идет об одинаковых атомах в одинаковых состояниях,Pa = Pb = P, а так как P2 = 1, фазовый множитель оказывается равнымединице. Вводя для функции (6.3) сокращенное обозначение

Φma(1, . . .,N;Ma)a Φmb(N +1, . . .,2N;Mb)b ≡ Ψ(Ma,ma;Mb,mb), (6.9)

имеем:i Ψ(Ma,ma;Mb,mb) = Ψ(Mb,ma;Ma,mb), (6.10)

т. е. чистый результат операции инверсии для коллективной волновойфункции заключается в обмене орбитальных состояний двух центров aи b при сохранении их спиновых состояний.

На результат применения операции инверсии i можно посмотретьи несколько другим образом. Вспомним, что электроны в правильносимметризованной многоэлектронной волновой функции всегда можнопопарно переставлять между собой, при этом волновая функция будетпросто менять знак при каждой перестановке. Пользуясь этим и сде-лав в функции (6.8) N парных перестановок, мы вернем все электронына свои формально прежние места и получив в итоге множитель (−1)N .Здесь важно, что при такой перестановке обмениваются не только прост-ранственные, но и спиновые переменные электронов. Продолжая преоб-

Page 146: КВАНТОВАЯ МЕХАНИКА МОЛЕКУЛ · PDF file2 ББК В22.31 УДК 544.11+539.183 isbn 978-5-94356-719-3 Стась Д.В., Плюснин В.Ф. Квантовая

146 ГЛАВА 6. ТЕРМЫ ГОМОЯДЕРНОЙ МОЛЕКУЛЫ

разование (6.8), получаем:

i Ψ = τab Ia Φma(1,2, . . .,N;Ma)a τba Ib Φmb(N +1,N +2, . . .,2N;Mb)b =

= Φma(1,2, . . .,N;Ma)b Φmb(N +1,N +2, . . .,2N;Mb)a =

= (−1)NΦmb(1,2, . . .,N;Ma)a Φma(N +1,N +2, . . .,2N;Mb)b. (6.11)

Сравнивая исходную функцию с преобразованной, можно заметить, что,помимо возникшего в (6.11) фазового множителя (−1)N , отличаются онитолько перестановкой спиновых переменных: функции Φma()a и Φmb()b

обменялись спиновыми индексами, превратившись в Φmb()a и Φma()b,соответственно — инверсия в физическом пространстве оказалась экви-валентной перестановке спиновых переменных. Заметим также, что чет-ность числа N совпадает с четностью числа 2s, поскольку при четном N(четном числе электронов) суммарный спин атома обязательно будетцелым, а при нечетном N — полуцелым. Принимая это во вниманиеи опять обращаясь к сокращенным обозначениям (6.9), получаем второепредставление для преобразования инверсии:

i Ψ(Ma,ma;Mb,mb) = (−1)2sΨ(Ma,mb;Mb,ma). (6.12)

Итого получаем, что действие операции пространственной инверсии вцентре масс нашей системы приводит к обмену спиновых переменныхв коллективной волновой функции и умножению ее на фазовый мно-житель (−1)2s, не затрагивая пространственных переменных — такимнесколько странным образом спин в данной задаче оказался сцепленнымс физическим пространством. Обезличивая переменные Ma, Mb, ma, mb,окончательно имеем два представления операции инверсии:

i Ψ(M1,m1;M2,m2) = Ψ(M2,m1;M1,m2), (6.13)

i Ψ(M1,m1;M2,m2) = (−1)2s Ψ(M1,m2;M2,m1). (6.14)

Жирным шрифтом здесь выделены переставляемые переменные.

6.3 Пространства состояний с нечетными ΛКак мы видели в предыдущей главе, при сведении атомов с моментами

La и Lb в двухатомную молекулу можно получить молекулярные термы

Page 147: КВАНТОВАЯ МЕХАНИКА МОЛЕКУЛ · PDF file2 ББК В22.31 УДК 544.11+539.183 isbn 978-5-94356-719-3 Стась Д.В., Плюснин В.Ф. Квантовая

6.3 ПРОСТРАНСТВА СОСТОЯНИЙ С НЕЧЕТНЫМИ Λ 147

с Λ от La +Lb до 0. В нашей ситуации одинаковых атомов в одинаковыхсостояниях мы имеем La = Lb = L, так что возможны следующие Λ:

Λ = 2L − 1 разΛ = 2L−1 − 2 разаΛ = 2L−2 − 3 раза

· · ·Λ = 0 − 2L+1 раз

(6.15)

Состояний с четным Λ (Λ = 2L,2L−2, . . .,0) всегда оказывается нечетноеколичество, а состояний с нечетным Λ (Λ = 2L−1,2L−3, . . .,1) — всегдачетное количество. С анализа последних мы и начнем.

Давайте посмотрим, каким образом в нашей системе можно набратьнечетные значения Λ:

Λ = 2L−1 : (L;L−1), (L−1;L)

Λ = 2L−3 : (L;L−3), (L−3;L)(L−1;L−2), (L−2;L−1)

Λ = 2L−5 : (L;L−5), (L−5;L)(L−1;L−4), (L−4;L−1)(L−2;L−3), (L−3;L−2)

· · ·

(6.16)

В скобках ( ; ) первой записана проекция орбитального момента дляатома a, второй — для атома b. Напомним, что все термы с Λ > 0 двух-кратно вырождены и содержат «зеркальные» состояния с тем же набо-ром проекций с перевернутыми знаками. Видно, что для нечетных Λорбитальные состояния всегда идут парами, которые отличаются друг отдруга только перестановкой (Mα ;Mβ) ↔ (Mβ ;Mα). Другими словами, всеорбитальные состояния здесь можно индексировать неупорядоченнымипарами Mα ;Mβ с Mα 6= Mβ :

Λ = 2L−1 : L;L−1Λ = 2L−3 : L;L−3,L−1;L−2Λ = 2L−5 : L;L−5,L−1;L−4,L−2;L−3

· · ·

(6.17)

Для каждого орбитального состояния атомов у нас имеется полныйкомплект из 2s+1 спиновых состояний, так что для пары Mα ;Mβ име-ется целое пространство из 2× (2s+1)2 состояний — (2s+1)2 пар вида

Page 148: КВАНТОВАЯ МЕХАНИКА МОЛЕКУЛ · PDF file2 ББК В22.31 УДК 544.11+539.183 isbn 978-5-94356-719-3 Стась Д.В., Плюснин В.Ф. Квантовая

148 ГЛАВА 6. ТЕРМЫ ГОМОЯДЕРНОЙ МОЛЕКУЛЫ

Ψαβ = Ψ(Mα ,m1;Mβ ,m2) и Ψβ α = Ψ(Mβ ,m1;Mα ,m2), где индексы m1 и m2

независимо пробегают все возможные 2s+1 значений от s до −s.Давайте посмотрим, каким образом операция инверсии i действует

на пару функций Ψαβ и Ψβ α . Беря более удобное здесь представление(6.13), имеем:

i Ψαβ = i Ψ(Mα ,m1;Mβ ,m2) = Ψ(Mβ ,m1;Mα,m2) = Ψβ α ,

i Ψβ α = i Ψ(Mβ ,m1;Mα,m2) = Ψ(Mα ,m1;Mβ ,m2) = Ψαβ .(6.18)

Получаем, что в каждой паре функции Ψαβ и Ψβ α переводятся друг вдруга операцией i, а значит, их можно симметризовать относительно i:

i (Ψαβ +Ψβ α) = Ψαβ +Ψβ α ,

i (Ψαβ −Ψβ α) = −(Ψαβ −Ψβ α),(6.19)

для каждого для каждого из (2s + 1)2 наборов спиновых переменных(m1,m2). Таким образом, для каждой неупорядоченной пары Mα ;Mβс Mα 6= Mβ имеющееся пространство из 2× (2s + 1)2 состояний разби-вается на два подпространства из (2s + 1)2 состояний каждое, одно изкоторых состоит только из четных относительно операции инверсии iфункций вида Ψαβ +Ψβ α , второе — только из функций вида Ψαβ −Ψβ α ,нечетных относительно i, причем в каждом из подпространств спино-вые переменные m1 и m2 независимо пробегают все 2s + 1 возможныхзначений. Это означает, что каждое пространство функций, помеченныхфлажком Mα ;Mβ, разбивается в свою очередь на два подпространства,помеченных флажками g/u, в каждом из которых уже меняются толькоспиновые переменные, давая теперь обычную задачу на сложение мо-ментов (рис. 6.3). В результате в каждом из подпространств Mα ;Mβg/uмы ровно по одному разу получим все формально возможные значениясуммарного спина S от 2s до 0 через единицу. Отсюда следует первое

правило Вигнера—Витмера:

• для гомоядерных молекул, получаемых при сведении одинаковыхатомов в одинаковых состояниях, возможные значения Λ и S на-ходятся как для гетероядерных молекул, при этом для нечетных Λвсегда получается четное число термов, ровно половина из которыхбудет g термами и ровно половина — u термами для каждого извозможных значений суммарного спина 0,1, . . .,2s.

Page 149: КВАНТОВАЯ МЕХАНИКА МОЛЕКУЛ · PDF file2 ББК В22.31 УДК 544.11+539.183 isbn 978-5-94356-719-3 Стась Д.В., Плюснин В.Ф. Квантовая

6.4 ПРОСТРАНСТВА СОСТОЯНИЙ С ЧЕТНЫМИ Λ 149

! " #! $ %&

! " #! $ %'

(

) *) *) +) +) ,

) ,)- )-

(

Рис. 6.3. Разбиение пространства Mα ;Mβ на подпространства Mα ;Mβ g/u изатем на подпространства с определенным значением полного спина.

6.4 Пространства состояний с четными ΛДавайте теперь посмотрим, каким образом здесь можно получить чет-

ные значения Λ:

Λ = 2L : (L;L),

Λ = 2L−2 : (L;L−2), (L−2;L)(L−1;L−1)

Λ = 2L−4 : (L;L−4), (L−4;L)(L−1;L−3), (L−3;L−1)(L−2;L−2)

· · ·

(6.20)

Видно, что для каждого четного значения Λ получается нечетное числоорбитальных состояний, причем все они опять разбиваются на неупо-рядоченные пары Mα ;Mβ с Mα 6= Mβ , за исключением единственнойсимметричной комбинации вида [M;M], которая у нас осталась без пары.Кроме того, для каждого орбитального состояния опять имеется полныйкомплект по спиновым переменным. Неупорядоченные пары Mα ;Mβобрабатываются симметризацией по орбитальной проекции точно также, как для уже рассмотренного случая нечетных Λ, и дают прежнийрезультат: равное количество g и u термов для каждого из возможных

Page 150: КВАНТОВАЯ МЕХАНИКА МОЛЕКУЛ · PDF file2 ББК В22.31 УДК 544.11+539.183 isbn 978-5-94356-719-3 Стась Д.В., Плюснин В.Ф. Квантовая

150 ГЛАВА 6. ТЕРМЫ ГОМОЯДЕРНОЙ МОЛЕКУЛЫ

полных спинов S = 0,1, . . .,2s. Теперь осталось понять, что происходитв индексируемом комбинацией [M;M] пространстве из (2s+1)2 функцийвида Ψ(M,m1;M,m2), у которой не нашлось пары для симметризации попроекции орбитального момента. Ситуация здесь похожа на ситуацию споведением Σ термов относительно отражения в плоскости для гетеро-ядерной молекулы, которую мы анализировали в предыдущей главе.

Поскольку в нашем пространстве [M;M] имеется две независимыеспиновые переменные, пробегающие все возможные значения от −s до s,мы опять по одному разу получим каждое из возможных значений сум-марного спина от 0 до 2s. Вопрос только в том, какой четности эти термыбудут относительно операции инверсии. Поскольку пространственные пе-ременные в наших функциях одинаковы, возьмем второе представлениеоперации инверсии (6.14):

i Ψ(M,m1;M,m2) = (−1)2sΨ(M,m2;M,m1). (6.21)

Задача теперь превратилась в чисто спиновую: имеются две одинаковыеподсистемы со спином s каждая, описывающая их коллективное состоя-ние спиновая функция Ψ(m1;m2), где m1,2 — независимые проекции длядвух подсистем, и некоторая операция i, применение которой приводит кобмену подсистем проекциями и попутно умножает каждую функцию наодин и тот же коэффициент (−1)2s:

i Ψ(m1;m2) = (−1)2sΨ(m2;m1). (6.22)

В задаче узнается стандартная задача на сложение моментов, так что мыможем обычным образом построить функции с определенным значениемсуммарного спина S и его проекции MS = m1 + m2 — искомые функциитермов — в виде линейных комбинаций функций Ψ(m1;m2):

Φ(S,MS = m1 +m2) = ∑M=m1+m2

CS,MSm1,m2

Ψ(m1;m2). (6.23)

Задача теперь сводится к нахождению четности функций Φ(S,MS) отно-сительно операции i с учетом известного действия последней (6.22) наотдельные члены линейной комбинации (6.23):

i Φ(S,MS = m1 +m2) = i

(

∑M=m1+m2

CS,MSm1,m2

Ψ(m1;m2)

)

=

= (−1)2s ∑M=m1+m2

CS,MSm1,m2

Ψ(m2;m1). (6.24)

Page 151: КВАНТОВАЯ МЕХАНИКА МОЛЕКУЛ · PDF file2 ББК В22.31 УДК 544.11+539.183 isbn 978-5-94356-719-3 Стась Д.В., Плюснин В.Ф. Квантовая

6.4 ПРОСТРАНСТВА СОСТОЯНИЙ С ЧЕТНЫМИ Λ 151

Преобразованная функция в (6.24) отличается от исходной перестанов-кой индивидуальных спиновых состояний во всех слагаемых линейнойкомбинации. Теперь, чтобы вернуть функцию обратно саму в себя, намнужно переставить обратно индивидуальные спиновые состояния. Дляэтого нужно разобраться, как ведут себя функции полного спина (6.23)относительно перестановки индивидуальных спиновых состояний во всехслагающих ее функциях мультипликативного базиса.4

Вспомним, как строятся функции полного спина для этой задачи.Возвращаясь к привычным обозначениям Φ(S,M) → |S;M〉, Ψ(m1,m2) →|m1〉|m2〉, имеем:

S = Smax = 2s : |S = 2s;S〉 = |s〉|s〉

|S = 2s;S−1〉 =|s〉|s−1〉+|s−1〉|s〉√

2

|S = 2s;S−2〉 = |s〉|s−2〉+As|s−1〉|s−1〉+|s−2〉|s〉√2+A2

s

· · ·

(6.25)

Самая старшая функция в рассматриваемой системе по построениювсегда симметрична относительно перестановок индивидуальных спи-новых состояний, поскольку она просто есть произведение одинаковыхфункций. Можно также заметить, что все три выписанные в (6.25) функ-ции симметричны относительно перестановок. Более того, поскольку всефункции мультиплета проращиваются из одной лестничным операторомS− = s1− +s2−, который коммутирует с оператором перестановки индиви-дуальных состояний, все функции мультиплета всегда имеют одну и туже четность относительно операции перестановки. Таким образом, всефункции мультиплета с максимальным спином S = 2s будут четны отно-сительно перестановок индивидуальных спиновых переменных в своихслагаемых. Давайте посмотрим, что получится для спина, на единицуменьше максимального:

S = 2s−1 : |S = 2s−1;S〉 =|s〉|s−1〉−|s−1〉|s〉√

2

|S = 2s−1;S−1〉 =|s〉|s−2〉−|s−2〉|s〉√

2

· · ·

(6.26)

4См. задачу 3.39 в книге В. М. Галицкий, Б. М. Карнаков, В. И. Коган, Задачи поквантовой механике, М. Наука, 1981 г.

Page 152: КВАНТОВАЯ МЕХАНИКА МОЛЕКУЛ · PDF file2 ББК В22.31 УДК 544.11+539.183 isbn 978-5-94356-719-3 Стась Д.В., Плюснин В.Ф. Квантовая

152 ГЛАВА 6. ТЕРМЫ ГОМОЯДЕРНОЙ МОЛЕКУЛЫ

Все функции мультиплета со спином, на единицу меньше максимально-го, обязательно будут нечетными относительно операции перестановки.Возьмем следующий мультиплет:

S = 2s−2 : |S = 2s−2;S〉 =|s〉|s−2〉+αs|s−1〉|s−1〉+|s−2〉|s〉√

2+α2s

· · ·(6.27)

Старшая функция, а с ней и весь мультиплет, будут четны относительнооперации перестановки, и т. д.

Идя дальше по уменьшающемуся суммарному спину, получаем, чтовсе функции полного спина Φ(S,MS) для системы из двух одинаковыхспинов s обязательно будут иметь определенную четность Pt относитель-но операции перестановки индивидуальных спиновых состояний в со-ставляющих ее функциях |m1〉|m2〉, причем для любого спина s все функ-ции с максимальным суммарным спином (S = 2s) всегда будут четными,все функции с суммарным спином на единицу меньше максимального —нечетными, со спином на двойку меньше максимального — опять чет-ными, и дальше с каждым уменьшением суммарного спина на единицучетности будут чередоваться.

Таким образом, мы можем вернуть преобразованную функцию в (6.24)обратно в исходную, получая при этом:

i Φ(S,MS) = Φ(S,MS) × (−1)2s × (+1); S = 2s(−1); S = 2s−1(+1); S = 2s−2(−1); S = 2s−3

· · ·

(6.28)

Давайте теперь проследим, как связана четность функции относи-тельно инверсии i с ее полным спином. Пусть спин s полуцелый, тогда2s будет нечетным числом, (−1)2s = −1, максимальный суммарный спиннечетен, и для поведения функций Φ(S,MS) относительно инверсии i

i Φ(S,MS) = Pi Φ(S,MS) = (−1)2sPt Φ(S,MS) (6.29)

в зависимости от величины полного спина S и его четности PS получим

Page 153: КВАНТОВАЯ МЕХАНИКА МОЛЕКУЛ · PDF file2 ББК В22.31 УДК 544.11+539.183 isbn 978-5-94356-719-3 Стась Д.В., Плюснин В.Ф. Квантовая

6.4 ПРОСТРАНСТВА СОСТОЯНИЙ С ЧЕТНЫМИ Λ 153

следующие четности Pi:

S PS Pt (−1)2s Pi

2s (−1) (+1) (−1) (−1)

2s−1 (+1) (−1) (−1) (+1)

2s−2 (−1) (+1) (−1) (−1)

2s−3 (+1) (−1) (−1) (+1)

· · ·

(6.30)

Можно заметить, что в этом случае четность функции относительно ин-версии Pi совпадает с четностью ее суммарного спина PS — эти величиныв таблице подчеркнуты.

Пусть теперь спин s целый, тогда 2s будет четным числом, и для пове-дения функций Φ(S,MS) относительно операции инверсии аналогичнымобразом получаем:

S PS Pt (−1)2s Pi

2s (+1) (+1) (+1) (+1)

2s−1 (−1) (−1) (+1) (−1)

2s−2 (+1) (+1) (+1) (+1)

2s−3 (−1) (−1) (+1) (−1)

· · ·

(6.31)

И опять четность Pi совпадает с четностью суммарного спина PS.Таким образом, в каждом пространстве, индексируемом комбинацией

электронных моментов [M;M], имеется ровно по одному терму с каж-дым из возможных значений суммарного спина S = 0,1, . . .,2s, причемчетность его относительно операции инверсии i совпадает с четностьюсуммарного спина. Отсюда следует второе правило Вигнера—Витмера:

Page 154: КВАНТОВАЯ МЕХАНИКА МОЛЕКУЛ · PDF file2 ББК В22.31 УДК 544.11+539.183 isbn 978-5-94356-719-3 Стась Д.В., Плюснин В.Ф. Квантовая

154 ГЛАВА 6. ТЕРМЫ ГОМОЯДЕРНОЙ МОЛЕКУЛЫ

• для четных Λ имеется нечетное число 2k+1 термов с данным Λ длякаждого из возможных значений суммарного спина 0,1, . . .,2s, изних k термов будут g термами, k термов — u термами, а четностьпоследнего терма совпадает с четностью его суммарного спина.

6.5 Пространства состояний с Λ = 0

Последнее, что нам теперь осталось сделать — дополнительно клас-сифицировать состояния с Λ = 0 (Σ термы) по отношению к операцииотражения в плоскости, содержащей ось молекулы. Состояния с нулевойпроекцией орбитального момента в нашей системе даются функциямивида Ψ(M,m1;−M,m2) для всех −L ≤ M ≤ L. Функции опять распадают-ся на 2L+1 индексируемых числом M пространств из (2s+1)2 состояний,на которых снова решается задача о сложении спинов с тем же резуль-татом: полный спин S принимает все возможные значения от 0 до 2sровно по одному разу. Таким образом, для каждого пространства M мыполучим ровно по одному терму 2S+1Σ, как было и для гетероядерноймолекулы. Вопрос заключается в их поведении относительно операцийотражения σ (±) и инверсии i (g/u).

Давайте вспомним, как ведет себя волновая функция Ψ(M1,m1;M2,m2)относительно отражения в плоскости (5.38):

σ Ψ(M1,m1;M2,m2) = Pa Pb (−1)La (−1)Lb Ψ(−M1,m1;−M2,m2). (6.32)

Отражение изменило знаки орбитальных проекций и выдало фазовыймножитель. Поскольку мы рассматриваем ситуацию одинаковых атомовв одинаковых электронных состояниях, Pa = Pb, La = Lb, и фазовый мно-житель превращается в единицу. Кроме того, у нас сейчас M1 =−M2 = M,так что получаем:

σ Ψ(M,m1;−M,m2) = Ψ(−M,m1;M,m2). (6.33)

Хотя результат действия операции отражения в этой записи выглядиттак же, как результат действия операции инверсии i (6.13), на самомделе это не совсем так. В данном случае отражение просто поменялознаки всех проекций в системе, но не перенесло электроны с одногоцентра на другой, как инверсия i в (6.13). Чтобы свести этот случай куже разобранным, введем композитную операцию

R = σ · i, (6.34)

Page 155: КВАНТОВАЯ МЕХАНИКА МОЛЕКУЛ · PDF file2 ББК В22.31 УДК 544.11+539.183 isbn 978-5-94356-719-3 Стась Д.В., Плюснин В.Ф. Квантовая

6.6 ПРАВИЛА ВИГНЕРА—ВИТМЕРА 155

для которой получим:

RΨ(M,m1;−M,m2) = (−1)2sΨ(−M,m2;M,m1). (6.35)

Рассуждая прежним образом, можно из функций Ψ(M,m1;−M,m2)снова построить функции с определенным суммарным спином и его про-екцией ΦM(Λ = 0;S,MS) и убедиться, что действие операции R на нихсвелось к перестановке полных комплектов переменных для двух ато-мов — то же действие, что давала операция i на «лишних» функцияхпространства [M;M] из предыдущего случая. Значит, и результат его ока-жется тем же самым: четность PR относительно операции R для функцийΣ термов ΦM(Λ = 0;S,MS) совпадает с четностью суммарного спина дляданного терма. Поскольку все три операции в (6.34) коммутируют междусобой, функции имеют определенную четность по отношению к каждойиз них, и из (6.34) имеем:

PR = Pσ ×Pi. (6.36)

Получаем, что для PR = 1 все «+» термы (Pσ = 1) обязательно будут gтермами (Pi = 1), а все «−» термы (Pσ = −1) — u термами (Pi = 1). ДляPR = −1 получаем обратные соотношения: все «+» термы обязательнобудут «u» термами, а все «−» термы — «g» термами. Отсюда следуеттретье правило Вигнера—Витмера:

• для Λ = 0 четности относительно операций инверсии и отраженияв плоскости оказываются не независимыми и определяются следу-ющим образом: при четном суммарном спине все Σ+ термы будутΣ+

g термами, все Σ− термы — Σ−u термами; при нечетном спине

все Σ+ термы будут Σ+u термами, все Σ− термы — Σ−

g термами.

6.6 Правила Вигнера—Витмера

Результаты предыдущих разделов можно собрать вместе в следующиеправила соответствия между термами одинаковых атомов в одинаковыхэлектронных состояниях 2s+1L и термами образующихся при их сведениигомоядерных двухатомных молекул 2S+1Λ. В приближении свободногоспина возможные орбитальные Λ и спиновые S состояния находятся также, как для гетероядерной молекулы. Поведение термов относительнооперации инверсии i определяется следующим образом.

Page 156: КВАНТОВАЯ МЕХАНИКА МОЛЕКУЛ · PDF file2 ББК В22.31 УДК 544.11+539.183 isbn 978-5-94356-719-3 Стась Д.В., Плюснин В.Ф. Квантовая

156 ГЛАВА 6. ТЕРМЫ ГОМОЯДЕРНОЙ МОЛЕКУЛЫ

1. Для нечетных Λ:

• для всех S=0,1,. . . ,2s

2S+1Ng = 2S+1Nu = 2S+1Nгет/2, (6.37)

где 2S+1Ng/u — количество g/u термов данной мультиплетнос-ти для гомоядерной молекулы, 2S+1Nгет — количество термовданной мультиплетности для гетероядерной молекулы.

2. Для четных Λ > 0:

• для четных S:

2S+1Ng = 2S+1Nu +1; 2S+1Ng + 2S+1Nu = 2S+1Nгет, (6.38)

• для нечетных S:

2S+1Nu = 2S+1Ng +1; 2S+1Nu + 2S+1Ng = 2S+1Nгет, (6.39)

3. Для Λ = 0:

• для четных S:

2S+1N+g = 2S+1N−

u +1; 2S+1N+g + 2S+1N−

u = 2S+1Nгет, (6.40)

• для нечетных S:

2S+1N+u = 2S+1N−

g +1; 2S+1N+u + 2S+1N−

g = 2S+1Nгет. (6.41)

Эти правила и принято называть правилами Вигнера—Витмера. При ихпрактическом применении удобно рассуждать следующим образом:

• ищем возможные Λ и S как для гетероядерной молекулы, затемрасставляем индексы g/u:

• сначала для нечетных Λ: таких термов всегда получается четноеколичество для каждого возможного спина, половина термов полу-чает индекс g, вторая половина — индекс u.

• затем для четных Λ > 0: для каждого возможного спина получаемнечетное количество термов, один откладываем в сторону, остав-шиеся делим пополам и приписываем им индексы g/u, а четностьпоследнего, отложенного терма определяем по его спину.

Page 157: КВАНТОВАЯ МЕХАНИКА МОЛЕКУЛ · PDF file2 ББК В22.31 УДК 544.11+539.183 isbn 978-5-94356-719-3 Стась Д.В., Плюснин В.Ф. Квантовая

6.6 ПРАВИЛА ВИГНЕРА—ВИТМЕРА 157

• последними обрабатываем Σ термы: делаем все как для для чет-ных Λ > 0, а затем навешиваем индексы «+» и «−» так, чтобы «+»было больше. Отметим, что правило нахождения «±»-четности длягетероядерной молекулы на основе четности электронных оболочекатомов и их орбитальных моментов (5.41) здесь бесполезно, хотяформально соблюдается, поскольку, как оно и предсказывает дляодинаковых состояний одинаковых атомов, Σ+ термов всегда полу-чается на один больше.

В качестве примера давайте найдем термы молекулы F2, которые мо-гут быть получены при сведении двух атомов фтора в своих основныхсостояниях. Электронная конфигурация атома фтора 2p5, терм основногосостояния 2P.

• Сначала смотрим, что получилось бы для гетероядерной молекулы:собираем возможные значения Λ:

Λ = 2 : (1;1)Λ = 1 : (1;0), (0;1)Λ = 0 : (1;−1), (0;0), (−1;1),

(6.42)

по спинам имеем:

SF =12, ⇒ SF2 = 0,1. (6.43)

• Для нечетных Λ (у нас это Λ = 1) имеем два Π терма для каждогоспина, в соответствии с первым правилом один из них будет Πg,другой — Πu термом, итого получаем по одному 1Πg, 3Πg, 1Πu и3Πu терму.

• Для четных Λ > 0 (у нас это Λ = 2) имеем только один ∆ терм длякаждого спина, в соответствии со вторым правилом получаем один1∆g терм для четного спина S = 0 и один 3∆u терм для нечетногоспина S = 1.

• Для каждого спина имеем три Σ терма, и в соответствии с третьимправилом получаем следующие термы: 2×1Σ+

g , 1×1Σ−u для четного

спина S = 0 и 2× 3Σ+u , 1× 3Σ−

g для нечетного спина S = 1.

Итого для молекулы F2, получаемой при сведении двух атомов фтора восновных состояниях, можно получить следующие термы: 1∆g, 3∆u, 1Πg,3Πg, 1Πu, 3Πu, 2× 1Σ+

g ,1Σ−

u , 2× 3Σ+u ,

3Σ−g .

Page 158: КВАНТОВАЯ МЕХАНИКА МОЛЕКУЛ · PDF file2 ББК В22.31 УДК 544.11+539.183 isbn 978-5-94356-719-3 Стась Д.В., Плюснин В.Ф. Квантовая

158 ГЛАВА 6. ТЕРМЫ ГОМОЯДЕРНОЙ МОЛЕКУЛЫ

6.7 Одинаковые атомы в разных состояниях

Пока мы обсуждали только задачу о сведении одинаковых атомов вполностью одинаковых электронных состояниях. В этом случае переста-новка электронных состояний атомов не давала физически отличимогодругого состояния системы, что и приводило ко всем анализируемымв данной главе сложностям. Ситуация аналогична разбору поведенияфункции Ψ0Ψ0 для гетероядерной двухатомной молекулы относительнооперации отражения в плоскости: невозможность найти этой функциипару для симметризации относительно анализируемой операции σ за-ставляет скрупулезно описывать ее поведение относительно отражения,тогда как для любой функции ΨMΨ−M с M 6= 0 всегда найдется парная ейфункция Ψ−MΨM, и эту пару переводимых друг в друга функций всегдаможно симметризовать относительно σ , получив соответственно четныйи нечетный терм. Детали действия операции σ на функцию ΨMΨ−M приэтом становятся неважны, достаточно просто знать, что имеется парапереходящих друг в друга функций.

При сведении одинаковых атомов в разных электронных состояни-ях ситуация получается аналогичная: поскольку ядра одинаковы, мы всеравно получим гомоядерную молекулу AA и в системе имеется центринверсии, но, так как состояния атомов различаются, их перестановкадаст другое физическое состояние системы, и мы в итоге получаем парупереходящих друг в друга функций для симметризации. Если состоянияатомов описываются функциями Φα и Φβ , то коллективное состояниесистемы описывается произведением Φa

αΦbβ и эквивалентным ему Φa

β Φbα ,

где первым в произведении всегда идет функция, центрированная на ато-ме a. Поскольку при инверсии i центрированная на атоме a функция Φa

α ,βперейдет в центрированную на атоме b функцию Φb

α ,β и обратно, возмож-но с фазовым множителем:

Φaα ,β

i⇔ P1,2 Φbα ,β , (6.44)

то можно составить линейные комбинации

Ψ± = Φaα Φb

β ±Φaβ Φb

α , (6.45)

которые под действием операции инверсии будут переходить сами в себя:

i Ψ+ = i(

Φaα Φb

β +Φaβ Φb

α

)

= P1P2

(

ΦbαΦa

β +Φbβ Φa

α

)

=

= P1P2

(

Φaβ Φb

α +ΦaαΦb

β

)

= P1P2 Ψ+, (6.46)

Page 159: КВАНТОВАЯ МЕХАНИКА МОЛЕКУЛ · PDF file2 ББК В22.31 УДК 544.11+539.183 isbn 978-5-94356-719-3 Стась Д.В., Плюснин В.Ф. Квантовая

6.7 ОДИНАКОВЫЕ АТОМЫ В РАЗНЫХ СОСТОЯНИЯХ 159

. /0 01 2

34

35. /0

01 2

. /0

01 2

6

. /0

01 2

. /0

01 2

3 7

386

Рис. 6.4. Иллюстрация природы «g/u удвоения» на примере электронных состо-яний, возникающих при разведении молекулярного иона H+

2 .

i Ψ− = i(

Φaα Φb

β −Φaβ Φb

α

)

= P1P2

(

Φbα Φa

β −Φbβ Φa

α

)

=

= P1P2

(

Φaβ Φb

α −Φaα Φb

β

)

= −P1P2 Ψ−. (6.47)

Преобразованные функции здесь опять приведены к виду, в котором пер-вой в произведении идет функция, центрированная на атоме a.

Таким образом, для любой пары разных состояний одинаковых ато-мов Φα ,Φβ, дававших для гетероядерной молекулы одно состояние,в данном случае получается по два молекулярных состояния, одно g иодно u. Эта ситуация называется «g/u удвоение термов». На практике та-кие задачи решаются следующим образом: термы ищутся как для гетеро-ядерной молекулы, а затем каждому полученному терму приписываетсяиндекс g/u, тем самым удваивая их количество.

Примирить происходящее с интуитивно ожидаемым помогает обра-щение рассмотренной задачи — переход к анализу распада двухатомнойгомоядерной молекулы на два атома. На рис. 6.4 показан простейшийсодержательный пример такого процесса: разведение молекулярного ио-на H+

2 на два «атома водорода». Поскольку в системе имеется всего одинэлектрон, реально при разведении мы получим один протон и один атомводорода, причем электрон имеет равные шансы оказаться как на центре

Page 160: КВАНТОВАЯ МЕХАНИКА МОЛЕКУЛ · PDF file2 ББК В22.31 УДК 544.11+539.183 isbn 978-5-94356-719-3 Стась Д.В., Плюснин В.Ф. Квантовая

160 ГЛАВА 6. ТЕРМЫ ГОМОЯДЕРНОЙ МОЛЕКУЛЫ

a, так и на центре b. Это дает первую пару показанных на рисунке раз-веденных состояний ψa и ψb. Взяв теперь их (нормированную) сумму иразность, получим приведенные по симметрии относительно операции iфункции ψg = (ψa +ψb)/

√2 и ψu = (ψa −ψb)/

√2, показанные на рисунке

справа. Так же как при построении многоэлектронных волновых функ-ций физика требует здесь перехода от описаний вида «первый атом всостоянии Φα , второй — в состоянии Φβ» к описаниям «один атом всостоянии Φα , другой — в состоянии Φβ».

В качестве примера давайте найдем термы, в которых может ока-заться молекула кислорода при сведении ее из двух атомов кислородав основных электронных конфигурациях, один из которых находится восновном состоянии 3P, а второй — в возбужденном состоянии 1D:

O(3P)+O

(1D)→ O2. (6.48)

Сначала решаем задачу как задачу о двух разных атомах A и B с элек-тронными конфигурациями p4 → p2, находящимися в термах 3P и 1D,соответственно. По спину имеется единственная возможность SO2 = 1.Строим возможные значения Λ, указывая первым проекцию орбитально-го момента для атома кислорода с большим моментом «B»:

Λ = 3 : (2;1)Λ = 2 : (2;0), (1;1)Λ = 1 : (2;−1), (1;0), (0;1)Λ = 0 : (1;−1), (0;0), (−1;1).

(6.49)

Получаем один 3Φ терм, два 3∆ терма, три 3Π терма и три 3Σ терма,один из которых будет 3Σ+, второй — 3Σ−. Четность третьего 3Σ термаотносительно отражения определяем по стандартным правилам (5.39):

PA PB (−1)LA (−1)LB = (−1)3 (−1)3(−1)1 (−1)2 = −1. (6.50)

Третий 3Σ терм будет 3Σ− термом. Итого для нашей эффективной двух-атомной молекулы AB получаем следующие термы: 3Φ, 2× 3∆, 3× 3Π,2× 3Σ−, 3Σ+. Теперь удваиваем все термы, добавляя индексы g/u: дляполучаемой в реакции (6.48) молекулы кислорода можно получить тер-мы 3Φg, 2× 3∆g, 3× 3Πg, 2× 3Σ−

g ,3Σ+

g (полный четный комплект) и 3Φu,2× 3∆u, 3× 3Πu, 2× 3Σ−

u ,3Σ+

u (полный нечетный комплект).

Page 161: КВАНТОВАЯ МЕХАНИКА МОЛЕКУЛ · PDF file2 ББК В22.31 УДК 544.11+539.183 isbn 978-5-94356-719-3 Стась Д.В., Плюснин В.Ф. Квантовая

6.8 ОПОЗНАНИЕ ЗАДАЧИ НА СВЕДЕНИЕ АТОМОВ 161

6.8 Опознание задачи на сведение атомов

Таким образом, задача на связь термов атомов с термами образуемойпри их сведении двухатомной молекулы в зависимости от конкретныхусловий может решаться по трем разным алгоритмам: два разных атома,два одинаковых атома в разных электронных состояниях, и два оди-наковых атома в одинаковых электронных состояниях (по увеличениюсложности). Поэтому, прежде чем браться за решение, нужно сначалаопознать задачу, а затем уже следовать одному из трех разобранныхспособов. И если с гетероядерными молекулами все однозначно, то приобразовании гомоядерных молекул нужно быть внимательными. Давайтев качестве примера несколькими разными способами образуем молекулуазота N2 и посмотрим, какие при этом получатся возможные термы.

• Два атома азота в основных электронных конфигурациях (p3).

Электронная конфигурация p3 порождает термы 2D, 2P и 4S, изкоторых последний будет основным. Всего возможны 6 комбинацийтермов:

– N(2D)+N(2D) — одинаковые атомы в одинаковых состояниях:возможны S = 0,1, Λ = 4,2×3,3×2,4×1,5×0. Применяяправила Вигнера—Витмера, получаем следующие термы:

1Γg,3Γu,

1,3Φg,1,3Φu,

2× 1∆g,1∆u,2× 3∆u,

3∆g,

2× 1,3Πg,2× 1,3Πu,

3× 1Σ+g ,2× 1Σ−

u ,3× 3Σ+u ,2× 3Σ−

g .

(6.51)

– N(2D)+N(2P) — одинаковые атомы в разных состояниях:здесь возможны S = 0,1, Λ = 3,2×2,3×1,3×0, четность«лишнего» Σ терма — «−». Применяя правила g/u удвоения,получаем следующие термы:

1,3Φg,1,3Φu,

2× 1,3∆g,2× 1,3∆u,

3× 1,3Πg,3× 1,3Πu,

2× 1,3Σ−g , 1,3Σ+

g ,2× 1,3Σ−u , 1,3Σ+

u .

(6.52)

Page 162: КВАНТОВАЯ МЕХАНИКА МОЛЕКУЛ · PDF file2 ББК В22.31 УДК 544.11+539.183 isbn 978-5-94356-719-3 Стась Д.В., Плюснин В.Ф. Квантовая

162 ГЛАВА 6. ТЕРМЫ ГОМОЯДЕРНОЙ МОЛЕКУЛЫ

– N(2D)+N(4S) — одинаковые атомы в разных состояниях:

возможны S = 1,2, Λ = 2, единственный Σ терм будет Σ+

термом. Применяем правила g/u удвоения и получаем термы:

3,5∆g,3,5∆u,

3,5Πg,3,5Πu,

3,5Σ+g , 3,5Σ+

u .

(6.53)

– N(2P)+N(2P) — одинаковые атомы в одинаковых состояниях:

возможны S = 0,1, Λ = 2,2× 1,3× 0. Применяя правилаВигнера—Витмера, получаем следующие термы:

1∆g,3∆u,

1,3Πg,1,3Πu,

2× 1Σ+g , 1Σ−

u ,2× 3Σ+u , 3Σ−

g .

(6.54)

– N(2P)+N(4S) — одинаковые атомы в разных состояниях:

возможны S = 1,2, Λ = 1, единственный Σ терм будет Σ−

термом. Применяем правила g/u удвоения и получаем:

3,5Πg,3,5Πu,

3,5Σ−g , 3,5Σ−

u .(6.55)

– N(4S)+N(4S) — одинаковые атомы в одинаковых состояниях:

возможны S = 0,1,2,3, Λ = 0. Имеем только Σ термы, длякоторых из правил Вигнера—Витмера получаем:

1Σ+g , 3Σ+

u , 5Σ+g , 7Σ+

u . (6.56)

• Один атома азота в основной электронной конфигурации p3, дру-гой — в возбужденной конфигурации s1p4

Электронная конфигурация p3 порождает термы 2D, 2P и 4S. Длянахождения термов конфигурации s1p4 сначала превратим конфи-гурацию p4 в эквивалентную ей p2, которая порождает термы 1D,3P и 1S, и затем добавим s электрон, получая термы 2D, 2,4P и 2S.

Page 163: КВАНТОВАЯ МЕХАНИКА МОЛЕКУЛ · PDF file2 ББК В22.31 УДК 544.11+539.183 isbn 978-5-94356-719-3 Стась Д.В., Плюснин В.Ф. Квантовая

6.8 ОПОЗНАНИЕ ЗАДАЧИ НА СВЕДЕНИЕ АТОМОВ 163

Несмотря на то, что у двух атомов получились формально совпа-дающие термы 2D и 2P, в данном случае они порождаются раз-

ными электронными конфигурациями, и мы всегда будем иметьситуацию «одинаковые атомы в разных состояниях». Всего возмож-ны 3×4 = 12 комбинаций термов, поэтому мы рассмотрим толькокомбинации с участием 2D терма атома, находящегося в основнойэлектронной конфигурации, помечая атом в возбужденной конфи-гурации звездочкой:

– N(2D)+N∗(2D) — одинаковые атомы в разных состояниях:возможны S = 0,1, Λ = 4,2×3,3×2,4×1,5×0, четность«лишнего» Σ терма равна «−», поскольку здесь в одном атомеимеется три, а в другом — четыре «нечетных» p электрона.Применяя правила g/u удвоения, получаем следующие термы:

1,3Γg,1,3Γu,

2× 1,3Φg,2× 1,3Φu,

3× 1,3∆g,3× 1,3∆u,

4× 1,3Πg,4× 1,3Πu,

3× 1,3Σ−g ,2× 1,3Σ+

g ,3× 1,3Σ−u ,2× 1,3Σ+

u .

(6.57)

– N(2D)+N∗(2P) — одинаковые атомы в разных состояниях:возможны S = 0,1, Λ = 3,2×2,3×1,3×0, для «лишнего» Σтерма получим четность «+», на чем и заканчиваются отличияот уже рассмотренного случая (6.52). Результат можно взятьпрямо оттуда, просто перевернув индексы «±» у Σ термов:

1,3Φg,1,3Φu,

2× 1,3∆g,2× 1,3∆u,

3× 1,3Πg,3× 1,3Πu,

2× 1,3Σ+g , 1,3Σ−

g ,2× 1,3Σ+u , 1,3Σ−

u .

(6.58)

– N(2D)+N∗(4P) — одинаковые атомы в разных состояниях:

возможны S = 1,2, Λ = 3,2×2,3×1,3×0, «лишний» Σ термимеет четность «+». От предыдущего случая этот отличаетсятолько сдвижкой спинов с S = 0,1 до S = 1,2, поэтому

Page 164: КВАНТОВАЯ МЕХАНИКА МОЛЕКУЛ · PDF file2 ББК В22.31 УДК 544.11+539.183 isbn 978-5-94356-719-3 Стась Д.В., Плюснин В.Ф. Квантовая

164 ГЛАВА 6. ТЕРМЫ ГОМОЯДЕРНОЙ МОЛЕКУЛЫ

готовый результат можно взять из (6.58), заменив везде муль-типлетности 1,3 на 3,5:

3,5Φg,3,5Φu,

2× 3,5∆g,2× 3,5∆u,

3× 3,5Πg,3× 3,5Πu,

2× 3,5Σ+g , 3,5Σ−

g ,2× 3,5Σ+u , 3,5Σ−

u .

(6.59)

– N(2D)+N∗(2S) — одинаковые атомы в разных состояниях:

возможны S = 0,1, Λ = 2, единственный Σ терм будет Σ−

термом. Применяем правила g/u удвоения и получаем термы:

1,3∆g,1,3∆u,

1,3Πg,1,3Πu,

1,3Σ−g , 1,3Σ−

u .

(6.60)

• Ионы N+ и N− в своих основных электронных конфигурациях

Ионы N+ и N− имеют основные электронные конфигурации p2 иp4 → p2, соответственно, которые порождают одинаковые наборытермов: 1D, 3P и 1S. Однако одинаковые атомы здесь находятсяв разных зарядовых состояниях, и мы опять всегда будем иметьситуацию «одинаковые атомы в разных состояниях». Из 9 возмож-ных комбинаций термов мы рассмотрим только три комбинации сучастием 3P терма иона N+:

– N+(3P)+N−(1D) — одинаковые атомы в разных состояниях:

возможны S = 1, Λ = 3,2×2,3×1,3×0, «лишний» Σ термимеет четность «−». Опять используем алгоритм g/u удвоенияи получаем следующие термы:

3Φg,3Φu,

2× 3∆g,2× 3∆u,

3× 3Πg,3× 3Πu,

2× 3Σ−g , 3Σ+

g ,2× 3Σ−u , 3Σ+

u .

(6.61)

Page 165: КВАНТОВАЯ МЕХАНИКА МОЛЕКУЛ · PDF file2 ББК В22.31 УДК 544.11+539.183 isbn 978-5-94356-719-3 Стась Д.В., Плюснин В.Ф. Квантовая

6.9 УЧЕТ СПИН-ОРБИТАЛЬНОГО ВЗАИМОДЕЙСТВИЯ 165

– N+(3P)+N−(3P) — одинаковые атомы в разных состояниях:

возможны S = 0,1,2, Λ = 1,2× 1,3×0, «лишний» Σ термимеет четность «+». Получаем следующие термы:

1,3,5∆g,1,3,5∆u,

2× 1,3,5Πg,2× 1,3,5Πu,

2× 1,3,5Σ+g , 1,3,5Σ−

g ,2× 1,3,5Σ+u , 1,3,5Σ−

u .

(6.62)

– N+(3P)+N−(1S) — одинаковые атомы в разных состояниях:

возможны S = 1, Λ = 1,, единственный Σ терм будет Σ−

термом. Получаем следующие термы:

3Πg,3Πu,

3Σ−g , 3Σ−

u .(6.63)

6.9 Учет спин-орбитального взаимодействия

Учет спин-орбитального взаимодействия при классификации термовдвухатомной гомоядерной молекулы проводится таким же образом, какдля гетероядерной молекулы (см. раздел 5.8) и приводит к расщепле-нию ее термов по проекции полного момента на ось молекулы, а задачао нахождении соответствия между термами атомов и молекулы сноварешается в два этапа — построение термов в приближении доминирова-ния спин-орбитального взаимодействия при отсутствии вращения (слу-чай c по Гунду) и затем их корреляция с термами в приближении слабогоспин-орбитального взаимодействия при отсутствии вращения (случай aпо Гунду). Термы двух предельных случаев обозначаются так же, как идля гетероядерной молекулы, с добавлением индексов g/u, например, 3∆2u

и 0+g . Если для терма предельного случая a по Гунду величина проекции

полного момента однозначно определяется мультиплетностью и величи-ной Λ терма, то индекс Ω опускается, например, 1Σ+

g вместо 1Σ+0g и 1Πg

вместо 1Π1g, но 3Π2g, 3Π1g, 3Π0g для расщепляющегося терма 3Πg. Дляпостроения соответствия между молекулярными термами предела силь-ного взаимодействия спин-ось и термами атомов используется либо алго-ритм g/u удвоения термов, найденных как для гетероядерной молекулы(для одинаковых атомов в разных состояниях), либо адаптация правилВигнера—Витмера, которые принимают здесь следующий вид:

Page 166: КВАНТОВАЯ МЕХАНИКА МОЛЕКУЛ · PDF file2 ББК В22.31 УДК 544.11+539.183 isbn 978-5-94356-719-3 Стась Д.В., Плюснин В.Ф. Квантовая

166 ГЛАВА 6. ТЕРМЫ ГОМОЯДЕРНОЙ МОЛЕКУЛЫ

• При сведении двух одинаковых атомов в одинаковых состоянияхс полными моментами J1 = J2 = J общее число состояний каждоготипа по Ω остается тем же самым, что для гетероядерной молекулы,а их поведение относительно операции инверсии i определяетсяследующим образом:

• для целых J:

– для четных Ω:

Ng = Nu +1; Ng +Nu = Nгет; (6.64)

– для нечетных Ω:

Ng = Nu; Ng +Nu = Nгет; (6.65)

• для полуцелых J:

– для четных Ω:Ng = Nu; Ng +Nu = Nгет; (6.66)

– для нечетных Ω:

Nu = Ng +1; Ng +Nu = Nгет. (6.67)

• все 0+ термы будут четными, все 0− термы — нечетными.

В качестве примера давайте рассмотрим образование молекулы I2 издвух атомов йода, описанное в работе5. Считая, что атомы находятся всвоих основных электронных конфигурация, имеем термы 2P1/2 и 2P3/2.

Таким образом, нам нужно рассмотреть пары моментов J1 = J2 = 32,

J1 = J2 = 12 (одинаковые атомы в одинаковых состояниях, применяем пра-

вила Вигнера—Витмера) и J1 = 12, J2 = 3

2 (одинаковые атомы в разныхсостояниях, применяем алгоритм g/u удвоения). Получаем следующиетермы предела сильного спин-орбитального взаимодействия:

1. J1 = J2 = 32

(a) J1z = ±32 , J2z = ±3

2 : Ω = 3u, 0+g , 0−u ;

(b) J1z = ±12 , J2z = ±3

2 или J1z = ±32, J2z = ±1

2: Ω = 2g, 2u, 1g, 1u;

5R. S. Mulliken, Phys. Rev. 36, 1440 (1930).

Page 167: КВАНТОВАЯ МЕХАНИКА МОЛЕКУЛ · PDF file2 ББК В22.31 УДК 544.11+539.183 isbn 978-5-94356-719-3 Стась Д.В., Плюснин В.Ф. Квантовая

6.9 УЧЕТ СПИН-ОРБИТАЛЬНОГО ВЗАИМОДЕЙСТВИЯ 167

(c) J1z = ±12 , J2z = ±1

2 : Ω = 1u, 0+g , 0−u ;

2. J1 = 12, J2 = 3

2 или J1 = 32, J2 = 1

2

(a) J1 = 32, J1z = ±3

2, J2 = 12, J2z = ±1

2 и J2 = 32, J2z = ±3

2 , J1 = 12,

J1z = ±12 : Ω = 2g, 2u 1g, 1u;

(b) J1 = 32, J1z = ±1

2, J2 = 12, J2z = ±1

2 и J2 = 32, J2z = ±1

2 , J1 = 12,

J1z = ±12 : Ω = 1g, 1u 0+

g , 0+u , 0−g , 0−u ;

3. J1 = J2 = 12

(a) J1z = ±12 , J2z = ±1

2 : Ω = 1u, 0+g , 0−u .

Итого пара полных моментов J1 = J2 = 32 дает термы 3u, 2g, 2u, 1g, 2×1u,

2×0+g , 2×0−u , пара J1 = 1

2 , J2 = 32 — термы 2g, 2u 2×1g, 2×1u, 0+

g , 0+u , 0−g ,

0−u , и пара J1 = J2 = 12 — термы 1u, 0+

g , 0−u .Теперь нам нужно сопоставить полученные термы с термами преде-

ла слабого спин-орбитального взаимодействия. Для этого применяютсякорреляционные диаграммы, аналогичные использованные нами ранеедля сопоставления термов предельных случаев LS и j j-связи (см. гл. 3,рис. 3.7, 3.10) и опирающиеся на правило непересечения термов однойсимметрии для изменении внутреннего параметра системы — в данномслучае межатомного расстояния. Самое нижнее по энергии состояниетипа 0+

g предела сильного спин-орбитального взаимодействия (большо-го межатомного расстояния) должно перейти в самое нижнее по энер-гии четное относительно инверсии и отражения в плоскости состояниес нулевой проекцией полного момента на ось молекулы 2s+1Σ+

0g пределаслабого спин-орбитального взаимодействия (малого расстояния междуядрами), следующее по энергии состояние типа 0+

g — в следующее поэнергии состояние типа 2s+1Σ+

0g и так далее, для всех возможных комби-наций симметрийных признаков.

Прежде всего нужно найти соответствие между обозначениями ви-да 0+

g , 1u,. . . и обозначениями термов предела слабого спин-орбитальноговзаимодействия. Эти состояния (термы 1Σ+

g ,2Π3/2g ,

3∆2u и т. д.) такжеимеют вполне определенные значения Ω. Так, например, терм 1Σ+

g соот-ветствует терму 0+

g противоположного предельного случая, терм 1Σ−u —

терму 0−u , терм1Πg соответствует терму 1g, терм 1∆u — терму 2u, терм 3Π0g

расщепится на термы 0+g и 0−g , так как нулевую полную проекцию можно

получить двумя способами, терм 3Σ+g расщепится на 1g и 0−g и т. д.

Page 168: КВАНТОВАЯ МЕХАНИКА МОЛЕКУЛ · PDF file2 ББК В22.31 УДК 544.11+539.183 isbn 978-5-94356-719-3 Стась Д.В., Плюснин В.Ф. Квантовая

168 ГЛАВА 6. ТЕРМЫ ГОМОЯДЕРНОЙ МОЛЕКУЛЫ

9:;<=:><=?9<=?@<=?=<=:><9AB=ACB=A9B=A@B9:>B9?B=:;B9A<=AC<=A9<=A@<9:>B

D< >E<DB >D< ;F<E<EBDB ;EBFBDB >D< >

G<D< ;FBDB>EBE<D< ;E<F<DB>

HIJKHL HIJKH

HIJKHL HIMKH

HIMKHL HIMKH

E<

NOP QРис. 6.5. Корреляция атомных и молекулярных термов для молекулы I2 с учетом

спин-орбитального взаимодействия.

Затем по обычным правилам Вигнера—Витмера ищем термы гомо-ядерной молекулы I2, образуемой из двух атомов йода I(2P). Следуя стан-дартной процедуре, получим термы 1∆g, 3∆u, 1Πg, 1Πu, 3Πg, 3Πu, 2× 1Σ+

g ,1Σ−

u , 2× 3Σ+u ,

3Σ−g .

Теперь нам нужно расположить эти термы по возрастанию энергии.Сделать это из чисто симметрийных соображений невозможно, на дан-ном этапе требуется привлечение дополнительной информации, и опытработы с молекулами типа I2 позволяет с разумной долей вероятностирасположить термы в следующем порядке: 1Σ+

g ,3Πu, 1Πu, 3Σ−

g ,1∆g, 1Σ+

g ,3Πg, 1Πg, 3Σ+

u ,3∆u, 3Σ+

u ,1Σ−

u .

Page 169: КВАНТОВАЯ МЕХАНИКА МОЛЕКУЛ · PDF file2 ББК В22.31 УДК 544.11+539.183 isbn 978-5-94356-719-3 Стась Д.В., Плюснин В.Ф. Квантовая

6.10 КЛАССИФИКАЦИЯ ТЕРМОВ И ПРАВИЛА ОТБОРА 169

И наконец можно попытаться построить корреляции между термамиодной симметрии, постепенно поднимаясь по энергии. Основное состоя-ние молекулы I2 — терм 1Σ+

g — должен обязательно перейти в нижнийпо энергии терм типа 0+

g . В нашем примере им окажется один из тер-мов 0+

g , который можно получить из пары атомов в основных состоянияхI(2P3/2) (случаи 1a или 1c со стр. 166). Эту логику можно опять обра-тить и сказать, что при распаде молекулы I2 из своего основного состоя-ния 1Σ+

g на пару нейтральных атомов йода атомы окажутся в состояниях2P3/2. Для краткости эту связку можно записать в виде 1Σ+

g , 0+g

(32, 3

2

).

Идя теперь по термам в порядке возрастания энергии и при необхо-димости расщепляя их по Ω, последовательно получим: 3Π2u, 2u

(32, 3

2

);

3Π1u, 1u(

32 , 3

2

); 3Π0u, 0−u

(32 , 3

2

)и 0+

u

(32, 1

2

)(поскольку пара J1 = J2 = 3

2не дает требуемого по корреляции терма 0+

u , он заимствуется из сле-дующей по энергии группы уровней с J1,2 = 3

2, J2,1 = 12);

1Πu, 1u(3

2, 32

);

3Σ−g , 1g

(32, 3

2

)и 0+

g

(32, 3

2

); 1∆g, 2g

(32 , 3

2

); 1Σ+

g , 0+g

(32 , 1

2

); 3Π2g, 2g

(32, 1

2

);

3Π1g, 1g(3

2 , 12

); 3Π0g, 0−g

(32, 1

2

)и 0+

g

(12 , 1

2

); 1Πg, 1g

(32, 1

2

); 3Σ+

u , 0−u(3

2, 32

)

и 1u(3

2, 12

); 3∆3u, 3u

(32, 3

2

); 3∆2u , 2u

(32, 1

2

); 3∆1u, 1u

(32 , 1

2

); 3Σ+

u , 0−u(3

2, 12

1u(

12 , 1

2

); 1Σ−

u , 0−u(

12 , 1

2

). На рис. 6.5 эти правила корреляции показаны в

виде схемы соответствия уровней для двух предельных ситуаций.

6.10 Классификация термов и правила

отбора

Может возникнуть резонный вопрос — а зачем вообще нужна стольизощренная система классификации состояний атомов и молекул? Естьли это просто некий способ пересчитать состояния и этим ограничить-ся, или символы термов и стоящие за ними приведенные по симметрииволновые функции действительно полезны? Конечно ответ на этот во-прос будет положительным, и симметрийная классификация состояний,в компактной форме закодированная в обозначении терма, оказываетсяисключительно полезна для реальных расчетов.

Как мы уже не раз обсуждали, значительная часть расчетной кванто-вой механики состоит в вычислении матричных элементов вида

Vαβ = 〈φα |V |φβ 〉. (6.68)

Их приходится искать при построении матриц операторов, для описаниявероятностей переходов и т. д. Поскольку как правило базисы φi в

Page 170: КВАНТОВАЯ МЕХАНИКА МОЛЕКУЛ · PDF file2 ББК В22.31 УДК 544.11+539.183 isbn 978-5-94356-719-3 Стась Д.В., Плюснин В.Ф. Квантовая

170 ГЛАВА 6. ТЕРМЫ ГОМОЯДЕРНОЙ МОЛЕКУЛЫ

квантовой механике бесконечны, и матричных элементов (6.68) нужнобыло бы считать бесконечное количество. Даже при обрезании базисовдо конечных наборов, что делается в рамках всех основных приближе-ний расчетной квантовой механики, количество матричных элементов иразмеры матриц остаются слишком велики для комфортных расчетов.Однако удачным выбором базиса можно обратить некоторые матричныеэлементы (6.68) в нуль — чем больше будет таких нулей, тем боль-ше шансы добраться до искомого выражения. В обращении в нуль всехнедиагональных матричных элементов Vα 6=β и заключается решение тойзначительной части задач, которые связаны с нахождением собственныхчисел операторов.

Матричные элементы (6.68) представляют из себя интегралы вида

Vαβ =

φ ∗α(~r,ξ)V φβ (~r,ξ)d~rdξ (6.69)

по пространственным переменным ~r и остальным имеющимся в систе-ме степеням свободы ξ . Будучи скаляром, матричный элемент (6.69) недолжен меняться при любых преобразованиях, оставляющих описыва-емую систему неизменной, в том числе и при любых допускаемых ейпространственных преобразованиях. Если, например, при расчете интег-рала (6.69) для гомоядерной двухатомной молекулы в результате опера-ции инверсии i подынтегральное выражение, а с ним и весь интеграл,поменяли знак, то такой матричный элемент будет заведомо равен нулю.Общий формализм такого рода рассуждений мы оставим до обсужденияприложений теории групп в квантовомеханических расчетах. Сейчас жемы ограничимся полуинтуитивным обсуждением так называемых пра-вил отбора — правил, позволяющих без вычислений находить заведомонулевые матричные элементы — для четырех разобранных систем: од-ноэлектронного и многоэлектронного атома, гетероядерной и гомоядер-ной двухатомной молекулы. В качестве оператора V возьмем компонентырадиус-вектора ~r = (x,y, z) — оператора, определяющего самые интенсив-ные электрические дипольные переходы в электронных оболочках ато-мов и молекул, и ограничимся приближением свободного спина. Однозамечание в этих условиях можно сделать сразу: поскольку оператор ~rне затрагивает спиновых степеней свободы, матричный элемент (6.69)будет заведомо равен нулю для любой пары функций разной мультип-летности. Поэтому в дальнейшем мы будем анализировать его поведениетолько относительно пространственных преобразований.

Page 171: КВАНТОВАЯ МЕХАНИКА МОЛЕКУЛ · PDF file2 ББК В22.31 УДК 544.11+539.183 isbn 978-5-94356-719-3 Стась Д.В., Плюснин В.Ф. Квантовая

6.10 КЛАССИФИКАЦИЯ ТЕРМОВ И ПРАВИЛА ОТБОРА 171

Давайте начнем с одноэлектронного атома, например, атома H илииона He+. Базисом функций φi в этом случае будут водородоподобныефункции Rnl(r)Ylm(Ω), в которых все симметрийные свойства относитель-но преобразований пространства заключены в угловой части Ylm(Ω). Чтокасается оператора ~r, то для него преобразования пространства тожезатрагивают только угловую зависимость единичного вектора ~ρ =~r/r.Таким образом, нам нужно будет анализировать поведение матричныхэлементов вида

Vαβ = 〈Y αl ′m′ |ρx,y,z|Y β

lm〉 (6.70)

относительно имеющихся независимых операций пространственных пре-образований, допускаемых свободным атомом: поворотов и инверсии вцентре атома I.

Вспомним, что компоненты единичного вектора ~ρ эквивалентны сфе-рическим гармоникам с l = 1:

ρx = sinϑ cosϕρy = sinϑ sinϕ

∼ Y11±Y1−1

ρz = cosϑ ∼ Y10,(6.71)

поэтому сомножитель ρx,y,z в (6.70) эквивалентен Y1k:

Vαβ = 〈Y αl ′m′ |ρx,y,z|Yβ

lm〉 ∼ 〈Y αl ′m′ |Y1k|Y

βlm〉. (6.72)

Вспоминая стандартную задачу о сложении моментов, стоящее в (6.72)произведение Y1kYlm можно представить в виде некоторой линейной ком-бинации гармоник с суммарным моментом l ∈ l−1, l, l+1:

Y1kY βlm = ∑

l∈l−1,l,l+1ClqYlq. (6.73)

Соответственно и интеграл (6.72) распадется в линейную комбинациюскалярных произведений сферических гармоник:

〈Y αl ′m′ |ρx,y,z|Y β

lm〉 ∼ ∑l∈l−1,l,l+1

Clq〈Y αl ′m′ |Ylq〉. (6.74)

Так как функции Ylm с разными l ортогональны друг другу, матричныйэлемент (6.72) будет заведомо равен нулю, если l′ 6∈ l−1, l, l+1.

Page 172: КВАНТОВАЯ МЕХАНИКА МОЛЕКУЛ · PDF file2 ББК В22.31 УДК 544.11+539.183 isbn 978-5-94356-719-3 Стась Д.В., Плюснин В.Ф. Квантовая

172 ГЛАВА 6. ТЕРМЫ ГОМОЯДЕРНОЙ МОЛЕКУЛЫ

Отсюда получаем, что матричный элемент 〈2S+1L′|ρx,y,z|2S+1L〉 будетзаведомо равен нулю, если L отличается от L′ больше чем на единицу.Словами говорят, что векторный оператор ~ρ несет на себе момент l = 1 ипоэтому не может изменить момент состояния больше чем на единицу.

Анализируя аналогичным образом проекции момента, заключаем, чтоz компонента вектора, эквивалентная функции Yl0, не может изменитьпроекцию состояния, а компоненты x,y, несущие проекции ±1 каждая,должны обязательно менять проекцию состояния на единицу. Такое раз-деление ролей компонент вектора ρz и ρx,y сохраняется практически вовсех системах, поэтому для них вводятся стандартные обозначения ρ‖(«продольная компонента») и ρ⊥ («поперечная компонента»).

Наконец, и оператор ~ρ, и функции Ylm имеют вполне определеннуючетность относительно операции инверсии в центре атома I:

I~ρ = −~ρ,IYlm = (−1)l Ylm.

(6.75)

Поэтому применение операции инверсии к матричному элементу (6.70)дает:

IVαβ = I 〈Y αl ′m′ |ρx,y,z|Y β

lm〉 = (−1)l ′ (−1) (−1)l 〈Y αl ′m′ |ρx,y,z|Y β

lm〉 =

= (−1)l+l ′+1Vαβ . (6.76)

Поскольку для скаляра Vαβ обязательно

IVαβ = (+1)Vαβ , (6.77)

моменты l и l′ в (6.76) должны быть обязательно разной четности, чтоисключает возможность перехода в состояние с тем же моментом.

Обозначая спин, его проекцию, орбитальный момент и его проекциюсимволами S, MS, L, ML, соответственно, можно теперь записать

• Правила отбора для дипольных переходов в одноэлектронном атоме

S → S;MS → MS;L → L±1;ML → ML (ρ‖);

ML ±1 (ρ⊥).

(6.78)

Page 173: КВАНТОВАЯ МЕХАНИКА МОЛЕКУЛ · PDF file2 ББК В22.31 УДК 544.11+539.183 isbn 978-5-94356-719-3 Стась Д.В., Плюснин В.Ф. Квантовая

6.10 КЛАССИФИКАЦИЯ ТЕРМОВ И ПРАВИЛА ОТБОРА 173

Заведомо будут равны нулю все матричные элементы 〈3P | ρ‖,ρ⊥ | 5D〉(по спину), 〈4S | ρ‖,ρ⊥ | 4D〉 (по орбитальному моменту), 〈3P | ρ‖,ρ⊥ | 3P〉(по инверсии), а из 15 матричных элементов 〈3P |ρ‖ | 3D〉 и 〈3P |ρ⊥ | 3D〉ненулевыми могут оказаться только три для ρ‖ (с одинаковыми орби-тальными проекциями двух функций) и шесть для ρ⊥ (с отличающимисяна единицу орбитальными проекциями) — для каждой из трех спиновыхпроекций. Теперь, чтобы найти заведомо нулевые матричные элементы,не нужно ничего вычислять — достаточно просто посмотреть на символытермов начального и конечного состояния.

Давайте посмотрим, что изменится для многоэлектронного атома.Правила отбора по спину и его проекции остаются в силе. Запрет наизменение орбитального момента больше чем на единицу также остаетсяв силе, поскольку многоэлектронные функции ведут себя относительновращения так же, как обычные сферические гармоники. Однако пове-дение функции относительно инверсии теперь описывается не ее мо-ментом L, а четностью конфигурации P, поэтому снимается запрет напереход в состояние с тем же моментом и появляется новое независи-мое правило отбора по четности P. Кроме того, формально возможныйпереход из S в S состояние оказывается запрещен, поскольку умножениефункции с нулевым моментом на векторный оператор в (6.73) обязатель-но даст суммарный момент 1: ~0+~1 =~1. Вводя дополнительно обозначе-ние P, получаем следующие

• Правила отбора для переходов в многоэлектронном атоме

S → S;MS → MS;L → L,L±1, кроме переходов L = 0→ L = 0;ML → ML (ρ‖);

ML ±1 (ρ⊥);P → −P.

(6.79)

Так, все матричные элементы между любыми состояниями термов,порождаемых одной электронной конфигурацией и поэтому имеющиходинаковую четность P, заведомо будут равны нулю.

В двухатомной гетероядерной молекуле состояния классифицируютсяуже не по орбитальному моменту, а по модулю Λ его проекции на осьмолекулы, которая здесь становится выделенным направлением (именноему соответствует компонента оператора ρ‖). Поскольку компоненты ρ⊥

Page 174: КВАНТОВАЯ МЕХАНИКА МОЛЕКУЛ · PDF file2 ББК В22.31 УДК 544.11+539.183 isbn 978-5-94356-719-3 Стась Д.В., Плюснин В.Ф. Квантовая

174 ГЛАВА 6. ТЕРМЫ ГОМОЯДЕРНОЙ МОЛЕКУЛЫ

несут проекцию ±1, они сдвигают Λ на единицу, а параллельная оси ком-понента вектора, имеющая нулевую проекцию, не меняет Λ. Что касаетсяклассификации по индексу Pσ = ±, то здесь нужно рассмотреть толькопереход между двумя Σ термами, поскольку для термов с Λ > 0 четностьотносительно отражения в плоскости не определяется. Переход междудвумя Σ термами может индуцироваться только продольной компонентойρ‖ — уложенной на ось молекулы стрелкой, которая не изменяется приотражении в плоскости. Итого получаем следующие

• Правила отбора для переходов в гетероядерной молекуле

S → S;MS → MS;Λ → Λ (ρ‖);

Λ±1 (ρ⊥);± → ± (ρ‖).

(6.80)

Если в качестве базиса для описания двухкратно вырожденных термов2S+1Λ с Λ > 0 выбрать функции с определенными значениями проекцииорбитального момента на ось молекулы M1 = Λ и M2 =−Λ, то все матрич-ные элементы внутри термов окажутся равны нулю, поскольку потребуютизменения проекции на 2Λ ≥ 2.

В гомоядерной молекуле дополнительно появляется классификацияg/u по поведению относительно операции инверсии в центре молекулы i.Поскольку исходный оператор ~r у нас связан с дипольным моментом,привязанным к центру системы, у него тоже имеется определенная чет-ность относительно операции i: будучи привязанным к центру инверсии,он под действием операции i меняет знак. Это добавляет к (6.80) ещеодно правило отбора — на смену g/u четности состояний при переходе.Итого имеем

• Правила отбора для переходов в гомороядерной молекуле

S → S;MS → MS;Λ → Λ (ρ‖);

Λ±1 (ρ⊥);± → ± (ρ‖);gu

ug

.

(6.81)

Page 175: КВАНТОВАЯ МЕХАНИКА МОЛЕКУЛ · PDF file2 ББК В22.31 УДК 544.11+539.183 isbn 978-5-94356-719-3 Стась Д.В., Плюснин В.Ф. Квантовая

6.10 КЛАССИФИКАЦИЯ ТЕРМОВ И ПРАВИЛА ОТБОРА 175

Состояние 3Σ−g молекулы I2 (шестое по порядку состояние на рис. 6.5)

будет метастабильным, поскольку прямой дипольный переход из него восновное состояние 1Σ+

g оказывается запрещенным по спину, четности Pσи четности Pi. Частично запрет снимается для перехода с «пересадкой»на одном из термов группы 3Πu, однако и здесь запрет по спину остаетсяв силе и резко снижает и без того малую вероятность двухступенчатогоперехода.

При учете слабого спин-орбитального взаимодействия появляютсядополнительные правила отбора для величины полного момента J, егопроекции Jz и ее модуля Ω (в зависимости от системы). Отметим, что те-перь для двухатомной молекулы становятся возможны переходы междупарами состояний внутри термов типа 2Π1/2, поскольку проекция момен-та на ось здесь должна измениться на 1 (с +1

2 на −12).

Page 176: КВАНТОВАЯ МЕХАНИКА МОЛЕКУЛ · PDF file2 ББК В22.31 УДК 544.11+539.183 isbn 978-5-94356-719-3 Стась Д.В., Плюснин В.Ф. Квантовая
Page 177: КВАНТОВАЯ МЕХАНИКА МОЛЕКУЛ · PDF file2 ББК В22.31 УДК 544.11+539.183 isbn 978-5-94356-719-3 Стась Д.В., Плюснин В.Ф. Квантовая

Приложение A

Задачи

A.1 Общие свойства связанной системы

1.1 Исследуйте качественно возможность наличия связанных состоянийв потенциале с асимптотикой U(r)∼−β/rα , r →∞. Рассмотрите частныеслучаи: кулоново поле, гармонический осциллятор, Ван-дер-Ваальсововзаимодействие, экранированный кулон, потенциал Морзе. При наличиисвязанные состояния укажите, конечно или бесконечно их число?

1.2 Исследуйте, при каких α, β возможно падение частицы на центр впотенциале с асимптотикой U(r)∼−β/rα , r→0?

1.3 Исследуйте, при каких условиях потенциальную энергию U(r) мож-но рассматривать как возмущение для свободно движущейся частицы?

1.4 Найдите уровни энергии и нормированные волновые функции ста-ционарных состояний плоского и сферического осцилляторов (потенциалU(r) = αr2/2), определите кратности вырождения уровней.

1.5 На находящуюся при 4 K подложку наморожен молекулярный крис-талл из молекул HD. Найдите среднеквадратичное смещение атомов H иD, считая, что вращение молекул в кристалле в этих условиях полностьюзаморожено. Известно, что самая интенсивная линия в ИК спектре по-глощения для молекулы HD находится примерно на 4000 см−1.

Page 178: КВАНТОВАЯ МЕХАНИКА МОЛЕКУЛ · PDF file2 ББК В22.31 УДК 544.11+539.183 isbn 978-5-94356-719-3 Стась Д.В., Плюснин В.Ф. Квантовая

178 ГЛАВА A. ЗАДАЧИ

A.2 Одноэлектронный атом

2.6 Найдите вероятность возбуждения иона с одним электроном при из-менении его заряда ядра на 1 (Z→Z ±1, например, H→He+; He+→H).

2.7 Найдите энергию и потенциал ионизации иона H− вариационнымметодом.

2.8 Найдите поправку к энергии 1s уровня атома водорода при учетеконечного размера ядра. Считайте ядро шаром, по объему которого рав-номерно распределен заряд.

2.9 Найдите потенциал ионизации атома мюония, атома дейтерия и ионагелия, считая его известным для атома водорода. Найдите <U > и < T >в основных состояниях этих частиц.

2.10 Найдите электрический потенциал, создаваемый атомом водорода восновном состоянии.

2.11 Найдите электрическое поле, создаваемое атомом водорода в 2p со-стоянии с определенным значением m = 0 проекции момента электронана ось z на больших расстояниях от атома.

2.12 Качественно определите расщепление уровней атома водорода сглавным квантовым числом n = 4, 5, 6. . . в слабом постоянном элек-трическом поле.

2.13 Рассчитайте величину дипольного момента перехода 1s→2p дляатома водорода.

2.14 Оцените по порядку величины характерные размеры, энергии, ско-рости, времена и магнитные поля для водородоподобного иона. Как онизависят от заряда иона? Когда и почему становятся несправедливымитакие оценки?

2.15 Оцените по порядку величины характерные времена электронного,колебательного и вращательного движений для двухатомной молекулы.Как они зависят от масс входящих в молекулу атомов?

2.16 В сосуде, заполненном смесью водорода и гелия при низком дав-лении, горит электрический разряд, приводящий к образованию атомов

Page 179: КВАНТОВАЯ МЕХАНИКА МОЛЕКУЛ · PDF file2 ББК В22.31 УДК 544.11+539.183 isbn 978-5-94356-719-3 Стась Д.В., Плюснин В.Ф. Квантовая

A.3 МНОГОЭЛЕКТРОННЫЙ АТОМ 179

водорода, дейтерия (если он присутствует, то в следовых количествах), иионов He+. У вас имеется возможность снять один оптический спектр ввидимой/УФ области в узком диапазоне длин волн (∼1 нм). Предложитеобласть, в которой нужно записывать спектр, и укажите необходимоеспектральное разрешение прибора, требуемое для одновременной иден-тификации всех трех частиц.

2.17 Изобразите качественно радиальные, угловые и полные функциидля состояний водородоподобного иона до n = 3 включительно.

2.18 Постройте вещественные s, p, d, f функции из соответствующихсферических гармоник, припишите им говорящие индексы. Качественноизобразите построенные функции при разных значениях главного кван-тового числа n. Если s, p, d функции канонизированы, то для f функцийимеется несколько широко используемых наборов, например, так назы-ваемые «высокосимметричный» и «низкосимметричный» базисы. Что этотакое и зачем они нужны?

2.19 Постройте p функции в направлении (1, 1, 0), (1, 1, 1); в произ-вольном направлении, задаваемом углами α, β , γ между направляющимвектором и осями x, y и z.

2.20 Постройте в явном виде одноцентровые sp, sp2, sp3 гибридные ор-битали, используя в качестве базиса набор функций s, px , py , pz.

2.21 Кроме spn гибридных орбиталей часто оказываются полезными иs, p,d гибридные орбитали. В качестве примеров можно привести «тетра-эдрические» sd3, «квадратные» sp2d и «октаэдрические» sp3d2 орбитали.Как их построить из набора функций s, p, d?

A.3 Многоэлектронный атом

3.22 Найдите зависимость < rn > от Z в модели Томаса—Ферми.

3.23 Найдите зависимость характерной величины орбитального моментаэлектрона и энергии полной ионизации атома от Z в модели Томаса—Ферми.

3.24 Постройте слейтеровские орбитали для иона 18Ar2+ (электроннаяконфигурация 1s22s22p23s23p4).

Page 180: КВАНТОВАЯ МЕХАНИКА МОЛЕКУЛ · PDF file2 ББК В22.31 УДК 544.11+539.183 isbn 978-5-94356-719-3 Стась Д.В., Плюснин В.Ф. Квантовая

180 ГЛАВА A. ЗАДАЧИ

3.25 Используя слейтеровские функции, оцените размеры атома Fe0 иионов Fe1+, Fe2+, Fe3+, Fe4+ и сравните их с опытными данными.

3.26 Первый потенциал ионизации атома кислорода равен 13,614 эВ,серы — 10,357 эВ, селена — 9,75 эВ. Дайте качественное объяснениеэтому изменению. Оцените потенциал ионизации, если в приближениислейтеровских функций средняя энергия < E > электрона, находяще-гося на орбитали с главным квантовым числом n, дается выражением

< E >= −(Z∗n∗ )2 e2

2a0.

3.27 Найдите первый потенциал ионизации и потенциал ионизации 3dэлектронов атомов железа, никеля и цинка в приближении слейтеров-ских орбиталей и сравните полученные значения с опытными данными.

3.28 Найдите первый потенциал ионизации и сродство к электрону ато-мов натрия и калия в приближении слейтеровских орбиталей и сравнитеполученные значения с опытными данными.

3.29 Постройте рентгеновские спектры атомов элементов первого пе-риода в приближении слейтеровских орбиталей и сравните полученныезначения с опытными данными.

3.30 Расстояние между ионами K и Cl в кристалле KCl равно 3,14 A.Определите ионные радиусы этих частиц.

3.31 Изобразите качественно валентные орбитали атома кремния.

3.32 Постройте электронные конфигурации атомов элементов с 1 по 4период, объясните регулярности и нерегулярности в заполнении элек-тронных оболочек атомов с увеличением атомного номера.

3.33 Постройте качественно зависимости потенциалов ионизации IP исродства к электрону EA атомов элементов первых периодов, объяснитерегулярности и нерегулярности в построенной зависимости и сравнитесо справочными данными.

3.34 Укажите возможные электронные конфигурации ионов элементовпервого переходного блока, ответственных за парамагнетизм солей ме-таллов, и соответствующие им магнитные моменты.

Page 181: КВАНТОВАЯ МЕХАНИКА МОЛЕКУЛ · PDF file2 ББК В22.31 УДК 544.11+539.183 isbn 978-5-94356-719-3 Стась Д.В., Плюснин В.Ф. Квантовая

A.4 КЛАССИФИКАЦИЯ СОСТОЯНИЙ 181

A.4 Классификация состояний

4.35 Найдите средние значения операторов Lx, Lz, L2 в состояниях px,py, pz?

4.36 Найдите явный вид операторов Lx, Ly, Lz, L+ и L− как функцийуглов ϑ , φ и их производных.

4.37 Определите результат действия операторов Lx, Lz, L2 на функцииdx2, dxz, dyz, dxy, dx2−y2.

4.38 Разложите произведение Y1m1Y1m2 на сферические гармоники дляслучаев m1 = 1 и m2 = 1; m1 = 1 и m2 = 0; m1 = 1 и m2 = −1.

4.39 Определите, какие комбинации квадратов проекций спина электро-на на оси x, y, z могут одновременно иметь определенные значения.

4.40 Найдите кратности вырождения и четности электронных конфигу-раций s1p1d1, s2p2d2 и т. д.

4.41 Найдите термы, порождаемые электронными конфигурациями p2,d3, s1p1, s1p1d1 и т. д.

4.42 Найдите термы, порождаемые электронной конфигурацией p3 вслучаях LS- и j j-связи, укажите в обоих случаях терм основного со-стояния.

4.43 Постройте диаграмму уровней с учетом слабого спин-орбитальноговзаимодействия для конфигураций p2, d2, p1d1 и т. д.

4.44 Найдите полное количество различных термов, порождаемых элек-тронной конфигурацией ln.

4.45 Найдите полное количество различных термов каждой возможноймультиплетности, порождаемых электронной конфигурацией ln.

4.46 Найдите спин-орбитальное расщепление основного терма конфигу-раций d4, d5, d6 и т. д.

4.47 Найдите терм основного состояния для электронной конфигура-ции ln в случае LS- и j j-связи.

Page 182: КВАНТОВАЯ МЕХАНИКА МОЛЕКУЛ · PDF file2 ББК В22.31 УДК 544.11+539.183 isbn 978-5-94356-719-3 Стась Д.В., Плюснин В.Ф. Квантовая

182 ГЛАВА A. ЗАДАЧИ

4.48 Найдите термы, которые порождаются основными электроннымиконфигурациями атомов азота и висмута, рассматриваемых как предель-ные случаи очень легкого и очень тяжелого атомов. Укажите в обоихслучаях терм основного состояния.

4.49 Электронная конфигурация атома железа 3d64s2. В приближенииLS-связи найдите все порождаемые ею термы и укажите терм основногосостояния с учетом заметного для железа спин-орбитального взаимо-действия с константой |λ |. Из эксперимента известно, что для иона Fe+

основным термом является 6D9/2. Какой из валентных электронов атомажелеза (d или s) удаляется при такой ионизации?

4.50 Активным веществом твердотельного лазера на неодимовом стеклеявляется ион Nd3+, электронная конфигурация 4 f 3. Найдите терм основ-ного состояния иона в приближении LS-связи, укажите его кратность вы-рождения. Постройте схему уровней, возникающих из найденного термапри учете спин-орбитального взаимодействия с константой |λ |. Известно,что оптическая генерация в таком лазере возникает на переходе 4F3/2 →один из термов мультиплета основного состояния — разрешен/запрещентакой переход? Почему в этом случае для описания электронного состо-яния иона в твердом теле можно использовать язык свободного атома?

4.51 Укажите, в какие состояния разрешены электронные переходы изтермов 1S, 2P, 3D и т. д.

4.52 Определите, на сколько подуровней расщепляются термы 2D, 3F, 4Gи т. д. в сильном и слабом магнитных полях.

4.53 Найдите результат действия операторы L2, S2 и J2 на функциитермов 2P1/2 и 2P3/2 электронной конфигурации p1.

A.5 Многоэлектронные волновые функции

5.54 Выпишите все слейтеровские определители для метастабильногооднократно возбужденного (1s→2s) состояния атома гелия, для основно-го состояния атома лития.

5.55 Найдите полные волновые функции системы из двух электронов,находящихся на орбиталях n1p и n2p и приведите их к виду с расцеп-ленными орбитальными и спиновыми степенями свободы.

Page 183: КВАНТОВАЯ МЕХАНИКА МОЛЕКУЛ · PDF file2 ББК В22.31 УДК 544.11+539.183 isbn 978-5-94356-719-3 Стась Д.В., Плюснин В.Ф. Квантовая

A.6 КОРРЕЛЯЦИЯ АТОМНЫХ И МОЛЕКУЛЯРНЫХ ТЕРМОВ 183

5.56 Выполните процедуру последовательного перехода из базиса в ба-зис для электронных конфигураций p2 и d2: постройте из одноэлектрон-ных функций |l,m > мультипликативный базис |l1,m1 > |l2,m2 > → по-стройте базис приведенных по перестановочной симметрии функций сопределенными значениями ML и MS, → постройте базис |L,ML,S,MS >→ постройте базис функций полного момента |J,Jz,L,S >.

5.57 Найдите в явном виде волновую функцию основного состоянияэлектронной конфигурации p2 в случае LS-связи при учете слабого спин-орбитального взаимодействия.

5.58 Найдите в явном виде волновую функцию основного состоянияэлектронной конфигурации d2 в случае LS-связи при учете слабого спин-орбитального взаимодействия и еще более слабого внешнего постоянногомагнитного поля.

5.59 Выпишите в явном виде функции основного терма электроннойконфигурации p3 случае LS-связи.

5.60 Выпишите в явном виде волновые функции нижнего по энергиимультиплета конфигурации s1p1 в случае LS-связи.

5.61 Постройте в явном виде волновую функцию основного состоянияэлектронной конфигурации d2 в случае j j-связи в слабом внешнем по-стоянном магнитном поле.

5.62 Постройте в явном виде волновую функцию основного состоянияатома углерода.

5.63 Постройте в явном виде волновую функцию основного состояниятрехзарядного иона тантала.

A.6 Корреляция атомных и молекулярных

состояний

6.64 Найдите законы преобразования электронных состояний молекулы,получаемой при сведении пары атомов A и B с известными электронны-ми конфигурациями и находящихся в состояниях 2SA+1LA, 2SB+1LB, поддействием операции отражения в плоскости, содержащей ось молекулы.

Page 184: КВАНТОВАЯ МЕХАНИКА МОЛЕКУЛ · PDF file2 ББК В22.31 УДК 544.11+539.183 isbn 978-5-94356-719-3 Стась Д.В., Плюснин В.Ф. Квантовая

184 ГЛАВА A. ЗАДАЧИ

6.65 Найдите термы, которые могут возникнуть при сближении следу-ющих пар атомов в основных состояниях: C + O; O + H; O + F , B + N,S +Cl.

6.66 Определите кратности вырождения термов 1Σ+g ,

3Σ−u ,

3Π, 4Φu длядвухатомных молекул.

6.67 Найдите термы молекулы NO, образуемой при сведении атомов восновных электронных состояниях, с учетом слабого, но конечного вза-имодействия спин-ось (случай а по Гунду).

6.68 Имеется 10 одинаковых орбитальных моментов l = 1. Найдите,сколько раз встретится значение полного момента L = 1 (сколько раз-личных мультиплетов и сколько различных индивидуальных состояний)для такой системы, если не учитывать тождественности частиц.

6.69 Спиновое состояние системы из двух электронов описывается вол-новой функцией ψ = αβ . Укажите, какие значения полного спина и скакими вероятностями можно обнаружить при измерении.

6.70 Спиновая функция системы из N электронов имеет вид ψ = α1 . . .αn×βn+1 . . .βN (n штук α-функций и N −n штук β -функций). Найдите сред-нее значение квадрата суммарного спина, а также вероятности различ-ных значений полного спина S в частных случаях n = 1 и n = N −1.

6.71 Определите вид спинового оператора Sn проекции на ось~n для элек-трона. Чему равно среднее значение проекции спина на ось ~n в состояниис определенной проекцией спина на ось z: Sz = ±1

2? Каковы вероятностипроекции ±1

2 на направление ~n в указанных состояниях?

6.72 Найдите в явном виде спиновые функции для системы из трехнетождественных частиц со спинами 1, 1

2 и 12.

6.73 Спины N частиц, равные каждый s, складываются в результирую-щий спин S = Ns. Найдите суммарный спин любых 2, 3, . . .n частиц вуказанном состоянии.

6.74 Покажите действие операторов (S1x±S2x), (S1y±S2y) и (S1z±S2z) наспиновые функции синглетного и триплетного состояний двух электро-нов. Какие операторы смешивают эти состояния?

Page 185: КВАНТОВАЯ МЕХАНИКА МОЛЕКУЛ · PDF file2 ББК В22.31 УДК 544.11+539.183 isbn 978-5-94356-719-3 Стась Д.В., Плюснин В.Ф. Квантовая

A.6 КОРРЕЛЯЦИЯ АТОМНЫХ И МОЛЕКУЛЯРНЫХ ТЕРМОВ 185

6.75 Найдите число состояний с различными возможными значениямиполного спина и с различными возможными значениями проекции пол-ного спина для N тождественных частиц со спином 1

2 .

6.76 Три тождественных бозона со спином s = 1 находятся в одинаковыхорбитальных состояниях с волновой функцией ϕ(r). Определите число инапишите в явном виде нормированные волновые функции возможныхнезависимых состояний системы.

6.77 Система состоит из двух слабовзаимодействующих частиц с коор-динатными функциями ψ1 и ψ2. Запишите полную волновую функциюсистемы для следующих случаев: a) неразличимые частицы со спином 1

2;б) различимые частицы со спином 1

2; в) неразличимые частицы со спи-ном 1; г) частицы со спинами s1 = 1

2; s2 = 1.

6.78 Моменты двух слабовзаимодействующих подсистем l1 = l2 = l скла-дываются в полный момент L. Покажите, что коллективная волноваяфункция ΨL(m1,m2) состояния системы с определенным значением L вl1zl2z представлении обладает определенной симметрией по отношению кперестановке местами величин m1,m2. Как зависит от L характер такойсимметрии волновой функции?

6.79 Для системы из двух одинаковых частиц со спином s найдите чис-ло различных спиновых состояний, симметричных и антисимметричныхпо отношению к перестановкам спиновых переменных частиц. Укажи-те характер симметрии спиновых состояний с определенным значениемсуммарного спина обеих частиц.

6.80 Найдите термы молекулы N2, которые могут возникнуть при ееобразовании из а) двух атомов азота, имеющих основную электроннуюконфигурацию; б) двух атомов азота, один из которых имеет однократновозбужденную (s→p) электронную конфигурацию; с) ионов N+ и N−,имеющих основную электронную конфигурацию.

6.81 Найдите термы молекулы O2, которые могут возникнуть при ее об-разовании из а) двух атомов кислорода, имеющих основную электроннуюконфигурацию; б) двух атомов кислорода, один из которых имеет одно-кратно возбужденную (s → p) электронную конфигурацию; с) ионов O+

и O−, имеющих основную электронную конфигурацию.

Page 186: КВАНТОВАЯ МЕХАНИКА МОЛЕКУЛ · PDF file2 ББК В22.31 УДК 544.11+539.183 isbn 978-5-94356-719-3 Стась Д.В., Плюснин В.Ф. Квантовая

186 ГЛАВА A. ЗАДАЧИ

6.82 Найдите молекулярные термы, которые могут возникнуть при сбли-жении двух ионов золота Au3+, находящихся в основных состояниях, иобразовании кластера Au6+

2 .

6.83 Найдите молекулярные термы, которые могут возникнуть при сим-метричном сближении двух атомов фтора и гидрид-иона H− с образова-нием линейного иона FHF−.

6.84 Найдите возможные термы молекулярного иона водорода H+2 , а так-

же возможные для них значения момента орбитального момента электро-на относительно центра молекулы.

6.85 Сформулируйте и объясните правила отбора для двухатомных го-моядерных молекул для параллельной и перпендикулярной ориентациивозбуждающего поля.